contents · pdf filegai batu loncatan untuk membuktikan teorema-teorema lainnya. sebuah akibat...

155
Contents Contents iv List of Tables v List of Figures vi 1 Mathematics Proof Method Metode Pembuktian Matematis 1 1.1 Direct Proof Bukti Langsung ............................. 3 1.2 Indirect Proof Bukti Tak Langsung ........................... 6 2 Number Theory Teori Bilangan 11 2.1 Divisibility Keterbagian ............................... 11 2.2 Special Number Bilangan Khusus ............................ 14 2.2.1 A Prime and Composite Number Bilangan Prima dan Komposit ................ 14 2.2.2 Perfect Square Bilangan Kuadrat Sempurna ................. 15 i

Upload: vankhue

Post on 02-Feb-2018

221 views

Category:

Documents


1 download

TRANSCRIPT

Page 1: Contents · PDF filegai batu loncatan untuk membuktikan teorema-teorema lainnya. Sebuah akibat dari beberapa teorema atau lemma disebut dengan korolary. Sebuah pernyataan yang tidak

Contents

Contents iv

List of Tables v

List of Figures vi

1 Mathematics Proof MethodMetode Pembuktian Matematis 1

1.1 Direct Proof

Bukti Langsung . . . . . . . . . . . . . . . . . . . . . . . . . . . . . 3

1.2 Indirect Proof

Bukti Tak Langsung . . . . . . . . . . . . . . . . . . . . . . . . . . . 6

2 Number TheoryTeori Bilangan 11

2.1 Divisibility

Keterbagian . . . . . . . . . . . . . . . . . . . . . . . . . . . . . . . 11

2.2 Special Number

Bilangan Khusus . . . . . . . . . . . . . . . . . . . . . . . . . . . . 14

2.2.1 A Prime and Composite Number

Bilangan Prima dan Komposit . . . . . . . . . . . . . . . . 14

2.2.2 Perfect Square

Bilangan Kuadrat Sempurna . . . . . . . . . . . . . . . . . 15

i

Page 2: Contents · PDF filegai batu loncatan untuk membuktikan teorema-teorema lainnya. Sebuah akibat dari beberapa teorema atau lemma disebut dengan korolary. Sebuah pernyataan yang tidak

CONTENTS ii

2.3 GCD dan Algoritma Euclid . . . . . . . . . . . . . . . . . . . . . . 17

2.4 Modular Arithmetic

Modulo Aritmatik . . . . . . . . . . . . . . . . . . . . . . . . . . . . 19

2.5 Linear Diophantine Equations

Persamaan Linier Diophantin . . . . . . . . . . . . . . . . . . . . . 23

3 Algebra FunctionsFungsi Aljabar 28

3.1 Polynomials Inequality

Pertidaksamaan Polinomial . . . . . . . . . . . . . . . . . . . . . . 28

3.1.1 Inverse Function

Fungsi Invers . . . . . . . . . . . . . . . . . . . . . . . . . . 32

3.1.2 Arithmetic and Geometric Sequence

Barisan Aritmatik dan Geometrik . . . . . . . . . . . . . . 34

3.2 Arithmetic, Geometric, Harmonic, and Quadratic Means

Rataan Aritmatik, Geometrik, Harmonik dan

Kuadratik . . . . . . . . . . . . . . . . . . . . . . . . . . . . . . . . 43

3.3 The Polynomials and Remainder Theorem

Suku Banyak dan Teorema Sisa . . . . . . . . . . . . . . . . . . . . 47

3.3.1 Polynomials Division

Pembagian Suku Banyak . . . . . . . . . . . . . . . . . . . 47

3.3.2 Remainder Theorem

Teorema Sisa . . . . . . . . . . . . . . . . . . . . . . . . . . 49

3.3.3 Factor Theorem

Teorema faktor . . . . . . . . . . . . . . . . . . . . . . . . . 49

Page 3: Contents · PDF filegai batu loncatan untuk membuktikan teorema-teorema lainnya. Sebuah akibat dari beberapa teorema atau lemma disebut dengan korolary. Sebuah pernyataan yang tidak

CONTENTS iii

3.3.4 Properties of Polynomial Roots

Sifat-Sifat Akar-Akar Suku Banyak . . . . . . . . . . . . . . 50

4 TrigonometryTrigonometri 57

4.1 Trigonometric Function

Fungsi Trigonometri . . . . . . . . . . . . . . . . . . . . . . . . . . 58

4.1.1 Sine and Cosine Rule

Aturan Sinus dan Cosinus . . . . . . . . . . . . . . . . . . . 63

4.1.2 Formulas of Sum and Difference of Angles

Rumus-rumus Jumlah dan Selisih Sudut . . . . . . . . . . 65

4.1.3 Trigonometric Equation

Persamaan Trigonometri . . . . . . . . . . . . . . . . . . . . 69

4.2 Limit Fungsi . . . . . . . . . . . . . . . . . . . . . . . . . . . . . . . 71

4.2.1 Solution Techniques

Metode Penyelesaian . . . . . . . . . . . . . . . . . . . . . . 75

4.2.2 Limit of Algebraic Function

Limit Fungsi Aljabar . . . . . . . . . . . . . . . . . . . . . . 76

4.2.3 Limit of Trigonometric Function

Limit Fungsi Trigonometri . . . . . . . . . . . . . . . . . . . 79

5 Geometri 86

5.1 Segitiga . . . . . . . . . . . . . . . . . . . . . . . . . . . . . . . . . . 86

5.2 Segitiga . . . . . . . . . . . . . . . . . . . . . . . . . . . . . . . . . . 87

5.3 Lingkaran . . . . . . . . . . . . . . . . . . . . . . . . . . . . . . . . 96

Page 4: Contents · PDF filegai batu loncatan untuk membuktikan teorema-teorema lainnya. Sebuah akibat dari beberapa teorema atau lemma disebut dengan korolary. Sebuah pernyataan yang tidak

CONTENTS iv

6 Kombinatorika 125

6.1 Permutasi dan Kombinasi . . . . . . . . . . . . . . . . . . . . . . . 125

6.2 Prinsip Inklusi-Ekslusi dan Peluang . . . . . . . . . . . . . . . . . 126

6.3 Koefisien Binomial . . . . . . . . . . . . . . . . . . . . . . . . . . . 127

6.4 Prinsip Sarang Merpati . . . . . . . . . . . . . . . . . . . . . . . . . 131

6.5 Paritas . . . . . . . . . . . . . . . . . . . . . . . . . . . . . . . . . . 132

6.6 Relasi Rekurensi . . . . . . . . . . . . . . . . . . . . . . . . . . . . . 133

6.7 Soal-soal dan Pembahasan . . . . . . . . . . . . . . . . . . . . . . . 133

Page 5: Contents · PDF filegai batu loncatan untuk membuktikan teorema-teorema lainnya. Sebuah akibat dari beberapa teorema atau lemma disebut dengan korolary. Sebuah pernyataan yang tidak

List of Tables

4.1 Trigonometric quadrant system . . . . . . . . . . . . . . . . . . . . 59

4.2 The value of trigonometric functions for special angles . . . . . . 60

4.3 The value of trigonometric functions for any angle (Xo − α) . . . 61

4.4 The value of trigonometric functions for negative angles . . . . . 62

4.5 The value of trigonometric functions for any angle (Xo + α) . . . 62

v

Page 6: Contents · PDF filegai batu loncatan untuk membuktikan teorema-teorema lainnya. Sebuah akibat dari beberapa teorema atau lemma disebut dengan korolary. Sebuah pernyataan yang tidak

List of Figures

3.1 Squares in the circles. . . . . . . . . . . . . . . . . . . . . . . . . . . 37

4.1 The right triangle trigonometric system . . . . . . . . . . . . . . . 58

4.2 Triangle and circle of radius R . . . . . . . . . . . . . . . . . . . . . 64

4.3 Sum and Difference of Angles . . . . . . . . . . . . . . . . . . . . . 66

5.1 . . . . . . . . . . . . . . . . . . . . . . . . . . . . . . . . . . . . . . 86

vi

Page 7: Contents · PDF filegai batu loncatan untuk membuktikan teorema-teorema lainnya. Sebuah akibat dari beberapa teorema atau lemma disebut dengan korolary. Sebuah pernyataan yang tidak

¥¥¥¥¥¥ CHAPTER 1

Mathematics Proof MethodMetode Pembuktian Matematis

In mathematics, a proof is a convincing demonstration (within the accepted stan-

dards of a field study) to show that some mathematical statement is necessarily

true. Proofs are obtained from deductive reasoning, rather than from inductive

or empirical arguments. That is, a proof must demonstrate that a statement is

true in all cases, without a single exception.

Dalam matematika, bukti adalah suatu demonstarasi meyakinkan (mengikuti

beberapa standar yang diterima dari suatu bidang kajian tertentu) untuk me-

nunjukkan bahwa pernyataan matematika itu betul-betul benar. Pembuktian

lebih diperoleh dari penarikan kesimpulan secara deduktif dibandingkan den-

gan penarikan kesimpulan yang secara induktif atau empiris. Dengan demikian,

bukti harus menunjukkan bahwa sebuah pernyataan itu adalah benar disegala

hal tanpa suatu perkecualian sedikitpun.

The statement that is proved is often called a theorem. Once a theorem is proved,

it can be used as the basis to prove further statements. A theorem may also be

referred to as a lemma, that is a sub theorem, especially if it is intended for use

as a stepping stone in the proof of another theorem. An implication of theorems

or lemmas is known as a corollary. An unproved proposition that is believed to

be true is known as a conjecture.

Sebuah pernyataan yang sudah terbuktikan disebut dengan teorema. Sekali teo-

rema itu terbuktikan maka hal ini dapat digunakan sebagai dasar untuk mem-

buktikan pernyataan-pernyataan selanjutnya. Kadangkala teorema disebut juga

1

Page 8: Contents · PDF filegai batu loncatan untuk membuktikan teorema-teorema lainnya. Sebuah akibat dari beberapa teorema atau lemma disebut dengan korolary. Sebuah pernyataan yang tidak

Chapter 1. Mathematics Proof Method 2

dengan lemma, bagian kecil dari teorema, khusunya jika hal ini digunakan seba-

gai batu loncatan untuk membuktikan teorema-teorema lainnya. Sebuah akibat

dari beberapa teorema atau lemma disebut dengan korolary. Sebuah pernyataan

yang tidak terbuktikan namun diyakini kebenarannya dikenal dengan istilah

konjektur.

Proofs employ logic but usually include some amount of natural language which

usually admits some ambiguity. In fact, the vast majority of proofs in writ-

ten mathematics can be considered as applications of rigorous informal logic.

Purely formal proofs, written in symbolic language instead of natural language,

are considered in proof theory. The distinction between formal and informal

proofs has led to much examination of current and historical mathematical prac-

tice, quasi-empiricism in mathematics, and so-called folk mathematics (in both

senses of that term). Therefore, the philosophy of mathematics is concerned with

the role of language and logic in proofs, and mathematics as a language.

Beberapa pembuktian matematika menggunakan logika, namun biasanya juga

melibatkan beberapa bahasa biasa yang kadangkala memunculkan dua arti. Na-

mun demikian fakta menunjukkan bahwa hampir semua pembuktian dalam

pernyataan matematika dapat dikatakan sebagai suatu aplikasi dari logika mate-

matika informal. Dalam pembuktian formal yang asli, penulisan dengan simbol-

simbol matematika dibandingkan dengan penulisan dengan bahasa biasa lebih

dipilih dalam teori pembuktian. Perbedaan antara pembuktian formal dan in-

formal telah mendasari beberapa evaluasi matematika dan sejarah latihan mate-

matika akhir-akhir ini, termasuk juga matematika semi empiris, sehingga kita

mempunyai istilah yang dikenal dengan matematika untuk umum (terhadap

kedua istilah itu). Dengan demikian, filosofi matematika sesungguhnya adalah

terkait dengan bagaimana perananan bahasa dan logika dalam matematika itu

sendiri, sehingga matematika menjadi suatu bahasa.

Page 9: Contents · PDF filegai batu loncatan untuk membuktikan teorema-teorema lainnya. Sebuah akibat dari beberapa teorema atau lemma disebut dengan korolary. Sebuah pernyataan yang tidak

Chapter 1. Mathematics Proof Method 3

Mathematics statement can be either true or false. A statement which is always

true is called a tautology, a statement which is always false is called a contradic-

tion. To prove a truth of mathematics statement, we need a proof technique. Ba-

sically, there are two types of proof technique, namely direct proof and indirect

proof. In the following, we describe how the two techniques are implemented

in proving the truth of a statement.

Pernyataan matematika dapat bernilai benar atau salah. Suatu pernyataan yang

selalu bernilai benar disebut tautologi, sedangkan pernyataan yang selalu berni-

lai salah disebut kontradiksi. Untuk membuktikan kebenaran suatu pernyataan

matematika dibutuhkan suatu metode pembuktian. Pada prinsipnya terdapat

dua metode pembuktian, yaitu bukti langsung dan bukti tak langsung. Berikut

ini akan dijelaskan bagaimana kedua metode itu diterapkan untuk membuk-

tikan kebenaran suatu pernyataan.

1.1 Direct ProofBukti Langsung

In this case, to prove a truth of mathematics statement is utilized a direct way

with a particular technique in direction of having a conclusion. In general, there

are three direct proofs, namely one way proof (implication), two ways proof

(biimplication/equivalence) and mathematics induction. Some examples of the

use of those methods are presented in the following.

Dalam hal ini, pembuktian dalam kebenaran matematika dibuktikan dengan

cara langsung dengan teknik-teknik tertentu sampai mencapai kesimpulan. Se-

cara umum, terdapat tiga pembuktian langsung yaitu pembuktian satu arah

(implikasi), pembuktian dua arah (biimplikasi/ekuivalensi) dan induksi matematika.

Beberapa contoh penggunaan metode tersebut adalah sebagai berikut.

Page 10: Contents · PDF filegai batu loncatan untuk membuktikan teorema-teorema lainnya. Sebuah akibat dari beberapa teorema atau lemma disebut dengan korolary. Sebuah pernyataan yang tidak

Chapter 1. Mathematics Proof Method 4

Lemma 1.1.1 If n is a natural number then 1 + 2 + 3 + · · ·+ n = n2(n + 1) for any n.

Lema 1.1.1 Jika n suatu bilangan asli maka 1 + 2 + 3 + · · · + n = n2(n + 1) untuk

setiap n.

Proof. Implication Proof: Suppose Un = n2 =⇒ Un+1 = (n + 1)2 = n2 + 2n +

1 =⇒ Un+1 − Un = 2n + 1. Then we have

Bukti. Pembuktian Implikasi: Misal Un = n2 =⇒ Un+1 = (n + 1)2 = n2 + 2n +

1 =⇒ Un+1 − Un = 2n + 1. Maka didapat

Un+1 − Un = 2n + 1

Un − Un−1 = 2(n− 1) + 1...

U3 − U2 = 2 · 2 + 1

U2 − U1 = 2 · 1 + 1.

Sum up the n equations above, we get

Jumlahkan seluruh n persamaan di atas maka akan didapat

Un+1 − U1 = 2(1 + 2 + · · ·+ n) + (1 + 1 + · · ·+ 1)

1 + 2 + · · ·+ n =n2 + n

2

Therefore

Dengan demikian 1 + 2 + · · ·+ n = n2(n + 1). 2

Lemma 1.1.2 Let a, b be two integer numbers and n be a positif integer. For any integern, n|a and n|b will give the same remainder if and only if n|(a− b).

Lema 1.1.2 Diberikan dua bilangan bulat a, b dan bilangan bulat positip n. Untuksebarang bilangan bulat n, a

ndan b

nakan mempunyai sisa yang sama jika dan hanya

jika n|(a− b).

Page 11: Contents · PDF filegai batu loncatan untuk membuktikan teorema-teorema lainnya. Sebuah akibat dari beberapa teorema atau lemma disebut dengan korolary. Sebuah pernyataan yang tidak

Chapter 1. Mathematics Proof Method 5

Proof. Equivalence Proof:

Bukti. Pembuktian Ekuivalensi:

(=⇒)

Let s be a remainder of a and b divided by n, we have a = kn + s and b = jn + s

for 0 ≤ s ≤ n dan k, j ∈ I .

Misal sisa pembagian bilangan a dan b oleh n adalah s maka a = kn + s dan

b = jn + s, dengan 0 ≤ s ≤ n dan k, j ∈ I .

a− b = (kn + s)− (jn + s)

= (kn− jn)

= (k − j)n.

Since k, j ∈ I , k − j = p where p is also integer, and a − b = pn, it shows that

n|(a− b).

Karena k, j ∈ I maka k−j = p dimana p juga bilangan bulat, sehingga a−b = pn,

yang artinya n|(a− b).

(⇐=)

Suppose that n|(a− b). We will prove that a and b will give the same remainder

when they are divided by n. Let a = kn + s1 and b = jn + s2 for 0 ≤ s1 ≤ n and

0 ≤ s2 ≤ n, we will show that s1 = s2.

Misal n|(a − b). Akan dibuktikan bahwa a dan b akan mempunyai sisa yang

sama bila dibagi n. Misal a = kn + s1 dan b = jn + s2 untuk 0 ≤ s1 ≤ n dan

0 ≤ s2 ≤ n, maka akan ditunjukkan s1 = s2.

a− b = pn

a = b + pn

= (jn + s2) + pn

= (j + p)n + s2

= kn + s2.

Page 12: Contents · PDF filegai batu loncatan untuk membuktikan teorema-teorema lainnya. Sebuah akibat dari beberapa teorema atau lemma disebut dengan korolary. Sebuah pernyataan yang tidak

Chapter 1. Mathematics Proof Method 6

Since the remainder of n|a is single, we have s2 = s1

Karena sisa dari n|a adalah tunggal maka s2 = s1. 2

Lemma 1.1.3 Prove that 3|(22n − 1) for n ≥ 1.

Lema 1.1.3 Buktikan bahwa 3|(22n − 1) untuk n ≥ 1.

Proof. Mathematics Induction. For n = 1 −→ 3|(22−1) −→ 3|3 (true). Suppose

it is true for n = k, we have 3|(22k − 1). Thus, is that true for n = k + 1?

Bukti. Induksi Matematika. Untuk n = 1 −→ 3|(22 − 1) −→ 3|3 (benar). Misal

benar untuk n = k maka 3|(22k − 1). Selanjutnya apakah benar untuk n = k + 1?

3|(22(k+1) − 1) ?

3|(22k+2 − 1)

3|(22k · 22 − 1)

3|(22k · 22 − 22 + 3)

3|(22(22k − 1) + 3).

Since 3|22(22k − 1) and 3|3. It follows that 3|(22(22k − 1) + 3), hence it is also true

for n = k + 1.

Karena 3|22(22k− 1) dan 3|3 maka 3|(22(22k− 1)+3). Sehingga hal ini juga benar

untuk n = k + 1. 2

1.2 Indirect ProofBukti Tak Langsung

In this proof technique, we do not start involving the existing facts in direc-

tion of having a conclusion. We start the prof even from the opposite facts. In

Page 13: Contents · PDF filegai batu loncatan untuk membuktikan teorema-teorema lainnya. Sebuah akibat dari beberapa teorema atau lemma disebut dengan korolary. Sebuah pernyataan yang tidak

Chapter 1. Mathematics Proof Method 7

general, there are two indirect proofs, namely contradictive proof and contra-

positive proof. Contradictive proof is also called reductio ad absurdum proof.

For instance, we will proof that A is true, we start the proof by assuming that A

is not true. Contrapositive proof is obtained by finding a contraposition of an

implication statement in logic math. For instance, we will proof that p −→ q is

true, we start the proof by determining the contraposition of p −→ (q ∨ r), i.e.

∼ (q ∨ r) −→∼ p. As we know, in logic math p −→ (q ∨ r) =∼ (q ∨ r) −→∼ p. In

the following, we present how to use these methods.

Dalam teknik pembuktian ini, fakta-fakta yang ada tidak digunakan secara lang-

sung untuk menuju pada kesimpulan. Pembuktia dimulai justru dari hal se-

baliknya. Secara umum terdapat dua pembuktian tak langsung, yakni pembuk-

tian kontradiksi dan pembuktian kontraposisi. Pembuktian kontradiksi disebut juga

pembuktian kemustahilan. Misal yang akan dibuktikan adalah benarnya perny-

ataan A, maka pembuktian dimulai dengan mengandaikan bahwa A adalah

salah. Pembuktian kontraposisi diperoleh dari menentukan kontraposisi dari

sebuah pernyataan implikasi dalam logika matematika. Misal akan dibuktikan

bahwa pernyataan p =⇒ (q∨r) benar, maka pembuktian diawali dengan menen-

tukan kontraposisi dari p =⇒ (q ∨ r), yaitu ∼ (q ∨ r) =⇒∼ p. Seperti yang kita

ketahui, dalam logika matematika p =⇒ (q ∨ r) =∼ (q ∨ r) =⇒∼ p. Berikut ini

kita jelaskan bagaimana menggunakan metode pembuktian ini.

Lemma 1.2.1 Prove that√

2 is an irrational number.

Lema 1.2.1 Buktikan bahwa√

2 adalah suatu bilangan irasional.

Proof. Contradictive Proof. Suppose that√

2 is a rational number. We will

have√

2 = ab, where a

bis a simplified form. By squaring the two sides, we

obtain a2 = 2b2. It follows that a2 is even which implies that a is even. Suppose

a = 2k −→, we have (2k)2 = 2b2 −→ b2 = 2k2 which implies that b is also even.

Page 14: Contents · PDF filegai batu loncatan untuk membuktikan teorema-teorema lainnya. Sebuah akibat dari beberapa teorema atau lemma disebut dengan korolary. Sebuah pernyataan yang tidak

Chapter 1. Mathematics Proof Method 8

Since both a and b are even, ab

is not a simplified form any more, which is a

contradiction.

Bukti. Pembuktian kontradiksi. Misal√

2 adalah bilangan rasional maka√

2 =ab

(dimana ab

adalah bentuk yang paling sederhana). Kuadratkan kedua ruasnya

diperoleh a2 = 2b2, sehingga a2 adalah bilangan genap dan a pasti genap. Misal

a = 2k −→ (2k)2 = 2b2 −→ b2 = 2k2 yang berakibat b juga genap. Bila a dan b

adalah sama-sama genap maka ab

bukan bentuk yang paling sederhana lagi, ini

jelas kontradiktif. 2

Lemma 1.2.2 Prove that if m + n ≥ 73 then m ≥ 37 or n ≥ 37, for m,n ∈ I .

Lema 1.2.2 Buktikan bahwa jika m + n ≥ 73 maka m ≥ 37 atau n ≥ 37, untukm,n ∈ I .

Proof. Contrapositive Proof. Consider the statement as a logic expression p =⇒(q ∨ r), where p ≡ m + n ≥ 73, q ≡ m ≥ 37 and r ≡ n ≥ 37. The contraposition

of the statement is ∼ (q ∨ r) =⇒∼ p or (∼ q∧ ∼ r) =⇒∼ p. Therefore, to prove

the truth of the statement above, we can start proving that if m < 37 and n < 37

then m+n < 73. Suppose the two any numbers are m ≤ 36 dan n ≤ 36, we have

m + n ≤ 36 + 36 −→ m + n ≤ 72 −→ m + n < 73. It completes the prove that if

m + n ≥ 73 then m ≥ 37 or n ≥ 37, for m,n ∈ I .

Bukti. Pembuktian kontraposisi. Misal pernyataan tersebut disajikan dalam

p =⇒ (q ∨ r), dimana p ≡ m + n ≥ 73, q ≡ m ≥ 37 dan r ≡ n ≥ 37. Kontraposisi

pernyataan tersebut adalah ∼ (q ∨ r) =⇒∼ p atau (∼ q∧ ∼ r) =⇒∼ p. Sehingga

untuk membuktikan kebenaran pernyataan di atas dapat dimulai dengan mem-

buktikan bahwa jika m < 37 dan n < 37 maka m + n < 73. Misal sebarang dua

bilangan itu adalah m ≤ 36 dan n ≤ 36, maka m + n ≤ 36 + 36 −→ m + n ≤72 −→ m + n < 73, terbukti. Sehingga terbukti pulalah kebenaran pernyataan

semula. 2

Page 15: Contents · PDF filegai batu loncatan untuk membuktikan teorema-teorema lainnya. Sebuah akibat dari beberapa teorema atau lemma disebut dengan korolary. Sebuah pernyataan yang tidak

Chapter 1. Mathematics Proof Method 9

PROBLEMS AND SOLUTIONS

SOAL-SOAL DAN PEMBAHASAN

1. Prove that (an + b)m = bm mod n.

Buktikan bahwa (an + b)m = bm mod n.

Solution. Direct Proof. To prove (an+ b)m = bm mod n, we need to show

that there exists an integer k such that (an + b)m − bm = kn.

Solusi. Bukti Langsung. Untuk membuktikan (an + b)m = bm mod n

perlu ditunjukkan bahwa terdapat bilangan bulat k sedemikian hingga

(an + b)m − bm = kn.

(an + b)m − bm =

((an)m + m(an)m−1b + ... + m(an)bm−1 + bm

)− bn

= (an)m + m(an)m−1b + ... + m(an)bm−1 + bm − bm

= (an)m + m(an)m−1b + ... + m(an)bm−1

=

((a)mnm−1 + m(a)m−1nm−1b + ... + m(a)bm−1bm−1

)n

Let k =

((a)mnm−1 + m(a)m−1nm−1b + ... + m(a)bm−1bm−1

), hence we get

(an + b)m − bm = kn. Therefore (an + b)m = bm mod n.

Bila k =

((a)mnm−1 + m(a)m−1nm−1b + ... + m(a)bm−1bm−1

), maka didapat

(an + b)m − bm = kn. Sehingga (an + b)m = bm mod n.

2. If p is a prime number and p|a1a2...an then p|ai for any 1 ≤ i ≤ n.

Jika p adalah bilangan prima dan p|a1a2...an maka p|ai untuk sebarang 1 ≤i ≤ n.

Solution. Mathematics Induction. Let P (k) be a representation state-

ment. Step I: If p|a1, then it is obvious that P |ai for 1 ≤ i ≤ 1. Step II:

Page 16: Contents · PDF filegai batu loncatan untuk membuktikan teorema-teorema lainnya. Sebuah akibat dari beberapa teorema atau lemma disebut dengan korolary. Sebuah pernyataan yang tidak

Chapter 1. Mathematics Proof Method 10

Assume it is true for P (k), that is if p|a1a2...ak then p|ai for any 1 ≤ i ≤ k.

We will show that P (k+1) is also true. Since p|a1a2...akak+1 and p is a prime

number, it holds p|a1a2...ak or p|a(k+1). From the two possibilities, we have

p|ai for 1 ≤ i ≤ k + 1 as P (k) is true. It concludes that P (k + 1) is also true.

Solusi. Induksi Matematika. Misal P (k) adalah representasi dari perny-

ataan tersebut. Langkah I : Jika p|a1, jelas bahwa P |ai untuk suatu 1 ≤i ≤ 1. Langkah II : Misalkan p(k) benar, artinya jika p|a1a2...ak maka p|ai

untuk suatu 1 ≤ i ≤ k. Akan dibuktikan bahwa P (k + 1) benar. Dike-

tahui p|a1a2...akak+1, maka karena p bilangan prima berlaku p|a1a2...ak atau

P |a(k+1). Dari kedua kemungkinan ini dikombinasikan, karena P (k) berni-

lai benar, maka didapatkan untuk suatu 1 ≤ i ≤ k +1. Jadi terbukti bahwa

P (k + 1) bernilai benar.

3. Prove that if xm is divisible by a prime p, then x is also divisible by p.

Bukatikan bahwa jika xm habis dibagi bilangan prima p, maka x habis

dibagi p.

Solution. Since p divides xm, we have p|xm or p|x.xm−1. Since p is a prime,

we have p|x or p|xm−1. Suppose p does not divide x, it follows p|xm−1 or

p|x.xm−2. Repeat the process, we will be able to show that p|x.

Solusi. Karena p membagi habis xm, didapat p|xm atau p|x.xm−1. Karena

p adalah bilangan prima maka p|x atau p|xm−1. Andaikan p tidak habis

membagi x, maka p|xm−1 atau p|x.xm−2. Jika proses diteruskan maka akan

didapatkan bahwa p|x.

Page 17: Contents · PDF filegai batu loncatan untuk membuktikan teorema-teorema lainnya. Sebuah akibat dari beberapa teorema atau lemma disebut dengan korolary. Sebuah pernyataan yang tidak

¥¥¥¥¥¥ CHAPTER 2

Number TheoryTeori Bilangan

2.1 DivisibilityKeterbagian

When 13 is divided by 5, it will give quotient 2 and remainder 3, denoted by135

= 2 + 35

or 13 = 2 × 5 + 3. In general, for any positive integers a and b there

exists a unique pair (q, r) of nonnegative integers such that b = q × a + r and

0 < r < a. We say that q is the quotient and r is the remainder when b is divided

by a. If r = 0 then we say that b is divisible by a or a divides b, denoted by a| b.

If b is not divisible by a then we denote as a - b.

Jika 13 dibagi 5 maka hasil baginya 2 dan sisanya 3 dan ditulis: 135

= 2 + 35

atau

13 = 2×5+3. Secara umum, apabila a bilangan bulat dan b bilangan bulat positif,

maka ada tepat satu bilangan bulat q dan r sedemikian hingga b = q× a + r dan

0 < r < a. Dalam hal ini, q disebut hasil bagi dan r adalah sisa pembagian bila b

dibagi a. Jika r = 0 maka dikatakan b habis dibagi a dan ditulis a| b. Bila b tidak

habis dibagi a maka ditulis a - b.

Lemma 2.1.1 (1) If a|b then a|bc for any c ∈ I ; (2) If a| b and b| c then a| c; (3) If ab| cthen a| c and b| c; (4) If a|b and b|a then a = ±b; (5) If a| b and a| c then a| (bx + cy) forany integers x and y.

11

Page 18: Contents · PDF filegai batu loncatan untuk membuktikan teorema-teorema lainnya. Sebuah akibat dari beberapa teorema atau lemma disebut dengan korolary. Sebuah pernyataan yang tidak

Chapter 2. Number Theory 12

Lema 2.1.1 (1) Jika a|b maka a|bc untuk sebarang c ∈ I ; (2) jika a| b dan b| c maka a| c;(3) jika ab| c maka a| c dan b| c; (4) jika a|b dan b|a maka a = ±b; (5) jika a| b dan a| cmaka a| (bx + cy) untuk sebarang bilangan bulat x dan y.

Proof. Property (1): If a| b then b = ka, and if b| c then c = lb = l(ka) = (kl)a.

It implies that a| c. Property (3): If a| b then b = ka −→ bx = kxa, if a| c then

c = la −→ cy = kya. We have bx + cy = (kx + ly)a, therefore a| (bx + cy).

Bukti. Sifat (1): a| b maka b = ka, dan b| c maka c = lb = l(ka) = (kl)a maka

a| c. Sifat (3): a| b maka b = ka −→ bx = kxa, dan a| c maka c = la −→ cy = kya.

Kemudian bx + cy = (kx + ly)a maka a| (bx + cy). 2

Lemma 2.1.2 A number a is divisible by 2n if the last n digit of the number is divisibleby 2n.

Lema 2.1.2 Suatu bilangan a habis dibagi 2n jika n angka terkhir dari bilangan tersebuthabis dibagi 2n.

Proof. Suppose n = 1, thus 2 divides a if the last digit of the number is di-

visible by 2. Let a = . . . a3 a2 a1 a0, we have a = 10(. . . a3 a2 a1) + a0. Since

2| 10(. . . a3 a2 a1), a0 must be divisible by 2 to have 2|a.

Bukti. Misal n = 1, berarti a habis dibagi 2 jika angka terakhir dari bilangan

tersebut habis dibadi 2. Misal a = . . . a3 , a2 a1 a0 maka a = 10(. . . a3 , a2 a1) + a0.

Karena 2| 10(. . . a3 , a2 a1) a0 harus habis dibagi 2 untuk memperoleh 2|a. 2

Example. Is 173332 divisible by 8? Since 23| 332, we have 8| 173332.

Contoh. Apakah 173332 habis dibagi oleh 8? Karena 23| 332 maka 8| 173332.

Page 19: Contents · PDF filegai batu loncatan untuk membuktikan teorema-teorema lainnya. Sebuah akibat dari beberapa teorema atau lemma disebut dengan korolary. Sebuah pernyataan yang tidak

Chapter 2. Number Theory 13

Example. Is 13 + 23 + · · ·+ 1003 divisible by 7? False, as we have 13 + 23 + · · ·+1003 = (1 + 2 + 3 + · · ·+ 100)2 = (5050)2 = 25502500 and 7 - 25502500.

Contoh. Apakah 13 +23 + · · ·+1003 habis dibagi 7? Tidak, karena 13 +23 + · · ·+1003 = (1 + 2 + 3 + · · ·+ 100)2 = (5050)2 = 25502500 dan 7 - 25502500.

Lemma 2.1.3 A number a = anan−1 . . . a1a0 is respectively divisible by 3,9 and 11 ifthe sum of its digits satisfies respectively the following: (an+an−1+an−2+· · ·+a1+a0)

is divisible by 3; (an + an−1 + an−2 + · · ·+ a1 + a0) is divisible by 9; and (an − an−1 +

an−2 − an−3 + . . . ) is divisible by 11.

Lema 2.1.3 Suatu bilangan a = anan−1 . . . a1a0 berturut-turut habis dibagi 3,9 dan11 jika jumlah angka-angkanya memenuhi masing-masing sifat berikut: (an + an−1 +

an−2 + · · ·+ a1 + a0) habis dibagi 3; (an + an−1 + an−2 + · · ·+ a1 + a0) habis dibagi 9;dan (an − an−1 + an−2 − an−3 + . . . ) habis dibagi 11.

Proof.

Bukti.

a = anan−1 . . . a1a0

= an × 10n + an−1 × 10n−1 + . . . a1 × 101 + a0 × 100

= an × (9 + 1)n + an−1 × (9 + 1)n−1 + . . . a1 × (9 + 1)1 + a0 × (9 + 1)0

= an[9n + n · 9n−1 + · · ·+ 9n] + an + an−1[9n−1 + (n− 1) · 9n−2 +

· · ·+ 9(n− 1)] + an−1 + · · ·+ 9a1 + a1 + a0

= an[9n + n · 9n−1 + · · ·+ 9n] + an−1[9n−1 + (n− 1) · 9n−2 + · · ·+

9(n− 1)] + · · ·+ 9a1 + an + an−1 + · · ·+ a1 + a0

= K(a) + Q(a)

Since (3 ∧ 9)|K(a), it must be (3 ∧ 9)|Q(a) to have (3 ∧ 9)| a .

Karena (3 ∧ 9)|K(a) maka haruslah (3 ∧ 9)|Q(a) supaya (3 ∧ 9)| a . 2

Page 20: Contents · PDF filegai batu loncatan untuk membuktikan teorema-teorema lainnya. Sebuah akibat dari beberapa teorema atau lemma disebut dengan korolary. Sebuah pernyataan yang tidak

Chapter 2. Number Theory 14

2.2 Special NumberBilangan Khusus

2.2.1 A Prime and Composite NumberBilangan Prima dan Komposit

The integer p > 1 is called a prime number if there is no integer d with d > 1

and d 6= p such that d|p. Any integer n > 1 has at least one prime divisor. If n is

a prime, then that prime divisor is n itself. An integer n > 1 that is not a prime

is called composite.

Bilangan bulat p > 1 disebut sebuah bilangan prima jika tidak ada bilangan

bulat d dimana d > 1 dan d 6= p sedemikian hingga d|p. Setiap bilangan bulat

n > 1 mempunyai paling sedikit satu pembagi prima. Jika n adalah bilangan

prima, maka pembagi primanya adalah bilngan n itu sendiri. Sebuah bilangan

bulat n > 1 yang bukan bilngan prima disebut bilangan komposit.

Theorem 2.2.1 Eratosthenes. For any composite n, there exists a prime p such thatp|n and p ≤ √

n. In other words ”If there does not exist a prime p which divides n,where p ≤ √

n, then n is a prime”.

Teorema 2.2.1 Eratosthenes. Untuk setiap bilangan komposit n ada bilangan primap sehingga p|n dan p ≤ √

n. Dengan kata lain ”Jika tidak ada bilangan prima p yangdapat membagi n dengan p ≤ √

n maka n adalah bilangan prima”.

Example. Are 157 and 221 prime numbers?. The primes which are less than√157 are 2, 3, 5, 7, 11. Since none of them divides 157, then 157 is a prime. The

primes which are less than√

221 are 2, 3, 5, 7, 11, 13. Since 13| 221 then 221 is a

composite number.

Page 21: Contents · PDF filegai batu loncatan untuk membuktikan teorema-teorema lainnya. Sebuah akibat dari beberapa teorema atau lemma disebut dengan korolary. Sebuah pernyataan yang tidak

Chapter 2. Number Theory 15

Contoh. Apakah bilangan 157 dan 221 bilangan prima?. Bilangan-bilangan

prima yang lebih kecil dari√

157 adalah 2, 3, 5, 7, 11. Karena tidak ada satupun

dari bilangan- bilangan prima itu yang dapat membagi 157, maka 157 meru-

pakan bilangan prima. Kemudian bilangan - bilangan prima yang lebih kecil

dari√

221 adalah 2, 3, 5, 7, 11, 13. Karena 13| 221 maka 221 merupakan bilangan

komposit.

2.2.2 Perfect SquareBilangan Kuadrat Sempurna

Any perfect square satisfies the following three properties:

• The possible ones of perfect square number is either 0, 1, 4, 5, 6, dan 9.

• If 4 divides a perfect square then the remainder is either 0 or 1.

• If p is a prime and p|x2 then p| z, where z = x2/p.

Sebarang bilangan kuadrat sempurna memenuhi tiga sifat berikut ini:

• Angka satuan yang mungkin untuk bilangan kuadrat sempurna adalah 0,

1, 4, 5, 6, atau 9.

• Jika 4 membagi bilangan kuadrat sempurna maka sisanya 0 atau 1.

• Jika p bilangan prima dan p| x2 maka p| z, dimana z = x2/p.

Example. Obtain a perfect square whose digits are k, k + 1, k + 2, 3k, k + 3.

Contoh. Carilah suatu bilangan kuadrat sempurna yang angka-angkanya berturut-

turut adalah k, k + 1, k + 2, 3k, k + 3.

Page 22: Contents · PDF filegai batu loncatan untuk membuktikan teorema-teorema lainnya. Sebuah akibat dari beberapa teorema atau lemma disebut dengan korolary. Sebuah pernyataan yang tidak

Chapter 2. Number Theory 16

Solution. The ones of the number is k+3, it follows that k can be either 1, 2, 3 or

6. Whilst the tens is 3k, it follows that k can be either 0, 1, 2 or 3. They imply that

the possible k is either 1, 2 or 3 which give perfect square numbers 12334, 23465

or 34596. Since the remainder of 12334 divided by 4 is 2, it gives that 12334 is

not a perfect square. The remainder of 23465 divided by 4 is 1 and 5|23465, but

5 - 4693, so that 23465 is not a perfect square. Now, 4|34596, and we have the

following

Solusi. Angka satuan bilangan kuadrat ini adalah k+3 sehingga k yang mung-

kin adalah 1, 2, 3, 6. Sedangkan angka puluhannya adalah 3k maka k yang mung-

kin adalah 0, 1, 2, 3. Dari kedua kemungkinan ini diperoleh k yang mungkin

adalah 1, 2, 3, dengan demikian bilangan kuadrat yang mungkin adalah 12334,

23465, 34596. Karena 12334 dibagi 4 bersisa 2 maka 12334 bukan bilangan kuadrat.

Bilangan 23465 dibagi 4 bersisa 1 dan 5|23465, akan tetapi 5 - 4693 maka 23465

bukan bilangan kuadrat. Sekarang, bilangan 4|34596, dan berikut ini berlaku

2 | 34596

2 | 17298

3 | 8649

3 | 2883

31 | 961

31 | 31

Therefore, 34596 = 22 · 32 · 312 = 1862 which is a perfect square.

Sehingga 34596 = 22 · 32 · 312 = 1862 yang merupakan bilangan kuadrat.

Page 23: Contents · PDF filegai batu loncatan untuk membuktikan teorema-teorema lainnya. Sebuah akibat dari beberapa teorema atau lemma disebut dengan korolary. Sebuah pernyataan yang tidak

Chapter 2. Number Theory 17

2.3 GCD dan Algoritma Euclid

Let a, b be any integers. An integer d satisfying d| a and d| b is called a common

divisor of a and b. The biggest (respectively, smallest) value of d is called Greater

Common Divisor/GCD, denoted by GCD(a, b)) (respectively, Least Common Mul-

tiple/LCM, denoted by LCM(a, b))). Several techniques have been developed to

obtain GCD or LCM, one of them is Euclidean algorithm.

Misal a dan b sembarang bilangan bulat. Bilangan bulat d yang memenuhi sifat

d| a dan d| b disebut faktor persekutuan dari a dan b. Nilai terbesar dari d disebut

faktor persekutuan terbesar atau FPB dan ditulis dengan FPB(a, b) sedangkan

nilai terkecil dari d disebut kelipatan persekutuan terkecil atau KPK dan ditulis

KPK(a, b). Beberapa tekni telah dikembangkan untuk menentukan FPB dan

KPK, salah satunya adalah algoritma Euclid.

Algorithm 2.3.1 Euclidean Algorithm. Given two integer numbers a and b with a >

b > 0, GCD(a, b) can be obtained by repeating the following algorithm:

a = q1b + r1; 0 < r1 < b

b = q2r1 + r2; 0 < r2 < r1

r1 = q3r2 + r3; 0 < r3 < r2

...

rn−2 = qnrn−1 + rn; 0 < rn < rn−1

rn−1 = qn+1rn + 0

If rn is the last divisor of the division process which gives a remainder 0 then rn isGCD(a, b).

Algoritma 2.3.1 Algoritma Euclide. Diberikan dua bilangan bulat a dan b dengan

Page 24: Contents · PDF filegai batu loncatan untuk membuktikan teorema-teorema lainnya. Sebuah akibat dari beberapa teorema atau lemma disebut dengan korolary. Sebuah pernyataan yang tidak

Chapter 2. Number Theory 18

a > b > 0, maka GCD(a, b) bisa dicari dengan mengulang algoritma berikut:

a = q1b + r1; 0 < r1 < b

b = q2r1 + r2; 0 < r2 < r1

r1 = q3r2 + r3; 0 < r3 < r2

...

rn−2 = qnrn−1 + rn; 0 < rn < rn−1

rn−1 = qn+1rn + 0

Jika rn merupakan pembagi terakhir dari pembagian di atas yang memberikan sisa 0

maka rn merupakan GCD(a, b).

Example. Determine GCD(4840, 1512). Using Euclidean Algorithm, the solu-

tion is the following:

Contoh. Tentukan GCD(4840, 1512). Dengan menggunakan Algoritma Euclid

maka solusinya adalah sebagai berikut:

4840 = 3× 1512 + 304

1512 = 4× 304 + 296

304 = 1× 296 + 8

296 = 37× 8 + 0

Therefore

Jadi GCD(4840, 1512) = 8.

Lemma 2.3.1 Any a|c and b|c imply ab|c if and only if GCD(a, b) = 1.

Lema 2.3.1 Sebarang a|c dan b|c berakibat ab|c jika dan hanya jika GCD(a, b) = 1.

Page 25: Contents · PDF filegai batu loncatan untuk membuktikan teorema-teorema lainnya. Sebuah akibat dari beberapa teorema atau lemma disebut dengan korolary. Sebuah pernyataan yang tidak

Chapter 2. Number Theory 19

Example. 3|30 and 5|30 imply 3× 5|30 as GCD(3, 5) = 1, however 2|30 and 6|30

imply 2× 6 - 30 as GCD(2, 6) 6= 1.

Contoh. 3|30 dan 5|30 maka 3×5|30 karena GCD(3, 5) = 1, akan tetapi 2|30 dan

6|30 maka 2× 6 - 30 karena GCD(2, 6) 6= 1.

2.4 Modular ArithmeticModulo Aritmatik

Let a, b, and m be integers, with m > 1. We say that a and b are congruent

modulo m, denoted by a ≡ b (mod m), if the remainder of a divided by m and b

divided by m are the same.

Diberikan bilangan bulat a, b dan m dimana m > 1. Bilangan a dikatakan kon-

gruen dengan b modulo m, dituliskan dengan a ≡ b (mod m), jika sisa pemba-

gian a oleh m dan b oleh m memberikan sisa yang sama.

Lemma 2.4.1 If a and b are congruent modulo m then m| (a− b).

Lema 2.4.1 Jika a dan b kongruen modulo m maka m| (a− b).

Proof. a ≡ b (mod m) =⇒ a = q1m+r and b = q2m+r. Hence a−b = (q1−q2)m,

it follows m| (a− b).

Bukti. a ≡ b (mod m) =⇒ a = q1m + r dan b = q2m + r. Kemudian a − b =

(q1 − q2)m sehingga m| (a− b). 2

Page 26: Contents · PDF filegai batu loncatan untuk membuktikan teorema-teorema lainnya. Sebuah akibat dari beberapa teorema atau lemma disebut dengan korolary. Sebuah pernyataan yang tidak

Chapter 2. Number Theory 20

Lemma 2.4.2 (1) a ≡ b (mod m), b ≡ a (mod m) are equivalent with a−b ≡ 0 (mod m);(2) if a ≡ b (mod m) and b ≡ c (mod m) then a ≡ c (mod m); (3) if a ≡ b (mod m)

and d|m then a ≡ b (mod d); (4) if a ≡ b (mod m) and c ≡ d (mod m) then ax + cy ≡bx + dy (mod m) and ac ≡ bd (mod m), for any x, y ∈ I .

Lema 2.4.2 (1) a ≡ b (mod m), b ≡ a (mod m) adalah setara dengan a−b ≡ 0 (mod m)

; (2) jika a ≡ b (mod m) dan b ≡ c (mod m) maka a ≡ c (mod m); (3) jika a ≡b (mod m) dan d|m maka a ≡ b (mod d); (4) jika a ≡ b (mod m) dan c ≡ d (mod m)

maka ax + cy ≡ bx + dy (mod m) dan ac ≡ bd (mod m), untuk sebarang x, y ∈ I .

Proof. a ≡ b (mod m) −→ m|(a − b) and c ≡ d (mod m) −→ m|(c − d). From

Lemma 2.1.1, we have m|((a−b)x+(c−d)y) or m|((ax+cy)−(bx+dy)), therefore

ax + cy ≡ bx + dy (mod m).

Bukti. a ≡ b (mod m) −→ m|(a − b) dan c ≡ d (mod m) −→ m|(c − d). Sesuai

Lema 2.1.1 didapat m|((a−b)x+(c−d)y) atau m|((ax+cy)−(bx+dy)), sehingga

ax + cy ≡ bx + dy (mod m). 2

It follows from Lemma 2.4.2, if f(x) is a polynomial of integer coefficients and

a ≡ b (mod m) then f(a) ≡ f(b) (mod m).

Akibat dari Lemma 2.4.2, jika f(x) adalah suatu fungsi polinom dengan koefisien-

koefisien bilangan bulat dan a ≡ b (mod m) maka f(a) ≡ f(b) (mod m).

Example. Prove that for any natural n, S = 2903n−803n+261n−464n is divisible

by 7 and 271. Furthermore, prove that 1897|S.

Contoh. Buktikan bahwa untuk setiap bilangan asli n maka S = 2903n−803n +

261n − 464n habis dibagi 7 dan 271. Buktikan juga bahwa 1897|S.

Solution. Since 2903 ≡ 803 (mod 7) and 464 ≡ 261 (mod 7), as well as 2903 ≡464 (mod 271) and 803 ≡ 261 (mod 271), from Lemma 2.4.2(4) we must have 7|S

Page 27: Contents · PDF filegai batu loncatan untuk membuktikan teorema-teorema lainnya. Sebuah akibat dari beberapa teorema atau lemma disebut dengan korolary. Sebuah pernyataan yang tidak

Chapter 2. Number Theory 21

and 271|S. Furthermore, since 1897 = 7 × 271 and GCD(7, 271) = 1, it follows

from Lemma 2.3.1 that 1897|S.

Solusi. Karena 2903 ≡ 803 (mod 7) dan 464 ≡ 261 (mod 7), demikian juga 2903 ≡464 (mod 271) dan 803 ≡ 261 (mod 271) maka sesuai dengan Lema 2.4.2(4) di-

pastikan bahwa 7|S dan 271|S. Selanjutnya karena 1897 = 7×271 dan GCD(7, 271) =

1, maka sesuai Lema 2.3.1 terbukti 1897|S.

Lemma 2.4.3 (am + b)n ≡ bn (mod m)

Lema 2.4.3 (am + b)n ≡ bn (mod m)

Proof. The proof is the same with showing that there exists an integer k such

that (am + b)n − bn = km.

Bukti. Pembuktian ini sama artinya dengan membuktikan ada bilangan bulat

k sehingga (am + b)n − bn = km.

(am + b)n − bn = (am)n + n(am)n−1 · b + · · ·+ n(am)bn−1 + bn − bn

= {a(am)n−1 + an(am)n−2 + · · ·+ an(b)n−1}m= km.

2

Example. Determine the ones of 19971991.

Contoh. Tentukan angka satuan bilangan 19971991.

Solution. From Lemma 2.4.3, the solution is as follows:

Page 28: Contents · PDF filegai batu loncatan untuk membuktikan teorema-teorema lainnya. Sebuah akibat dari beberapa teorema atau lemma disebut dengan korolary. Sebuah pernyataan yang tidak

Chapter 2. Number Theory 22

Solusi. Dengan menggunakan Lema 2.4.3 maka solusinya adalah sebagai berikut:

The ones (Angka satuan) 19971991 = The remainder (Sisa bagi) 19971991 oleh 10

= (199× 10 + 7)1991 (mod 10)

= 71991 (mod 10)

= 74×497+3 (mod 10)

= (74)497 × 73 (mod 10)

= (2401)497 × 343 (mod 10)

= (240× 10 + 1)497 × (34× 10 + 3) (mod 10)

= 1× 3 (mod 10)

= 3 (mod 10)

Thereforre, the ones of 19971991 is 3.

Sehingga, angka satuan bilangan 19971991 adalah 3.

It is easy to see that powering numbers 0, 1, 2, . . . , 9 give the following ones/units:

0 => 0 → 1 type, 1 => 1 → 1 type, 2 => 2, 4, 8, 6 → 4 types, 3 => 3, 9, 7, 1 → 4

types, 4 => 4, 6 → 2 types, 5 => 5 → 1 type, 6 => 6 → 1 type, 7 => 7, 9, 3, 1 →4 types, 8 => 8, 4, 2, 6 → 4 types, 9 => 9, 1 → 2 types.

Mudah dicermati bahwa perpangkatan bilangan 0, 1, 2, . . . , 9 akan menghasilkan

angka satuan berikut: 0 => 0 → 1 jenis, 1 => 1 → 1 jenis, 2 => 2, 4, 8, 6 → 4

jenis, 3 = 3, 9, 7, 1 → 4 jenis, 4 => 4, 6 → 2 jenis, 5 => 5 → 1 jenis, 6 => 6 → 1

jenis, 7 => 7, 9, 3, 1 → 4 jenis, 8 => 8, 4, 2, 6 → 4 jenis, 9 => 9, 1 → 2 jenis.

Therefore, finding the units of exponential number can be obtained by the fol-

lowing ways: Consider the example above. Determine the ones of 19971991. Since

the ones of the base is 7, the ones of the exponential number contains 4 types,

namely 7,9,3,1. Thus 1991/4 = 4× 497 + 3, it implies that 19971991 = 19974×497+3

which follows the ones of 73 → 3. Therefore the ones of 19971991 is 3.

Page 29: Contents · PDF filegai batu loncatan untuk membuktikan teorema-teorema lainnya. Sebuah akibat dari beberapa teorema atau lemma disebut dengan korolary. Sebuah pernyataan yang tidak

Chapter 2. Number Theory 23

Dengan demikian pencarian angka satuan dari perpangkatan bilangan dapat

dilakukan dengan cara berikut: Misal pada contoh soal di atas. Berapa angka

satuan bilangan 19971991. Karena angka satuan bilangan dasar adalah angka 7

maka angka satuan perpangkatan bilangan ini akan meliputi 4 jenis yaitu 7,9,3,1.

Selanjutnya 1991/4 = 4× 497+3 sehingga 19971991 = 19974×497+3 yang berakibat

angka satuan dari 73 → 3. Dengan demikian angka satuan bilangan 19971991

adalah 3.

Example. Determine the ones of 345678292784383951.

Contoh. Tentukan angka satuan bilangan 345678292784383951.

Solution. Since the ones of the base is 4, the ones of the exponential number

contains 2 types, namely 4,6. Thus 383951/2 = 2 × 191975 + 1, it implies that

345678292784383951 = 3456782927842×191975+1 which follows the ones of 41 → 4.

Therefore the ones of 345678292784383951 is 4.

Solusi. Karena angka satuan bilangan dasarnya adalah 4 maka angka satuan

perpangkatan bilangan ini akan meliputi 2 jenis yaitu 4,6. Selanjutnya tentukan

2|383951 = 2× 191975 + 1 sehingga 345678292784383951 = 3456782927842×191975+1

yang berakibat angka satuan dari 41 → 4. Dengan demikian angka satuan bi-

langan 345678292784383951 adalah 4.

2.5 Linear Diophantine EquationsPersamaan Linier Diophantin

An equation of the form ax + by = c where a, b, c are fixed integers and a, b are

all different from zero is called a linear Diophantine equation if the solutions x, y

respect to integers.

Page 30: Contents · PDF filegai batu loncatan untuk membuktikan teorema-teorema lainnya. Sebuah akibat dari beberapa teorema atau lemma disebut dengan korolary. Sebuah pernyataan yang tidak

Chapter 2. Number Theory 24

Persamaan ax + by = c dengan a, b, c bilangan-bilangan bulat dan a, b dua-

duanya bukan nol disebut persamaan linier Diophantine jika penyelesaiannya

dicari untuk bilangan-bilangan bulat.

Theorem 2.5.1 Diophantine equation ax+by = c is solvable if and only if GCD(a, b)|c.

Teorema 2.5.1 Persamaan Diophantine ax+ by = c mempunyai penyelesaian jika danhanya jika GCD(a, b)|c.

Proof. Let d = GCD(a, b) and d| c. We have d| c ⇐⇒ c = kd for any integers k.

Whilst d|GCD(a, b) ⇐⇒ am + bn = d for any m and n such that:

k(am + bn) = kd

a(km) + b(kn) = c,

it implies that x = mk dan y = nk

Bukti. Misal d = GCD(a, b) dan d| c, maka d| c ⇐⇒ c = kd untuk sebarang

bilangan bulat k. Sedangkan d|GCD(a, b) ⇐⇒ am + bn = d untuk sebarang

bilangan bulat m dan n sehingga:

k(am + bn) = kd

a(km) + b(kn) = c,

berarti x = mk dan y = nk

Theorem 2.5.2 If d = GCD(a, b) and x0, y0 are the solutions of Diophantine equationax + by = c, then the general solutions are

x = x0 +b

dk and y = y0 − a

dk; where k is an integer parameter.

Page 31: Contents · PDF filegai batu loncatan untuk membuktikan teorema-teorema lainnya. Sebuah akibat dari beberapa teorema atau lemma disebut dengan korolary. Sebuah pernyataan yang tidak

Chapter 2. Number Theory 25

Teorema 2.5.2 Jika d = GCD(a, b) dan x0, y0 merupakan penyelesaian persamaanDiophantine ax + by = c, maka penyelelesaian umum persamaan tersebut adalah :

x = x0 +b

dk dan y = y0 − a

dk; dengan k parameter bilangan bulat.

Example. Find the general solutions of Diophantine equation 738x+621y = 45.

Contoh. Tentukan solusi umum dari persamaan Diophantine 738x+621y = 45.

Solution. Finding GCD(738, 621) with Euclidean Algorithm

Solusi. Mencari GCD(738, 621) dengan Alogaritma Euclide

738 = 1× 621 + 117

621 = 5× 117 + 36

117 = 3× 36 + 9

36 = 4× 9 + 0.

we get GCD(738, 621) = 9. Since 9| 45, the equation is solvable. Consider the 9

as a linear combination of 738 and 621.

diperoleh GCD(738, 621) = 9. Karena 9| 45 maka persamaan di atas mempunyai

penyelesaian. Jadikan 9 sebagai kombinasi linear dari 738 dan 621.

9 = 117− 3 · 36

= 117− 3(621− 5× 117) = −3× 621 + 16× 117

= −3× 621 + 16(738− 621)

9 = 16× 738− 19× 621

multiplying the two sides by 5, we get 45 = 80 × 738 − 45 × 621. It implies that

x0 = 80, y0 = −95. Therefore the general solutions are:

Page 32: Contents · PDF filegai batu loncatan untuk membuktikan teorema-teorema lainnya. Sebuah akibat dari beberapa teorema atau lemma disebut dengan korolary. Sebuah pernyataan yang tidak

Chapter 2. Number Theory 26

Kalikan kedua ruas dengan 5, diperoleh 45 = 80 × 738 − 45 × 621, sehingga

didapat x0 = 80, y0 = −95. Dengan demikian penyelesaian umumnya adalah:

x = 80 +621

9k = 80 + 69k

y = −95− 738

9k = −95− 82k

PROBLEMS AND SOLUTIONS

SOAL-SOAL DAN PEMBAHASAN

1. A number A is the smallest natural number which is a product of the small-

est three of prime number. Determine two numbers between 200 and 300

which gives the same prime factor with A.

Bilangan A adalah bilangan asli terkecil yang merupakan hasil kali dari 3

bilangan prima terkecil. Sebutkan dua buah bilangan di antara 200 dan

300 yang mempunyai faktor prima yang serupa dengan bilangan A.

Solution. A = 2.3.5 = 30, the desired numbers are 24.3.5 = 240 and

2.32.5 = 270

Solusi. A = 2.3.5 = 30, jadi bilangan yang dicari adalah 24.3.5 = 240 dan

2.32.5 = 270

2. P (n) is a multiplication of the digits of number n, and S(n) is summation

of its digits. Determine the ones of n satisfying P (n) + S(n) = n if n is a

number consisting two digits.

P (n) didefinisikan sebagai perkalian antara angka-angka bilangan n dan

S(n) adalah penjumlahan antara angka-angka bilangan n. Tentukan angka

satuan n yang memenuhi P (n) + S(n) = n jika n adalah bilangan yang

terdiri dari dua angka.

Page 33: Contents · PDF filegai batu loncatan untuk membuktikan teorema-teorema lainnya. Sebuah akibat dari beberapa teorema atau lemma disebut dengan korolary. Sebuah pernyataan yang tidak

Chapter 2. Number Theory 27

Solution. Assume the number n is ab, we have a, b as a tens and ones.

Solution. Asumsikan bilangan tersebut adalah ab, maka kita mengartikan

a sebagai puluhan dan b sebagai satuan.

P (n) + S(n) = n

a.b + (a + b) = ab

a.b + (a + b) = 10a + b

ab = 9a = a9

Therefore, the ones of the number is 9

Dari sini didapatkan bahwa satuan bilangan n adalah 9

3. What is the remainder of 13 + 23 + 33 + ... + 1003 divided by 7?

Berapakah sisa pembagian 13 + 23 + 33 + ... + 1003 oleh 7?

Proof.

Bukti.

13 + 23 + 33 + ... + 1003 = (1 + 2 + 3 + ... + 100)3

= (5050)2

= (101x50)2 (mod 7)

= (101)2x(50)2 (mod 7)

= (2x72 + 3)2x(72 + 1)2 (mod 7)

= 32x1 (mod 7)

= 9 (mod 7)

= 2 (mod 7)

Page 34: Contents · PDF filegai batu loncatan untuk membuktikan teorema-teorema lainnya. Sebuah akibat dari beberapa teorema atau lemma disebut dengan korolary. Sebuah pernyataan yang tidak

¥¥¥¥¥¥ CHAPTER 3

Algebra FunctionsFungsi Aljabar

3.1 Polynomials InequalityPertidaksamaan Polinomial

In this section, we will show how so solve inequalities. Solving an inequality

means finding all of its solutions. A solution of an inequality is a number which

when substituted for the variable makes the inequality a true statement.

Dalam bagian ini, akan dipelajari bagaimana menyelesaikan pertidaksamaan.

Menyelesaikan pertidaksamaan berarti mencari semua himpunnan penyelesa-

ian. Penyelesaian dari pertidakasamaan adalah sebuah bilangan dimana pada

saat bilangan itu disubstitusikan maka pertidaksamaan menjadi pernyataan yang

benar.

Some properties related to inequality are as follows:

• If a > b then a = b + k, for any k.

• If a > b then a + c > b + c and a.c > b.c where c > 0.

• If a > b and b > c then a > c.

• If a > b and c > d then a + c > b + d.

• If a > b > 0 then 1/a > 1/b, and if a/b > 0 then a.b > 0.

28

Page 35: Contents · PDF filegai batu loncatan untuk membuktikan teorema-teorema lainnya. Sebuah akibat dari beberapa teorema atau lemma disebut dengan korolary. Sebuah pernyataan yang tidak

Chapter 3. Algebra Functions 29

Beberapa sifat-sifat terkait dengan pertidaksamaan adalah sebagai berikut:

• Jika a > b maka a = b + k, untuk sebarang k.

• Jika a > b maka a + c > b + c dan a.c > b.c dimana c > 0.

• Jika a > b dan b > c maka a > c.

• Jika a > b dan c > d maka a + c > b + d.

• Jika a > b > 0 maka 1/a > 1/b, dan jika a/b > 0 maka a.b > 0.

In general, we present inequality polynomials as:

Secara umum pertidaksamaan fungsi dinyatakan sebagai

a0xn + a1x

n−1 + · · ·+ an−1x + an ≥ · ∨ · ≤ 0

The following steps shows how to solve inequality:.

• factorizing the polynomials, if it is not factorizible then consider whether

the function is definite or not.

• Draw a number line and show the roots of polynomial equality on it.

• Put sign + or − on each interval of the number line respecting to the fol-

lowing rules:

– Consider the coefficient of the highest order of the inequality.

– If the coefficient of the the highest order is positive (respectively, or

negative) then the right side of line number interval is + (respectively,

or −). Hence, place an alternate sign on the remaining interval.

– Consider the sign of the inequality and confirm with the signs on the

interval to conclude the solution set.

Page 36: Contents · PDF filegai batu loncatan untuk membuktikan teorema-teorema lainnya. Sebuah akibat dari beberapa teorema atau lemma disebut dengan korolary. Sebuah pernyataan yang tidak

Chapter 3. Algebra Functions 30

Lagkah-langkah berikut menunjukkan bagaimana cara menyelesaikan persamaan

ini.

• Lakukan memfaktorkan terhadap fungsi polinomial, bila tidak dapat di-

faktorkan pertimbangkan apakah fungsi tersebut definit.

• Gambar sebuah garis bilangan dan letakkan semua akar-akar persamaan

polinomial dalam garis bilangan.

• Berikan tanda + atau − pada setiap interval dengan prinsip berikut:

– Lihat koefisien pangkat tinggi pertidaksamaan tersebut.

– Jika koefisien pangkat tertingginya positif maka interval paling kanan

adalah +, bila negatif maka tulis− dan kemudian interval selanjutnya

secara bergantian berlawanan tanda.

– Lihat tanda pertidaksamaannya dan cocokkan dengan tanda tanda

dalam interval untuk menentukan himpunan penyelesaiannya.

Example. Determine x satisfying inequality x2 − 2x− 80 ≤ 0 and

Contoh. Tentukan nilai-nilai x yang memenuhi kedua pertidaksamaan x2−2x−80 ≤ 0 dan

(2x− 5)501(x2 − 3x− 4)(−x2 + 7x− 80)(x3 + 4x2 − x + 1)2000

(x + 1)(x + 14)51≥ 0

Solution. To solve this inequality we should consider discriminant D = b2−4ac

of quadratic function ax2 + bx + c = 0. If D < 0 and a > 0 then the quadratic

function is positive definite, If D < 0 and a < 0 the quadratic function is negative

definite. Furthermore, all number of even power must be positive.

Jika

Page 37: Contents · PDF filegai batu loncatan untuk membuktikan teorema-teorema lainnya. Sebuah akibat dari beberapa teorema atau lemma disebut dengan korolary. Sebuah pernyataan yang tidak

Chapter 3. Algebra Functions 31

Solusi. Untuk menyelesaikan pertidaksamaan ini maka kita harus ingat kon-

sep Deskriminan D = b2 − 4ac untuk fungsi kuadratik ax2 + bx + c = 0. Bila

D < 0 dan a > 0 maka fungsi kuadratik adalah definit positif, Bila D < 0 dan

a < 0 maka fungsi kuadratik adalah definit negatif. Kemudian setiap bilangan

yang berpangkat genap pasti bernilai positif.

x2 − 2x− 80 ≤ 0

(x− 10)(x + 8) ≤ 0 ⇐⇒ Hp1 = {x| − 8 ≤ x ≤ 10}

Whilst (Sedangkan)

(2x− 5)501(x2 − 3x− 4)(−x2 + 7x− 80)(x3 + 4x2 − x + 1)2000

(x + 1)(x + 14)51≥ 0

(2x− 5)501(x− 4)ª⊕(x + 14)51

≥ 0

For (Untuk) x 6= −14, (2x− 5)501(x− 4)(x + 14)51 ª⊕ ≥ 0

For (Untuk) x 6= −14, (2x− 5)501(x− 4)(x + 14)51

︸ ︷︷ ︸ª

ª ≥ 0

It follows (Berarti) (2x− 5)501(x− 4)(x + 14)51 ≤ 0

Therefore (Sehingga) Hp2 = {x| − 14 < x ≤ 5

2· ∨ · x ≥ 4}

Since the problem shows a conjunction relation then both solution sets Hp1 and

Hp2 must be joined. It implies that Hp = {x| − 8 ≤ x ≤ 52· ∨ · 4 ≤ x ≤ 10}.

Karena masalah di atas menunjukkan relasi konjungsi, maka kedua himpunan

penyelesaian Hp1 dan Hp2 harus digabung. Sehingga diperoleh Hp = {x| − 8 ≤x ≤ 5

2· ∨ · 4 ≤ x ≤ 10}.

Example. The value of x satisfying inequality (x − 2)4(−x2 − x − 10)|x2 − x −1|(−x + 1)2 ≤ 1 is . . .

Contoh. Harga x yang memenuhi pertidaksamaan (x− 2)4(−x2 − x− 10)|x2 −x− 1|(−x + 1)2 ≤ 1 adalah . . .

Page 38: Contents · PDF filegai batu loncatan untuk membuktikan teorema-teorema lainnya. Sebuah akibat dari beberapa teorema atau lemma disebut dengan korolary. Sebuah pernyataan yang tidak

Chapter 3. Algebra Functions 32

(A) x ≤ −1 ∨ 0 ≤ x ≤ 1 ∨ x ≥ 2

(B) −12≤ x ≤ 0 ∨ 3

2≤ x ≤ 2

(C) −1 ≤ x ≤ 0 ∨ 1 ≤ x ≤ 2

(D) −12

< x ≤ 32

(E) All x (Semua harga x)

3.1.1 Inverse FunctionFungsi Invers

If f is a function from A to B then an inverse function for f is a function in the

opposite direction, from B to A, with the property that a round trip (a composi-

tion) from A to B to A (or from B to A to B) returns each element of the initial

set to itself. Thus, if an input x into the function produces an output y, then

inputting y into the inverse function f−1 (read f inverse, not to be confused with

exponentiation) produces the output x. Not every function has an inverse; those

that are called invertible.

Jika f adalah fungsi dari A ke B maka fungsi invers untuk f adalah suatu fungsi

balikan yang berlawanana arah, yaitu dari B ke A, dengan sifat rute balikan

itu (komposisi fungsi) adalah dari A ke B ke A (atau dari B ke A ke B) akan

mengembalikan masing-masing element dari himpunan asal ke elemen-element

itu sendiri. Dengan kata lain, jika dimasukkan x ke dalam fungsi memberikan

hasil y, maka dengan memasukkan nilai y ke dalam fungsi invers f−1 (baca f in-

vers, biar tidak dibingungkan dengan pengertian eksponensial) akan diperoleh

hasil x. Tidak setiap fungsi memiliki balikan, fungsi yang tidak mempunyai

fungsi invers ini disebut invertibel.

Page 39: Contents · PDF filegai batu loncatan untuk membuktikan teorema-teorema lainnya. Sebuah akibat dari beberapa teorema atau lemma disebut dengan korolary. Sebuah pernyataan yang tidak

Chapter 3. Algebra Functions 33

Theorem 3.1.1 If f(x) is function with x ∈ < then inverse function f−1(x) satisfiesthe following:

Teorema 3.1.1 Jika f(x) suatu fungsi dalam x ∈ <maka fungsi invers f−1(x) memenuhihal berikut:

1. f ◦ f−1(x) = x · ∧ · (f ◦ g)−1(x) = g−1 ◦ f−1(x)

2. f(x) = ax + b =⇒ f−1(x) = x−ba

3. f(x) = ax+bcx+d

=⇒ f−1(x) = −dx+bcx−a

4. f(x) = ax2 + bx + c =⇒ f−112 (x) = − b

2a±

√1a

(x + D

4a

)

5. f(x) =√

ax + b =⇒ f−1(x) = x2−ba

6. f(x) = ax + b · ∧ · f ◦ g = px + q =⇒ g(x) = px+q−ba

7. f(x) = abx =⇒ f−1(x) =alog x

b

Bukti. For no. 4. From (1), we know that f ◦ f−1(x) = x, so that for f(x) =

ax2 + bx + c we get

Proof. Untuk nomor 4. Dari (1) dipahami bahwa f ◦f−1(x) = x sehingga untuk

f(x) = ax2 + bx + c diperoleh

f(f−1) = x

a(f−1)2 + b(f−1) + c = x

a(f−1)2 + b(f−1) + c− x = 0

f−112 = − b

2a±

√1

a

(x +

b2 − 4ac

4a

)

2

Page 40: Contents · PDF filegai batu loncatan untuk membuktikan teorema-teorema lainnya. Sebuah akibat dari beberapa teorema atau lemma disebut dengan korolary. Sebuah pernyataan yang tidak

Chapter 3. Algebra Functions 34

3.1.2 Arithmetic and Geometric SequenceBarisan Aritmatik dan Geometrik

In mathematics, an arithmetic sequence or arithmetic progression is a sequence

of numbers such that the difference of any two successive members of the se-

quence is a constant. For instance, the sequence 3, 5, 7, 9, 11, 13, . . . is an arith-

metic progression with common difference 2. The sum of the terms of a arith-

metic progression is known as a arithmetic series. Thus, the general form of a

arithmetic sequence is a, a + b, a + 2b, . . . , a + (n − 1)b; and that of a geometric

series is (a) + (a + b) + (a + 2b) + · · ·+ (a + (n− 1)b).

Dalam matematika, suatu barisan aritmatika atau urutan aritmatika didefin-

isikan sebagai barisan bilangan sedemikian hingga beda dari dua bilangan yang

berurutan dari barisan itu adalah konstan. Sebagai contoh, barisan bilangan

3, 5, 7, 9, 11, 13, . . . adalah barisan aritmatika dengan beda 2. Jumlah suku-suku

dari barisan arimatika disebut dengan deret aritimatika. Bentuk umum dari

barisan aritmatika adalah a, a + b, a + 2b, . . . , a + (n − 1)b; sedangkan bentuk

umum dari deret aritmatika adalah (a) + (a + b) + (a + 2b) + · · ·+ (a + (n− 1)b).

Meanwhile, A geometric sequence is a sequence of numbers where each term

after the first is found by multiplying the previous one by a fixed non-zero num-

ber called the common ratio. For example, the sequence 2, 6, 18, 54,. . . is a

geometric progression with common ratio 3. Similarly 10, 5, 2.5, 1.25, . . . is a

geometric sequence with common ratio 1/2. The sum of the terms of a geo-

metric progression is known as a geometric series. Thus, the general form of

a geometric sequence is a, ar, ar2, ar3, ar4, . . .; and that of a geometric series is

a + ar + ar2 + ar3 + ar4 + . . .

Sementara, suatu barisan geometrik adalah barisan bilangan dimana masing-

masing suku setelah bilangan pertama didapat dari mengalikan bilangan se-

Page 41: Contents · PDF filegai batu loncatan untuk membuktikan teorema-teorema lainnya. Sebuah akibat dari beberapa teorema atau lemma disebut dengan korolary. Sebuah pernyataan yang tidak

Chapter 3. Algebra Functions 35

belumnya dengan bilangan konstan yang tidak nol, yang disebut dengan ra-

sio dan rasio ini selalu sama. Sebagai contoh, barisan bilangan 2, 6, 18, 54,. . .

adalah barisan geometrik dengan rasio 3. Sama hanya dengan barisan 10, 5,

2.5, 1.25, . . . adalah barisan geometrik dengan rasio 1/2. Jumlah suku-suku

dari barisan geometrik ini disebut dengan deret geometrik. Bentuk umum dari

barisan geometri adalah a, ar, ar2, ar3, ar4, . . .; sedangkan bentuk umum dari

deret geometri adalah a + ar + ar2 + ar3 + ar4 + . . .

Lemma 3.1.1 Given an arithmetic sequence a+(a+b)+(a+2b)+ · · ·+(a+(n−1)b),where a is an initial term an b is a deference of any two successive terms. Then, we havethe followings:

Lema 3.1.1 Diberikan suatu deret aritmatika a+(a+b)+(a+2b)+· · ·+(a+(n−1)b),dimana a adalah nilai suku awal dan b adalah beda suku-suku yang berurutan, makabeberapa hal berikut berlaku:

Un = a + (n− 1)b (3.1)

Sn =n

2[U1 + Un] (3.2)

Sn =n

2[2a + (n− 1)b] (3.3)

Ut = a +1

2(n− 1)b (3.4)

Lemma 3.1.2 If we insert some k numbers on between any two successive numbers ofarithmetic sequence and they form a new arithmetic sequence, then

Page 42: Contents · PDF filegai batu loncatan untuk membuktikan teorema-teorema lainnya. Sebuah akibat dari beberapa teorema atau lemma disebut dengan korolary. Sebuah pernyataan yang tidak

Chapter 3. Algebra Functions 36

Lema 3.1.2 Jika diantara dua buah bilangan yang berurutan dari suatu deret arit-matika disisipi k buah bilangan dan membentuk deret aritmatika baru maka

b′ =b

k + 1(3.5)

n′ = n + (n− 1)k (3.6)

Lemma 3.1.3 Let a + ar + ar2 + · · · + arn−1 be a geometric sequence, where a, r arerespectively an initial value and ratio. The followings hold for this squence

Lema 3.1.3 Diberikan suatu deret geometrik a + ar + ar2 + · · · + arn−1, dimana a

adalah nilai suku awal dan r adalah rasio suku-suku yang berurutan. Beberapa halberikut berlaku untuk deret ini.

Un = arn−1 (3.7)

Sn =a(rn − 1)

r − 1, r > 1 (3.8)

Sn =a(1− rn)

1− r, r < 1 (3.9)

Ut = a√

rn−1 (3.10)

Lemma 3.1.4 If we insert some k numbers on between any two successive numbers ofgeometric sequence and they form a new geometric sequence, then

Lema 3.1.4 Jika diantara dua buah bilangan yang berurutan dari suatu deret geometrikdisisipi k buah bilangan dan membentuk deret geometrik baru maka

r′ = k+1√

r (3.11)

n′ = n + (n− 1)k (3.12)

Page 43: Contents · PDF filegai batu loncatan untuk membuktikan teorema-teorema lainnya. Sebuah akibat dari beberapa teorema atau lemma disebut dengan korolary. Sebuah pernyataan yang tidak

Chapter 3. Algebra Functions 37

Lemma 3.1.5 For arithmetic and geometric series satisfy Un = Sn − S(n− 1).

Lema 3.1.5 Untuk deret aritmatik maupun geometrik berlaku Un = Sn − S(n− 1).

Lemma 3.1.6 An infinite convergence geometric series, where |r| < 1, satisfies S∞ =a

1−r.

Lema 3.1.6 Untuk deret geometrik tak hingga konvergen, dimana |r| < 1, berlakuS∞ = a

1−r.

Example. Determine the sum of all radius of circles whose numbers are infinity.

Example. Tentukan jumlah seluruh jari-jari lingkaran berikut ini sampai pada

banyaknya lingkaran tak higga.

Figure 3.1: Squares in the circles.

Page 44: Contents · PDF filegai batu loncatan untuk membuktikan teorema-teorema lainnya. Sebuah akibat dari beberapa teorema atau lemma disebut dengan korolary. Sebuah pernyataan yang tidak

Chapter 3. Algebra Functions 38

Furthermore, how to find the n-term of sequence 1, 4, 11, 22, 37, . . . . Both formu-

las Un can not be used to answer this problems as the sequence does not have

a common difference at first layer, namely 3,7,11,15. Thus then, this sequence

admits an arithmetic sequence of common difference 4. The solution can be ob-

tained by considering polynomial f(x) = akxk +ak−1x

k−1 +ak−2xk−2 + · · ·+a0x

0.

Selanjutnya, bagaimana menentukan suku ke−n dari barisan bilangan: 1, 4, 11,

22, 37, . . . . Kedua rumus Un di atas tidak dapat dipakai untuk menjawab per-

tanyaan ini, sebab beda pada layer pertama tidak sama, yaitu 3,7,11,15, baru

kemudian barisan beda ini merupakan barisan aritmatika dengan beda 4. So-

lusinya diperoleh dari mempertimbangkan fungsi polinomial f(x) = akxk +

ak−1xk−1 + ak−2x

k−2 + · · ·+ a0x0.

Lemma 3.1.7 Let f(x) = a1x + a0; x = 1, 2, . . . , n be a linear function series of thepolynomial, it has common difference b = a1 at firs layer.

Lema 3.1.7 Misal deret fungsi linear dari polinomial di atas adalah f(x) = a1x +

a0; x = 1, 2, . . . , n, maka deret ini mempunyai beda b = a1 pada layer pertama.

Proof. An expansion of the linear function for the n-terms is as follows:

Bukti. Ekspansi fungsi linear sampai suku ke−n adalah sebagai berikut:

a1 + a0, 2a1 + a0, . . . , a1(n− 3) + a0, a1(n− 2) + a0, a1(n− 1) + a0, a1n + a0

hence the common difference of the successive terms is b = f(n) − f(n − 1) =

f(n− 1)− f(n− 2) = · · · = f(2)− f(1) = a1, namely

sehingga beda dari suku-suku yang berurutan adalah b = f(n) − f(n − 1) =

f(n− 1)− f(n− 2) = · · · = f(2)− f(1) = a1, yaitu

a1 + a0, 2a1 + a0︸ ︷︷ ︸a1

, . . . , a1(n− 3) + a0, a1(n− 2) + a0︸ ︷︷ ︸a1

, a1(n− 1) + a0, a1n + a0︸ ︷︷ ︸a1

Page 45: Contents · PDF filegai batu loncatan untuk membuktikan teorema-teorema lainnya. Sebuah akibat dari beberapa teorema atau lemma disebut dengan korolary. Sebuah pernyataan yang tidak

Chapter 3. Algebra Functions 39

Lema 3.1.8 Let f(x) = a2x2 + a1x + a0; x = 1, 2, . . . , n be a quadratic function series

of the polynomial, it has common difference b = 2a2 at second layer.

Lema 3.1.9 Misal deret fungsi kuadrat dari polinomial di atas adalah f(x) = a2x2 +

a1x + a0; x = 1, 2, . . . , n, maka deret ini mempunyai beda b = 2a2 pada layer kedua.

Proof. An expansion of the quadratic function for the n-terms is as follows:

Bukti. Ekspansi fungsi quadratic untuk suku ke−n adalah sebagai berikut:

a2 + a1 + a0, 4a2 + 2a1 + a0, . . . , a2(n− 1)2 + a1(n− 1) + a0, a2n2 + a1n + a0

hence the differences of the successive terms are as follows:

sehingga beda layer pertama dari suku-suku yang berurutan adalah sebagai

berikut:

bn = f(n)− f(n− 1) = 2a2n− a2 + a1 = (2n− 1)a2 + a1

bn−1 = f(n− 1)− f(n− 2) = 2a2n− 3a2 + a1 = (2n− 3)a2 + a1

bn−2 = f(n− 2)− f(n− 3) = 2a2n− 5a2 + a1 = (2n− 5)a2 + a1

...

b3 = f(3)− f(2) = 5a2 + a1

b2 = f(2)− f(1) = 3a2 + a1.

Therefore, the common difference of the successive terms of the difference se-

quence of the first layer is

Dengan demikian, beda dari suku-suku yang berurutan pada barisan beda un-

tuk layer pertama adalah

b = bn − bn−1 = bn − bn−1 = · · · = b3 − b2 = 2a2

Page 46: Contents · PDF filegai batu loncatan untuk membuktikan teorema-teorema lainnya. Sebuah akibat dari beberapa teorema atau lemma disebut dengan korolary. Sebuah pernyataan yang tidak

Chapter 3. Algebra Functions 40

or

atau

a2 + a1 + a0, 4a2+︸ ︷︷ ︸3a2+a1

2a1 + a0, 9a2+︸ ︷︷ ︸5a2+a1

3a1 + a0, 16a2+︸ ︷︷ ︸7a2+a1

4a1 + a0, 25a2+︸ ︷︷ ︸9a2+a1→Layer I

5a1 + a0

︸ ︷︷ ︸2a2

︸ ︷︷ ︸2a2

︸ ︷︷ ︸2a2→Layer II

2

Lema 3.1.10 Let f(x) = a3x3 + a2x

2 + a1x + a0; x = 1, 2, . . . , n, be a cubic functionseries of the polynomial, it has common difference b = 6a3 at third layer.

Lema 3.1.11 Misal deret fungsi kubik dari polinomial di atas adalah f(x) = a3x3 +

a2x2 + a1x + a0; x = 1, 2, . . . , n, maka deret ini mempunyai beda b = 6a3 pada layer

ketiga.

Proof. As exercise.

Bukti. Sebagai latihan. 2

We conclude inductively in the following theorem:

Secara induktif dapat disimpulkan dalam teorema berikut:

Theorem 3.1.2 The series of polynomial function f(x) = akxk+ak−1x

k−1+ak−2xk−2+

· · ·+ a0x0; x = 1, 2, . . . , n has a common difference b = k!ak at layer k.

Teorema 3.1.2 Deret fungsi polinomial f(x) = akxk + ak−1x

k−1 + ak−2xk−2 + · · · +

a0x0; x = 1, 2, . . . , n, mempunyai beda yang sama b = k!ak pada layer ke−k.

Page 47: Contents · PDF filegai batu loncatan untuk membuktikan teorema-teorema lainnya. Sebuah akibat dari beberapa teorema atau lemma disebut dengan korolary. Sebuah pernyataan yang tidak

Chapter 3. Algebra Functions 41

Proof. As exercise.

Bukti. Sebagai latihan. 2

Example. Given a series 15 + 32 + 63 + 108 + 167 + . . . . Obtain the n−term and

the n−sum.

Example. Diberikan suatu deret 15 + 32 + 63 + 108 + 167 + . . . . Tentukan suku

ke−n dan jumlah ke−n.

Solution. The series above forms the following

Solusi. Deret di atas akan mengikuti pola berikut

15 + 32︸ ︷︷ ︸17

+ 63︸ ︷︷ ︸31

+ 108︸ ︷︷ ︸45

+ 167︸ ︷︷ ︸59

+ . . .

︸ ︷︷ ︸14

︸ ︷︷ ︸14

︸ ︷︷ ︸14

Since the series has a common difference at second layer, we have k = 2 and

the function f(x) = a2x2 + a1x + a0. We need to find the coefficients a2, a1, a0.

Theorem 3.1.2 implies b = k!ak = 2!a2 = 14 → a2 = 7, it follows f(x) = 7x2 +

a1x + a0. Let Un = f(n) and Un = 7n2 + a1n + a0. We have the following:

Karena deret fungsi mempunyai beda yang sama pada layer kedua maka k = 2

dan rumus f(x) = a2x2 + a1x + a0. Selanjutnya perlu dicari koefisien a2, a1, a0.

Dari Theorema 3.1.2 diperoleh b = k!ak = 2!a2 = 14 → a2 = 7. Sehingga

f(x) = 7x2 + a1x + a0. Misal Un = f(n) dan Un = 7n2 + a1n + a0, maka akan

didapat:

n = 1 → 7 + a1 + a0 = 15 → a1 + a0 = 8

n = 2 → 28 + 2a1 + a0 = 32 → 2a1 + a0 = 4

From both equations, we have a1 = −4 dan a0 = 12 which implies that the

n−term is Un = 7n2 − 4n + 12.

Page 48: Contents · PDF filegai batu loncatan untuk membuktikan teorema-teorema lainnya. Sebuah akibat dari beberapa teorema atau lemma disebut dengan korolary. Sebuah pernyataan yang tidak

Chapter 3. Algebra Functions 42

Dari kedua persamaan diperoleh nilai a1 = −4 dan a0 = 12. Sehingga suku

ke−n adalah Un = 7n2 − 4n + 12.

To find the n−sum of the above series, we consider the following series

Untuk menentukan jumlah ke−n dari deret di atas, maka kita lihat deret berikut

Un → 15, 32, 63, 108, 167, . . .

Sn → 15, 47, 110, 218, 385, . . .

15, 47︸ ︷︷ ︸32

, 110︸ ︷︷ ︸63

, 218︸ ︷︷ ︸108

, 385︸ ︷︷ ︸167

+ . . .

︸ ︷︷ ︸31

︸ ︷︷ ︸45

︸ ︷︷ ︸59

︸ ︷︷ ︸14

︸ ︷︷ ︸14

Since the sequence of function has a common difference on the third layer, we

have k = 3 and function f(x) = a3x3 + a2x

2 + a1x + a0. We need to find the

coefficients a3, a2, a1, a0. Theorem 3.1.2 implies b = k!ak = 3!a3 = 14 → a3 = 73.

Hence f(x) = 73x3 +a2x

2 +a1x+a0. Let Sn = f(n) and Sn = 73n3 +a2n

2 +a1n+a0.

We have the following:

Karena deret fungsi mempunyai beda yang sama pada layer ketiga maka k = 3

dan rumus f(x) = a3x3 + a2x

2 + a1x + a0. Selanjutnya perlu dicari koefisien

a3, a2, a1, a0. Dari Theorema 3.1.2 diperoleh b = k!ak = 3!a3 = 14 → a3 = 73.

Sehingga f(x) = 73x3 + a2x

2 + a1x + a0. Misal Sn = f(n) maka Sn = 73n3 + a2n

2 +

a1n + a0, maka akan didapat:

n = 1 → 7

3+ a2 + a1 + a0 = 15 → a2 + a1 + a0 =

38

3

n = 2 → 56

3+ 4a2 + 2a1 + a0 = 47 → 4a2 + 2a1 + a0 =

85

3

n = 3 → 189

3+ 9a2 + 3a1 + a0 = 110 → 9a2 + 3a1 + a0 =

141

3

Solving the three equations, we have a2 = 32, a1 = 67

6and a0 = 0 which implies

the n−sum is Sn = 73n3 + 3

2n2 + 67

6n.

Page 49: Contents · PDF filegai batu loncatan untuk membuktikan teorema-teorema lainnya. Sebuah akibat dari beberapa teorema atau lemma disebut dengan korolary. Sebuah pernyataan yang tidak

Chapter 3. Algebra Functions 43

Dengan menyelesaikan ketiga persamaan di atas diperoleh nilai a2 = 32, a1 = 67

6

dan a0 = 0. Sehingga jumlah suku ke−n adalah Sn = 73n3 + 3

2n2 + 67

6n. 2

3.2 Arithmetic, Geometric, Harmonic, and QuadraticMeansRataan Aritmatik, Geometrik, Harmonik danKuadratik

In mathematics or statistics, an arithmetic mean of a list of numbers is the sum

of all of the list divided by the number of items in the list. The arithmetic mean

is the most commonly-used type of average and is often referred to simply as

the average. A geometric mean is a type of mean or average, which indicates

a central tendency of numbers. It is similar to arithmetic mean, which is what

most people think of with the word ”average,” except that instead of adding the

set of numbers and then dividing the sum by the count of numbers in the set

n, the numbers are multiplied and then the nth root of the resulting product is

taken. A harmonic mean (formerly sometimes called the subcontrary mean) is

one of several kinds of average. Typically, it is appropriate for situations when

the average of rates is desired. A quadratic mean is a type of average which is

calculated as the square root of the mean of the squares.

Dalam matematika atau statistika, suatu rataan aritmatik dari daftar bilangan

adalah jumlah bilangan itu dibagi dengan banyaknya bilangan dalam daftar.

Rataan aritmatika adalah sesuatu yang paling sering dipakai dan secara umum

juga disebut dengan rata-rata. Rataan geometrik adalah jenis rataan atau rata-

rata yang merepresentasikan suatu tendensi sentral dari sekumpulan bilangan.

Hampir sama dengan rataan aritmatik dimana banyak orang berpikir bahwa ini

Page 50: Contents · PDF filegai batu loncatan untuk membuktikan teorema-teorema lainnya. Sebuah akibat dari beberapa teorema atau lemma disebut dengan korolary. Sebuah pernyataan yang tidak

Chapter 3. Algebra Functions 44

hanyalah rata-rata, namun dalam rata-rata seluruh bilangan dalam daftar di-

jumlahkan kemudian dibagi dengan banyaknya bilangan dalam daftar, untuk

rataan geometrik semua bilangan dalam daftar dikalikan kemudian ditarik akar

pangkat n. Rataan harmonik (dulu disebut sebagai kebalikan rataan) adalah

jenis lain dari rata-rata. Biasanya ini sesuai untuk menentukan rata-rata yang

mempertimbangkan tingkatan. Rataan kuadratik adalah tipe dari rata-rata yang

diperoleh dari menarik akar kuadrat dari rata-rata kuadratnya.

Let AM=Arithmetic Mean, GM=Geometric Mean, HM=Harmonic Mean and

QM=Quadratic Mean. For any positive real numbers a, b, it satisfies

Misal AM=Rataan Aritmatik, GM=Rataan Geometrik, HM=Rataan Harmonik

and QM=Rataan Kuadratik. Untuk sebarang a, b bilangan real positif, berlaku

(a− b)2 ≥ 0 ⇐⇒ a2 + b2 ≥ 2ab (3.13)

If a → √a and b →

√b then, from (3.13), it follows a + b ≥ 2

√ab or

Jika a → √a dan b →

√b maka (3.13) menjadi a + b ≥ 2

√ab atau

a + b

2≥√

ab (3.14)

In general we can present (3.14) as follows

Secara umum kita dapat menulis (3.14) sebagai

x1 + x2 + · · ·+ xn

n︸ ︷︷ ︸AM

≥ n√

x1x2 . . . xn︸ ︷︷ ︸GM

(3.15)

If a → 1a

and b → 1b

then (3.14) shows

Jika a → 1a

dan b → 1b

maka (3.14) menjadi

√ab ≥ 2

1a

+ 1b

(3.16)

Page 51: Contents · PDF filegai batu loncatan untuk membuktikan teorema-teorema lainnya. Sebuah akibat dari beberapa teorema atau lemma disebut dengan korolary. Sebuah pernyataan yang tidak

Chapter 3. Algebra Functions 45

In general, we can present (3.16) as

Secara umum kita dapat menulis (3.16) sebagai

n√

x1x2 . . . xn ≥ n1x1

+ 1x2

+ · · ·+ 1xn︸ ︷︷ ︸

HM

(3.17)

From (3.13), we get 2(a2 + b2) ≥ a2 +2ab+ b2 ⇐⇒ 2(a2 + b2) ≥ (a+ b)2. If the two

sides are divided by 4 then we have a2+b2

2≥ (

a+b2

)2, or

Dari (3.13) didapat 2(a2 + b2) ≥ a2 +2ab+ b2 ⇐⇒ 2(a2 + b2) ≥ (a+ b)2. Bila kedua

ruas dibagi 4, diperoleh a2+b2

2≥ (

a+b2

)2, atau√

a2 + b2

2≥ a + b

2(3.18)

In general (3.18) can be written as

Secara umum (3.18) ditulis sebagai√

x21 + x2

2 + · · ·+ x2n

n︸ ︷︷ ︸QM

≥ x1 + x2 + · · ·+ xn

n(3.19)

Therefore, from(3.15), (3.17) and (3.19) we can conclude that for any positive real

numbers x1, x2, . . . , xn satisfy:

Dengan demikian, berdasarkan (3.15), (3.17) dan (3.19) dapat disimpulkan bahwa

untuk setiap bilangan real positif x1, x2, . . . , xn berlaku:

QM ≥ AM ≥ GM ≥ HM

where

Page 52: Contents · PDF filegai batu loncatan untuk membuktikan teorema-teorema lainnya. Sebuah akibat dari beberapa teorema atau lemma disebut dengan korolary. Sebuah pernyataan yang tidak

Chapter 3. Algebra Functions 46

dimana

QM =

√x2

1 + x22 + · · ·+ x2

n

n

AM =x1 + x2 + · · ·+ xn

nGM = n

√x1x2 . . . xn

HM =n

1x1

+ 1x2

+ · · ·+ 1xn

Example. Let x, y, z be any real positive numbers such that x+ y + z = 1. Prove

that xy(x + y)2 + yz(y + z)2 + xz(x + z)2 ≥ 4xyz.

Contoh. Diketahui x, y, z adalah bilangan real positif sehingga x + y + z = 1.

Buktikan bahwa xy(x + y)2 + yz(y + z)2 + xz(x + z)2 ≥ 4xyz.

Solution. Since x + y + z = 1, we have

Solusi. Karena x + y + z = 1 maka didapat

xy(1− z)2 + yz(1− x)2 + xz(1− y)2 ≥ 4xyz

xy + yz + xz − 6xyz + xyz2 + x2yz + xy2z ≥ 4xyz

xy + yz + xz + xyz2 + x2yz + xy2z ≥ 10xyz1

x+

1

y+

1

z+ x + y + z ≥ 10

1

x+

1

y+

1

z≥ 9

1

3≥ 3

1x

+ 1y

+ 1z

∴ x + y + z

3︸ ︷︷ ︸AM

≥ 31x

+ 1y

+ 1z︸ ︷︷ ︸

HM

Page 53: Contents · PDF filegai batu loncatan untuk membuktikan teorema-teorema lainnya. Sebuah akibat dari beberapa teorema atau lemma disebut dengan korolary. Sebuah pernyataan yang tidak

Chapter 3. Algebra Functions 47

3.3 The Polynomials and Remainder TheoremSuku Banyak dan Teorema Sisa

A polynomial f(x) of degree n can be presented as:

Suatu suku banyak f(x) yang berderajad n dinyatakan dengan:

f(x) = a0xn + a1x

n−1 + · · ·+ an−1x + an

where a0, a1, . . . , an are constant, a0 6= 0 and n is a cardinal number.

dimana a0, a1, . . . , an adalah konstanta, a0 6= 0 dan n adalah bilangan cacah.

3.3.1 Polynomials DivisionPembagian Suku Banyak

The division of polynomial is similar to the division of numbers. For instance in

the number: Since 3 × 4 = 12, it follows 12 : 4 = 3 or 12 : 3 = 4. In the case of

12 : 4 = 3, numbers 4, 3 are respectively called divisor and quotient.

Pembagian suku banyak menyerupai pembagian bilangan. Sebagai contoh pada

bilangan: Karena 3×4 = 12 maka 12 : 4 = 3 atau 12 : 3 = 4. Pada kasus 12 : 4 = 3

maka bilangan 4, 3 masing-masing disebut pembagi dan hasil bagi.

A polynomial f(x) divided by a divisor P (x) will give a quotient H(x) and a

remainder S(x). Mathematically, we can write as:

f(x) = P (x)H(x) + S(x)

where: f(x) = is a polynomial of degree n; P (x) = is a polynomial of degree k;

H(x) = is a polynomial of degree n− k; S(x) = is a polynomial of degree k − 1.

Page 54: Contents · PDF filegai batu loncatan untuk membuktikan teorema-teorema lainnya. Sebuah akibat dari beberapa teorema atau lemma disebut dengan korolary. Sebuah pernyataan yang tidak

Chapter 3. Algebra Functions 48

Suatu suku banyak f(x) yang dibagi dengan pembagi P (x) akan menghasilkan

hasil bagi H(x) dan sisanya S(x). Secara matematis dapat ditulis sebagai:

f(x) = P (x)H(x) + S(x)

dimana: f(x) = suku banyak berderajad n; P (x) = suku banyak berderajad k;

H(x) = suku banyak berderajad n− k; S(x) = suku banyak berderajad k − 1.

When dividing polynomials, we need to consider the followings:

1. If the divisor is a linear term then the quotient and remainder can be ob-

tained by Horner technique.

2. If the divisor is not linear and not be able to be factorized into product of

linear terms then the quotient and remainder can be obtained by identity

technique.

Dalam melakukan pembagian terhadap suku banyak perlu diperhatikan hal-hal

berikut:

1. Jika pembaginya linier, maka hasil bagi dan sisanya dapat dicari dengan

menggunakan cara Horner.

2. Jika pembaginya bukan linier dan tidak dapat diuraikan menjadi bentuk

perpangkatan linier maka hasil bagi dan sisanya dapat dicari dengan per-

gunakan metoda Identitas.

Page 55: Contents · PDF filegai batu loncatan untuk membuktikan teorema-teorema lainnya. Sebuah akibat dari beberapa teorema atau lemma disebut dengan korolary. Sebuah pernyataan yang tidak

Chapter 3. Algebra Functions 49

3.3.2 Remainder TheoremTeorema Sisa

1. If polynomial f(x) is divided by (x± a) then the remainder is f(∓a).

2. If polynomial f(x) is divided by (ax± b) then the remainder is f(∓ ba).

3. If (x− a)|f(x) then f(a) = 0.

1. Jika suatu suku banyak f(x) dibagi dengan (x± a) maka sisanya f(∓a).

2. Jika suatu suku banyak f(x) dibagi dengan (ax± b) maka sisanya f(∓ ba).

3. Jika (x− a)|f(x) maka f(a) = 0.

3.3.3 Factor TheoremTeorema faktor

1. If (x− a) is a factor of f(x) then the root of f(x) = 0 is x = a.

2. If polynomial f(x) satisfies f(a) = 0, f(b) = 0 and f(c) = 0 then f(x) is

divisible by (x− a)(x− b)(x− c).

3. If f(x) is divided by (x− a)(x− b) then the remainder is S(x) = (x−a)(b−a)

f(b)+(x−b)(a−b)

f(a).

4. If f(x) is divided by (x − a)(x − b)(x − c) then the remainder is S(x) =(x−a)(x−b)(c−a)(c−b)

f(c) + (x−a)(x−c)(b−a)(b−c)

f(b) + (x−b)(x−c)(a−b)(a−c)

f(a).

Page 56: Contents · PDF filegai batu loncatan untuk membuktikan teorema-teorema lainnya. Sebuah akibat dari beberapa teorema atau lemma disebut dengan korolary. Sebuah pernyataan yang tidak

Chapter 3. Algebra Functions 50

1. Jika (x− a) adalah faktor dari f(x) maka akar dari f(x) = 0 adalah x = a.

2. Jika pada suku banyak f(x) berlaku f(a) = 0, f(b) = 0 dan f(c) = 0 maka

f(x) habis dibagi (x− a)(x− b)(x− c).

3. Jika f(x) dibagi dengan (x − a)(x − b) maka sisanya S(x) = (x−a)(b−a)

f(b) +(x−b)(a−b)

f(a).

4. Jika f(x) dibagi dengan (x−a)(x−b)(x−c) maka sisanya S(x) = (x−a)(x−b)(c−a)(c−b)

f(c)+(x−a)(x−c)(b−a)(b−c)

f(b) + (x−b)(x−c)(a−b)(a−c)

f(a).

3.3.4 Properties of Polynomial RootsSifat-Sifat Akar-Akar Suku Banyak

In this section, we consider Vieta’s Formula. Let si be the sum of the products of

distinct polynomial roots rj of the polynomial equation of degree n

Dalam hal ini akan disajikan penggunaan rumus Vieta. Misal si adalah jum-

lah dari hasil kali akar-akar polinomial yang berbeda rj dari sebuah polinomial

berderajad n

anxn + an−1xn−1 + ... + a1x + a0 = 0

where the roots are taken i at a time (i.e., si is defined as the symmetric polyno-

mial Πi(r1, ..., rn) for i = 1, ..., n). For example, the first few values of si are

dimana akar-akar itu dihitung sebanyak i dalam suatu proses (atau si didefin-

isikan sebagai polinomial simetrik Πi(r1, ..., rn) untuk i = 1, ..., n). Sebagai con-

toh, beberapa nilai si yang pertama adalah

s1 = r1 + r2 + r3 + r4 + . . .

s2 = r1r2 + r1r3 + r1r4 + r2r3 + . . .

s3 = r1r2r3 + r1r2r4 + r2r3r4 + . . ....

Page 57: Contents · PDF filegai batu loncatan untuk membuktikan teorema-teorema lainnya. Sebuah akibat dari beberapa teorema atau lemma disebut dengan korolary. Sebuah pernyataan yang tidak

Chapter 3. Algebra Functions 51

and so on. Then Vieta’s formulas states that

dan seterusnya. Maka rumus Vieta’s dinyatakan sebagai

si = (−1)i an−i

an

The followings are some example of Vieta’s formula.

Berikut ini adalah beberapa contoh dari rumusan Vieta.

1. For the polynomial ax3 + bx2 + cx + d = 0, we have:

Pada suku banyak ax3 + bx2 + cx + d = 0 berlaku:

1) x1 + x2 + x3 = −b/a

2) x1x2 + x1x3 + x2x3 = c/a

3) x1 · x2 · x3 = −d/a

2. For the polynomial ax4 + bx3 + cx2 + dx + e = 0, we have:

Pada suku banyak ax4 + bx3 + cx2 + dx + e = 0 berlaku:

1) x1 + x2 + x3 + x4 = −b/a

2) x1x2 + x1x3 + x1x4 + x2x3 + x2x4 + x3x4 = c/a

2) x1x2x3 + x1x2x4 + x1x3x4 + x2x3x4 = −d/a

3) x1 · x2 · x3 · x4 = e/a

We note that if the degree of polynomial is even then the the value of s is positive

otherwise it is negative.

Dapat dicatat bahwa bila pangkat tertinggi dari polinomial adalah genap maka

nilai s adalah positif jika ganjil maka nilai s adalah negatif.

To find some rational roots of polynomials can be used the following steps:

Untuk menentukan beberapa akar rasional dari suku banyak dapat digunakan

langkah-langkah berikut:

Page 58: Contents · PDF filegai batu loncatan untuk membuktikan teorema-teorema lainnya. Sebuah akibat dari beberapa teorema atau lemma disebut dengan korolary. Sebuah pernyataan yang tidak

Chapter 3. Algebra Functions 52

1. If the sum of all polynomial coefficients is equal to 0 then x = 1 is one of

the root.

Jika jumlah seluruh koefisien suku banyak sama dengan 0, maka x = 1

merupakan salah satu akarnya.

2. If the sum of the coefficients of odd order and even order are the same then

x = −1 is one of the root.

Jika jumlah koefisien pangkat ganjil dan genap adalah sama, maka x = −1

merupakan salah satu akarnya.

3. If (1) and (2) are not applicable then consider a trial and error technique by

finding factors of the coefficient of the lowest order and substituting into

f(x). Observe whether f(x) = 0 or not.

Jika langkah (1) dan (2) tidak memenuhi, maka gunakan cara coba-coba

yaitu dengan menentukan faktor dari koefisien pangkat terendahnya dan

masukkan ke dalam f(x). Amati apakah f(x) = 0 atau tidak.

PROBLEMS AND SOLUTIONS

SOAL-SOAL DAN PEMBAHASAN

1. A geometric series is presented as :

1

32+

1

4+ 2 + 16 + 128 + 1024

When we insert two numbers in between any two successive numbers

such that it forms a new geometric series, determine the new ratio and

the number of terms of the new geometric series.

Suatu deret geometri diketahui sebagai berikut:

1

32+

1

4+ 2 + 16 + 128 + 1024

Page 59: Contents · PDF filegai batu loncatan untuk membuktikan teorema-teorema lainnya. Sebuah akibat dari beberapa teorema atau lemma disebut dengan korolary. Sebuah pernyataan yang tidak

Chapter 3. Algebra Functions 53

Jika disisipkan dua buah bilangan kedalam dua suku yang berurutan pada

deret geometri ini sedemikian hingga deret itu membentuk deret geometri

baru, tentukan ratio dan banyaknya suku deret geometri baru tersebut.

Solution. We have the following:

Solusi. Diketahui berikut:1

32+

1

4+ 2 + 16 + 128 + 1024

So (Sehingga)

r =Un

Un − 1r = 8

Let k, n be number of inserted numbers and terms. We have k = 2 and

n = 6. The new ratio is:

Misal k, n masing-masing adalah banyaknya bilangan yang disisipkan dan

banyaknya suku sebelum disisikpan, maka diperoleh k = 2 dan n = 6.

Ratio yang baru adalah:

r′ = k+1√

r

r′ = 3√

8

r′ = 2

Number of the new terms is

Banyaknya suku yang baru adalah

n′ = n + (n− 1)k

n′ = 6 + (6− 1).2

n′ = 16

Page 60: Contents · PDF filegai batu loncatan untuk membuktikan teorema-teorema lainnya. Sebuah akibat dari beberapa teorema atau lemma disebut dengan korolary. Sebuah pernyataan yang tidak

Chapter 3. Algebra Functions 54

The desired geometric series is:

Deret geometri yang dicari adalah:

1

32+

1

16+

1

8︸ ︷︷ ︸+

1

4+

1

2+ 1

︸ ︷︷ ︸+2 + .... + 265 + 512︸ ︷︷ ︸

︸ ︷︷ ︸the inserted numbers (bilangan yang disisipkan)

+1024

2. Let a, b and c be any riel positive numbers such that abc = 1. Prove that1

a3(b+c)+ 1

b3(a+c)+ 1

c3(a+b)≥ 3

2.

Misal a, b dan c adalah bilangan riil positif sedemikian hingga abc = 1.

Buktikan bahwa 1a3(b+c)

+ 1b3(a+c)

+ 1c3(a+b)

≥ 32.

Solution. Let

S =1

a3(b + c)+

1

b3(a + c)+

1

c3(a + b)(3.20)

and a = 1x, b = 1

y, c = 1

z. Let T = x+y + z. Since abc = 1, we have xyz = 1.

Substituting the new a, b, c we have:

Solusi. Misal

S =1

a3(b + c)+

1

b3(a + c)+

1

c3(a + b)(3.21)

dan a = 1x, b = 1

y, c = 1

z. Misal T = x + y + z. Karena abc = 1 maka

xyz = 1. Dengan mensubstitusikan a, b, c yang baru didapat:

1

a3(b + c)=

x2

T − x=

T 2 − (T 2 − x2)

T − x=

T 2

T − x− T − x

1

b3(a + c)=

y2

T − y=

T 2 − (T 2 − y2)

T − y=

T 2

T − y− T − y

1

c3(a + b)=

z2

T − z=

T 2 − (T 2 − z2)

T − z=

T 2

T − z− T − z

Substituting the above equations into (3.21) we have the following.

Dengan mensubstitusikan semua persamaan di atas ke dalam (3.21) maka

Page 61: Contents · PDF filegai batu loncatan untuk membuktikan teorema-teorema lainnya. Sebuah akibat dari beberapa teorema atau lemma disebut dengan korolary. Sebuah pernyataan yang tidak

Chapter 3. Algebra Functions 55

diperoleh berikut ini.

S = T 2

(1

T − x+

1

T − y+

1

T − z

)− 3T − (x + y + z)

= T 2

(1

T − x+

1

T − y+

1

T − z

)− 4T

= T 2

(3

3

(1

T − x+

1

T − y+

1

T − z

))− 4T

= T 2 · 3( 1

T−x+ 1

T−y+ 1

T−z

3︸ ︷︷ ︸AM

)− 4T

≥ T 2 · 3(

311

T−x

+ 11

T−y

+ 11

T−z

3

︸ ︷︷ ︸HM

)− 4T

= T 2

(9

(T − x) + (T − y) + (T − z)

)− 4T

= T 2

(9

3T − (x + y + z)

)− 4T

= T 2

(9

2T

)− 4T

=9T

2− 4T =

T

2=

x + y + z

2=

3

2· x + y + z

3︸ ︷︷ ︸AM

≥ 3

23√

xyz =3

2

Therefore

Dengan demikian

∴ 1

a3(b + c)+

1

b3(a + c)+

1

c3(a + b)≥ 3

2

3. Given that f(x) is a polynomial of degree 2. When f(x) is divided by x+1,

the remainder is 3; when f(x) is divided by x− 3, the remainder is 23; and

Page 62: Contents · PDF filegai batu loncatan untuk membuktikan teorema-teorema lainnya. Sebuah akibat dari beberapa teorema atau lemma disebut dengan korolary. Sebuah pernyataan yang tidak

Chapter 3. Algebra Functions 56

when f(x) is divided by x − 2, the remainder is 15. Find the polynomial

f(x).

Diketahui f(x) adalah sebuah polinomial berderajad 2. Saat f(x) dibagi

dengan x + 1, maka sisanya adalah 3; saat f(x) dibagi dengan x− 3, maka

sisanya adalah 23; dan saat f(x) dibagi dengan x− 2, maka sisanya adalah

15. Tentukan polinomial f(x) tersebut.

Solution. Let f(x) = qx2 + mx + n. When f(x) is divided by x + 1, the

remainder is s(−1) = q(−1)2 + m(−1) + n = 3; when f(x) is divided by

x − 3, the remainder is s(3) = q(3)2 + m(3) + n = 23; and when f(x) is

divided by x− 2, the remainder is s(2) = q(2)2 + m(2) + n = 15. Thus, we

have

Misal f(x) = qx2 + mx + n. Saat f(x) dibagi dengan x + 1, maka sisanya

adalah s(−1) = q(−1)2 + m(−1) + n = 3; saat f(x) dibagi dengan x − 3,

maka sisanya adalah s(3) = q(3)2 + m(3) + n = 23; dan saat f(x) dibagi

dengan x−2, maka sisanya adalah s(2) = q(2)2 +m(2)+n = 15. Sehingga,

diperoleh

q −m + n = 3

9q + 3m + n = 23

4q + 2m + n = 15

Solving the three equations, we get q = 1,m = 3 and n = 5.

Dengan menyelesaikan ketiga persamaan di atas, maka didapat q = 1,m =

3 dan n = 5.

Page 63: Contents · PDF filegai batu loncatan untuk membuktikan teorema-teorema lainnya. Sebuah akibat dari beberapa teorema atau lemma disebut dengan korolary. Sebuah pernyataan yang tidak

¥¥¥¥¥¥ CHAPTER 4

TrigonometryTrigonometri

Trigonometry (from Greek trigonon ”triangle” + metron ”measure”) is a branch

of mathematics that deals with triangles, particularly those plane triangles in

which one angle has 90 degrees (right triangles). Trigonometry deals with rela-

tionships between the sides and the angles of triangles and with the trigonomet-

ric functions, which describe those relationships.

Trigonometri (berasal dari kata yunani trigonon ”segitiga” + metron ”mengukur”)

adalah suatu cabang matematika yang berkenaan dengan segitiga, khususnya

segitiga dalam bidang dimana satu sudut mempunyai besar sudut 90 derajad

(segitiga siku-siku). Trigonometri berkaitan dengan hubungan anatara sisi-sisi

dan sudut-sudut sebuah segitiga dengan fungsi trigonometri yang menggam-

barkan sebuah relasi diantaranya.

Trigonometry has applications in both pure mathematics and in applied math-

ematics, where it is essential in many branches of science and technology. It is

usually taught in secondary schools either as a separate course or as part of a

precalculus course. Trigonometry is informally called ”trig”.

Aplikasi trigonometri dapat ditemukan baik dalam matematika murni maupun

matematika terapan yang keduanya ini sangat berguna pada beberapa cabang

dalam sain dan teknologi. Trigonometri ini biasanya diajarkan di sekolah menen-

gah baik dalam matapelajaran tertentu atau merupakan bagian dari mata pela-

jaran kalkulus. Secara informal isitilah trigonometri ini disebut dengan ”trig”.

57

Page 64: Contents · PDF filegai batu loncatan untuk membuktikan teorema-teorema lainnya. Sebuah akibat dari beberapa teorema atau lemma disebut dengan korolary. Sebuah pernyataan yang tidak

Chapter 4. Trigonometry 58

4.1 Trigonometric FunctionFungsi Trigonometri

Consider the following figure, we define the trigonometric functions as : sin α =yr, cos α = x

r, tg α = y

x, ctg α = x

y, sec α = 1

cos α, cossec α = 1

sin α.

Perhatikan gambar berikut, maka fungsi-fungsi trigonometri didefinisikan se-

bagai: sin α = yr, cos α = x

r, tg α = y

x, ctg α = x

y, sec α = 1

cos α, cossec α = 1

sin α.

y

r yα

x

P (x, y)

α0

rx

Figure 4.1: The right triangle trigonometric system

Theorem 4.1.1 Pythagoras equality is stated as follow.

Teorema 4.1.1 Kesamaan Pythagoras dinyatakan dalam berikut ini.

1. sin2 α + cos2 α = 1, rumus identitas

2. 1 + ctg 2α = cossec 2α

3. tg 2α + 1 = sec2 α

The values of trigonometric function in every quadrant is described on a system

of quadrant, where 90o < α2 < 180o, 180o < α4 < 270o and 270 < α3 < 360o.

Considering the system of quadrant, the values of trigonometric function for any

angle α can be memorized easily. In Quadrant I, all of trigonometric function

Page 65: Contents · PDF filegai batu loncatan untuk membuktikan teorema-teorema lainnya. Sebuah akibat dari beberapa teorema atau lemma disebut dengan korolary. Sebuah pernyataan yang tidak

Chapter 4. Trigonometry 59

sign are positive, Quadrant II, only sinus is positive, Quadrant III, only tangent

is positive, and Quadrant IV, only cosinus is positive. Therefore, the keyword to

memorize it is ASTC.

Nilai fungsi pada masing-masing kuadran tertera dalam sistem kuadran, di-

mana 90o < α2 < 180o, 180o < α4 < 270o dan 270 < α3 < 360o. Dengan

memetakan pada sistem kuadran ini maka nilai fungsi trigonometri untuk se-

barang sudut α dapat dengan mudah diingat tandanya. Pada kuadaran I all

tanda fungsi trigonometri positif, kuadaran II hanya sinus yang positif, kuadaran

III hanya tangent yang positif dan kuadaran IV hanya cosinus yang positif.

Dengan demikian kata kunci untuk mengingatnya adalah ASTC.

Table 4.1: Trigonometric quadrant system

II I

sin α2 > 0 all > 0

III IV

tg α3 > 0 cos α4 > 0

Thus that, understanding the sign, to obtain the values of trigonometric function

we just need to remember its values at the first quadrant. Use the key words

ASTC to obtain the values on the following quadrant. For instance, to determine

sin 150o, we only need to know sin 30o as sin(180o − 30o) = sin 30o. We refer to

second quadrant that sinus is positive which follows that sin 30o = 12

Dengan memahami nilai tanda ini maka untuk hal tertentu kita hanya dapat

mengingat nilai nilai fungsi trigonometri pada kuadran pertama, selanjutnya

gunakan kata kunci ASTC untuk menentukan nilai-nilai di kuadran lainnya. Se-

bagai contoh: untuk menetukan sin 150o, kita cukup mengetahui sin 30o karena

Page 66: Contents · PDF filegai batu loncatan untuk membuktikan teorema-teorema lainnya. Sebuah akibat dari beberapa teorema atau lemma disebut dengan korolary. Sebuah pernyataan yang tidak

Chapter 4. Trigonometry 60

sin(180o − 30o) = sin 30o. Kita ingat di kuadran kedua bahwa sinus adalah po-

sisitif, sehingga sin 30o = 12

We summarize that the values of sinus, cosinus, tangent and cotangent for spe-

cial angles are as follows.

Nilai sinus, cosinus, tangent dan cotangent untuk sudut-sudut istimewa pada

kuadran pertama adalah sebagai berikut.

Table 4.2: The value of trigonometric functions for special angles

0o 30o 45o 60o 90o

sin 0 12

12

√2 1

2

√3 1

cos 1 12

√3 1

2

√2 1

20

tg 0 13

√3 1

√3 ∞

ctg ∞ √3 1 1

3

√3 0

The trigonometric functions in every quadrant does not remain the same. To

easily memorize the change of trigonometric functions of any angles (Xo − α),

it can be showed as the following system. From Quadrant I −→ IV in clock-

wise direction, the situations are B(berubah), Tb(tidak berubah), B(berubah),

Tb(tidak berubah) or in short form we have an acronym BTBT. Applying the

acronym ASTC, we easily put sign + or − on the resulted values.

Fungsi trigonometri di beberapa kuadran tidak selalu tetap. Untuk memper-

mudah ingatan kita tentang perubahan nilai fungsi trigonometri dari beberapa

sudut (Xo − α) dapat disajikan dalam sistem berikut. Dari kuadaran I −→IV berlawanan jarum jam, kondisinya adalah B(berubah), Tb(tidak berubah),

B(berubah), Tb(tidak berubah) atau dapat disingkat BTBT. Dengan menerap-

kan singkatan ASTC kita dapat memberikan tanda + dan − pada nilai-nilai itu

Page 67: Contents · PDF filegai batu loncatan untuk membuktikan teorema-teorema lainnya. Sebuah akibat dari beberapa teorema atau lemma disebut dengan korolary. Sebuah pernyataan yang tidak

Chapter 4. Trigonometry 61

dengan mudah.

Table 4.3: The value of trigonometric functions for any angle (Xo − α)

Tb B

sin(180o − α) = sin α sin(90o − α) = cos α

cos(180o − α) = − cos α cos(90o − α) = sin α

tg (180o − α) = − tg α tg (90o − α) = ctg α

ctg (180o − α) = − ctg α ctg (90o − α) = tg α

B Tb

sin(270o − α) = − cos α sin(360o − α) = − sin α

cos(270o − α) = − sin α cos(360o − α) = cos α

tg (270o − α) = ctg α tg (360o − α) = − tg α

ctg (270o − α) = tg α ctg (360o − α) = − ctg α

If an angle −α is obtained by turning a phase in clockwise direction then the

angle −α take a place in Quadrant IV. Thus, we know that the values of trigono-

metric functions for negative angles are exactly the values of trigonometric func-

tions on Quadrant IV. For detail, see the followings.

Bila sudut −α diperoleh dengan memutar jangka searah jarum jam maka sudut

−α berada pada kuadarn IV. Dengan demikian dapat dipahami bahwa nilai

fungsi trigonometri pada sudut negatif setara dengan nilai fungsi trigonometri

pada sistem di atas pada kuadran IV. Nilai selengkapnya disajikan dalam tabel

berikut.

Furthermore, we can develop a quadrant system for trigonometric function val-

ues for any angle (Xo + α). In this case, we generate an acronym TBTB as a

keyword to remember.

Page 68: Contents · PDF filegai batu loncatan untuk membuktikan teorema-teorema lainnya. Sebuah akibat dari beberapa teorema atau lemma disebut dengan korolary. Sebuah pernyataan yang tidak

Chapter 4. Trigonometry 62

Table 4.4: The value of trigonometric functions for negative angles

sin(−α) = sin(360o − α) = − sin α

cos(−α) = cos(360o − α) = cos α

tg (−α) = tg (360o − α) = − tg α

ctg (−α) = ctg (360o − α) = − ctg α

Disamping itu, dapat pula digambarkan sistem kuadran untuk menentukan

nilai fungsi trigonometri pada sudut (Xo + α). Dalam sistem ini diperoleh

singkatan TBTB sebagai kata kunci untuk mengingat.

Table 4.5: The value of trigonometric functions for any angle (Xo + α)

B Tb

sin(90o + α) = − cos α sin(360o + α) = sin α

cos(90o + α) = sin α cos(360o + α) = cos α

tg (90o + α) = − ctg α tg (360o + α) = tg α

ctg (90o + α) = − tg α ctg (360o + α) = ctg α

Tb B

sin(180o + α) = − sin α sin(270o + α) = cos α

cos(180o + α) = − cos α cos(270o + α) = − sin α

tg (180o + α) = tg α tg (270o + α) = − ctg α

ctg (180o + α) = ctg α ctg (270o + α) = − tg α

Page 69: Contents · PDF filegai batu loncatan untuk membuktikan teorema-teorema lainnya. Sebuah akibat dari beberapa teorema atau lemma disebut dengan korolary. Sebuah pernyataan yang tidak

Chapter 4. Trigonometry 63

Example.

Contoh.

1. Determine the value of sin(270o−30o)cos(180o−45o)

!

Tentukan nilai dari sin(270o−30o)cos(180o−45o)

!

Solution. Remember the angle (Xo−α), ASTC and BTBT, thus sin(270o−30o)cos(180o−45o)

=− cos 30o

− cos 45o =√

3/2.

Solusi. Ingat sudut (Xo−α), ASTC dan BTBT, maka sin(270o−30o)cos(180o−45o)

= − cos 30o

− cos 45o =√3/2.

2. Determine the value of tg (360o+60o)cos(180o+90o)

!

Tentukan nilai dari tg (360o+60o)cos(180o+90o)

!

Solution. Remember the angle (Xo+α), ASTC and TBTB, thus tg (360o+60o)cos(180o+90o)

=tg 60o

− sin 90o ==√

3.

Solusi. Ingat sudut (Xo+α), ASTC dan TBTB, maka tg (360o+60o)cos(180o+90o)

= tg 60o

− sin 90o =√3.

4.1.1 Sine and Cosine RuleAturan Sinus dan Cosinus

The following figure shows 4ABC and a circle of radius R.

Gambar dibawah ini memperlihatkan 4ABC dan lingkaran dengan jari-jari R.

Theorem 4.1.2 (Sine Rule) The sine rule states:

Theorem 4.1.3 (Aturan sinus) Aturan sinus mengatakan bahwa:a

sin α=

b

sin β=

c

sin γ= 2R

Page 70: Contents · PDF filegai batu loncatan untuk membuktikan teorema-teorema lainnya. Sebuah akibat dari beberapa teorema atau lemma disebut dengan korolary. Sebuah pernyataan yang tidak

Chapter 4. Trigonometry 64

Z C

B

c a

C

B

A b

β

β

0

C

B

c a

β

0

(i) (ii) (iii)

Figure 4.2: Triangle and circle of radius R

Proof. As an exercise.

Bukti. Sebagai bahan latihan.

Theorem 4.1.4 (Cosine Rule) The cosine rule states:

Teorema 4.1.2 (Aturan cosinus) Aturan cosinus mengatakan bahwa:

a2 = b2 + c2 − 2bc cos α

b2 = a2 + c2 − 2ac cos β

c2 = a2 + b2 − 2ab cos γ

Proof. Consider Figure 5.1(iii)

Bukti. Perhatikan Gambar 5.1(iii)

CZ = AC − AZ = b− c cos α

BZ = c sin α

BC2 = CZ2 + BZ2

a2 = (b− c cos α)2 + (c sin α)2

= b2 − 2bc cos α + c2(cos2 α + sin2 α)

= b2 − 2bc cos α + c2

Page 71: Contents · PDF filegai batu loncatan untuk membuktikan teorema-teorema lainnya. Sebuah akibat dari beberapa teorema atau lemma disebut dengan korolary. Sebuah pernyataan yang tidak

Chapter 4. Trigonometry 65

Thus a2 = b2 + c2 − 2bc cos α. In the same way, we can prove that b2 = a2 + c2 −2ac cos β and c2 = a2 + b2 − 2ab cos γ.

Sehingga a2 = b2 + c2 − 2bc cos α. Dengan cara yang sama, dapat dibuktikan

b2 = a2 + c2 − 2ac cos β dan c2 = a2 + b2 − 2ab cos γ. 2

Example. Prove that

Contoh. Buktikan bahwa

sin2 α = sin2 β + sin2 γ − 2 sin β sin γ cos α

sin2 β = sin2 α + sin2 γ − 2 sin α sin γ cos β

sin2 γ = sin2 α + sin2 β − 2 sin α sin β cos γ

Solution. The sine rule states that a = 2R sin α, b = 2R sin β and c = 2R sin γ.

Substituting into cosine rule above, we will get those formulas.

Solusi. Aturan sinus menyatakan bahwa a = 2R sin α, b = 2R sin β dan c =

2R sin γ. Substitusikan ke dalam aturan cosinus di atas maka akan diperoleh

rumus-rumus itu.

4.1.2 Formulas of Sum and Difference of AnglesRumus-rumus Jumlah dan Selisih Sudut

A circle of radius 1 with point A and B, where ∠AOX = α and ∠BOX = β, is

presented in Figure 4.3.

Sebuah lingkaran dengan jari-jari 1 dan titik A dan B, dimana ∠AOX = α dan

∠BOX = β, terlihat dalam Gambar 4.3.

Theorem 4.1.5 Let α and β be any angles. The formula of sum and difference of anglescan be stated as follows.

Page 72: Contents · PDF filegai batu loncatan untuk membuktikan teorema-teorema lainnya. Sebuah akibat dari beberapa teorema atau lemma disebut dengan korolary. Sebuah pernyataan yang tidak

Chapter 4. Trigonometry 66

O

A

y

Figure 4.3: Sum and Difference of Angles

Teorema 4.1.3 Diberikan sudut α and β, maka rumus penjumlahan dan pengurangansudut dapat dinyatakan sebagai berikut.

1. sin(α± β) = sin α cos β ± cos α sin β

2. cos(α± β) = cos α cos β ∓ sin α sin β

3. tg (α± β) =tg α± tg β

1∓ tg α tg β

Bukti. Consider Figure 4.3, we have A(cos α, sin α) and B(cos β, sin β). the length

of AB can be obtained by the formula:

Bukti. Perhatikan Gambar 4.3, kita memiliki A(cos α, sin α) dan B(cos β, sin β).

Panjang AB dapat dicari dengan rumus:

AB2 = (cos α− cos β)2 + (sin α− sin β)2

= 2(1− (cos α cos β + sin α sin β)). (4.1)

Cosine rule gives

Aturan cosinus memberikan Aturan cosinus memberikan

AB2 = OA2 + OB2 − 2 ·OA ·OB · cos ∠AOB

= 2(1− cos(β − α)). (4.2)

Page 73: Contents · PDF filegai batu loncatan untuk membuktikan teorema-teorema lainnya. Sebuah akibat dari beberapa teorema atau lemma disebut dengan korolary. Sebuah pernyataan yang tidak

Chapter 4. Trigonometry 67

From Equations (4.1) and (4.2), it follows

Dari persamaan (4.1) dan (4.2) memberikan

cos(α− β) = cos α cos β + sin α sin β,

which completes the proof. We leave other formulas as an exercise

dengan demikian terbuktilah. Rumus lainnya ditinggal untuk bahan latihan. 2

Theorem 4.1.6 Given an angle α, the formula of multiple angles is presented as follows.

Teorema 4.1.4 Diberikan sudut α, maka rumus rumus sudut rangkap dapat ditun-jukkan sebagai berikut.

1. sin(2α) = 2 sin α cos α (4.3)

2. cos(2α) = cos2 α− sin2 α (4.4)

3. tg (2α) =2 tg α

1− tg 2α(4.5)

Bukti. For Equation 4.3

Bukti. Untuk Persamaan 4.3

sin(2α) = sin(α + α) = sin α cos α± cos α sin α

= 2 sin α cos α.

We leave others for readers.

Pembuktian lainnya diserahkan kepada pembaca. 2

Theorem 4.1.7 Let α, β be any angles, the product formula of trigonometric functionis showed as follows:

Page 74: Contents · PDF filegai batu loncatan untuk membuktikan teorema-teorema lainnya. Sebuah akibat dari beberapa teorema atau lemma disebut dengan korolary. Sebuah pernyataan yang tidak

Chapter 4. Trigonometry 68

Teorema 4.1.5 Diberikan sebarang sudut α and β, maka rumus perkalian fungsi trigono-metri adalah sebagai berikut:

1. 2 sin α cos β = sin(α + β) + sin(α− β)

2. 2 cos α sin β = sin(α + β)− sin(α− β)

3. 2 cos α cos β = cos(α + β) + cos(α− β)

4. −2 sin α sin β = cos(α + β)− cos(α− β)

Theorem 4.1.8 Let x, y be any angles, the formula of sum and difference of trigonomet-ric functions are

Theorem 4.1.9 Diberikan sudut x and y, maka rumus penjumlahan dan penguranganfungsi trigonometri adalah:

1. sin x + sin y = 2 sin1

2(x + y) cos

1

2(x− y) (4.6)

2. cos x + cos y = 2 cos1

2(x + y) cos

1

2(x− y) (4.7)

3. sin x− sin y = 2 cos1

2(x + y) sin

1

2(x− y) (4.8)

4. cos x− cos y = −2 sin1

2(x + y) sin

1

2(x− y) (4.9)

Proof. Assuming that x = (α+β) and y = (α−β), and using Theorem4.1.5, we

can easily prove this theorem.

Bukti. Dengan memisalkan x = (α + β) dan y = (α − β), dan menggunakan

Teorema 4.1.5, maka teorema ini dapat dengan mudah dibuktikan. 2

Example. If 2 cos2 2x + sin(π2− 2x)− 6 = 0 then cos x = . . .

Contoh. Jika 2 cos2 2x + sin(π2− 2x)− 6 = 0 maka cos x = . . .

Page 75: Contents · PDF filegai batu loncatan untuk membuktikan teorema-teorema lainnya. Sebuah akibat dari beberapa teorema atau lemma disebut dengan korolary. Sebuah pernyataan yang tidak

Chapter 4. Trigonometry 69

Solution.

Solution.

2 cos2 2x + sin(π

2− 2x)− 6 = 0

2 cos2 2x + cos 2x− 6 = 0

(2 cos2 2x− 3)(cos 2x + 2) = 0

cos 2x =3

2∨ cos 2x = −2

For cos 2x = 32−→ 2 cos2 x − 1 = 3

2−→ cos x = ±1

2

√5. For cos 2x = −2 −→

2 cos2 x− 1 = −2, the value of cos x is undefined. Thus cos x = 12

√5 or −1

2

√5.

Untuk cos 2x = 32−→ 2 cos2 x− 1 = 3

2−→ cos x = ±1

2

√5. Untuk cos 2x = −2 −→

2 cos2 x− 1 = −2, nilai cos x tak terdefinisi. Sehingga cos x = 12

√5 atau −1

2

√5.

4.1.3 Trigonometric EquationPersamaan Trigonometri

The simple trigonometric equation can be solved by the following way:

Bentuk persamaan trigonometri sederhana dapat diselesaikan dengan cara berikut:

sin x = sin α =⇒ x1 = α + k · 360 ∨ x2 = (180o − α) + k · 360

cos x = sin α =⇒ x1 = α + k · 360 ∨ x2 = −α + k · 360

tg x = sin α =⇒ x1 = α + k · 180

ctg x = sin α =⇒ x1 = α + k · 180

To solve equation a cos x+ b sin x = c, we should transform into equation cos(x−α) = y first.

Sedangkan untuk menyelesaikan bentuk persamaan a cos x + b sin x = c, maka

Page 76: Contents · PDF filegai batu loncatan untuk membuktikan teorema-teorema lainnya. Sebuah akibat dari beberapa teorema atau lemma disebut dengan korolary. Sebuah pernyataan yang tidak

Chapter 4. Trigonometry 70

harus ditransformasikan terlebih dahulu ke dalam bentuk cos(x− α) = y.

a cos x + b sin x = c

cos x +b

asin x =

c

a

cos x +sin α

cos αsin x =

c

a, misal tg α =

b

a=

sin α

cos α

cos x cos α + sin α sin x =c

acos α

cos(x− α) =c

acos α, dimana cos α =

a√a2 + b2

(4.10)

So

Sehingga

a cos x + b sin x = c =⇒ cos(x− α) = c√a2+b2

Theorem 4.1.10 Equation of form a cos x + b sin x = c is solvable if c2 ≤ a2 + b2

Teorema 4.1.6 Bentuk persamaan a cos x + b sin x = c dapat diselesaikan apabila c2 ≤a2 + b2

From Equation (4.10), we have a cos x + b sin x = c =⇒ cos(x− α) = c√a2+b2

. The

value of cosine is −1 ≤ cos x ≤ 1, thus −1 ≤ c√a2+b2

≤ 1 =⇒ c2

a2+b2≤ 1 =⇒

c2 ≤ a2 + b2. Bukti. Sesuai Persamaan (4.10), maka a cos x + b sin x = c =⇒cos(x − α) = c√

a2+b2. Nilai cosinus terletak antara −1 ≤ cos x ≤ 1, sehingga

−1 ≤ c√a2+b2

≤ 1 =⇒ c2

a2+b2≤ 1 =⇒ c2 ≤ a2 + b2. 2

Example. Determine the value of x satisfying tg x + ctg x = 4 cos 2x

Contoh. Tentukan harga x yang memenuhi persamaan tg x + ctg x = 4 cos 2x.

Page 77: Contents · PDF filegai batu loncatan untuk membuktikan teorema-teorema lainnya. Sebuah akibat dari beberapa teorema atau lemma disebut dengan korolary. Sebuah pernyataan yang tidak

Chapter 4. Trigonometry 71

Solution.

Solusi.

tg x + ctg x = 4 cos 2x

sin2 x + cos2 x

cos x sin x= 4 cos 2x

1 = 4 cos 2x cos x sin x

sin 90o = 2 cos 2x sin 2x = sin 4x

The last equation shows sin 4x = sin 90o −→ 4x1 = 90o + k · 360o ∨ 4x1 =

(180o − 90o) + k · 360o. The value of x satisfying both of them {2212

o, 1121

2

o,

20212

o, 2921

2

o}.

Dari persamaan terakhir sin 4x = sin 90o −→ 4x1 = 90o + k · 360o ∨ 4x1 =

(180o − 90o) + k · 360o. Nilai x yang memenuhi keduanya adalah {2212

o, 1121

2

o,

20212

o, 2921

2

o}.

4.2 Limit Fungsi

In mathematics, the limit of a function is a fundamental concept in calculus and

analysis concerning the behavior of that function near a particular input. Infor-

mally, a function assigns an output f(x) to every input x. The function has a

limit L at an input p if f(x) is ”close” to L whenever x is ”close” to p. In other

words, f(x) becomes closer and closer to L as x moves closer and closer to p.

More specifically, when f is applied to each input sufficiently close to p, the re-

sult is an output value that is arbitrarily close to L. If the inputs ”close” to p

are taken to values that are very different, the limit is said to not exist. Formal

definitions, first devised in the early 19th century, are given below.

Dalam matematika, limit suatu fungsi adalah merupakan konsep dasar kalkulus

dan analisis terkait dengan karakteristik fungsi di sekitar suatu input tertentu.

Page 78: Contents · PDF filegai batu loncatan untuk membuktikan teorema-teorema lainnya. Sebuah akibat dari beberapa teorema atau lemma disebut dengan korolary. Sebuah pernyataan yang tidak

Chapter 4. Trigonometry 72

Secara informal, suatu fungsi akan memberikan output f(x) terhadap setiap in-

put x. Fungsi itu akan memberikan sebuah nilai pendekatan L atas suatu input

p jika f(x) ”dekat” dengan L apabila x juga ”dekat” dengan p. Dengan kata

lain, f(x) menjadi dekat dan semakin dekat dengan L sebagaimana x bergerak

dekat dan semakin dekat dengan p. Secara sepesifik, saat f diterapkan terhadap

setiap input yang cukup dekat dengan p, hasilnya adalah sebuah output yang

juga cukup dekat dengan L. Jika input yang dekat dengan p dimasukkan dan

hasilnya sangat berbeda maka limit ini dikatakan tidak terdefinisi. Definisi for-

mal, mucul pertama kali diawal abad 19-an, diberikan dalam berikut ini.

Definition 4.2.1 Intuitive Defintion of Limit. If f(x) is a real function and c is areal number then:

limx→c

f(x) = L,

that is f(x) can be sufficiently closer to L as x moves close to c.

Definisi 4.2.1 Pengertian limit secara intuitif. Jika f(x) adalah fungsi riil dan c

adalah bilangan riil, maka:limx→c

f(x) = L,

berarti f(x) dapat dibuat sedekat mungkin dengan L sebagaimana nilai x bergerak dekatdengan bilangan c.

Theorem 4.2.1 Let f, g be functions whose limits exist for c. Let k, n be respectivelyconstant and positive real number. We have the followings.

Page 79: Contents · PDF filegai batu loncatan untuk membuktikan teorema-teorema lainnya. Sebuah akibat dari beberapa teorema atau lemma disebut dengan korolary. Sebuah pernyataan yang tidak

Chapter 4. Trigonometry 73

Teorema 4.2.1 Misal f, g adalah fungsi-fungsi yang limitnya terdefinisi untuk c. Misalk, n adalah masing-masing sebuah konstanta dan bilangan positif real. Maka akanberlaku berikut ini.

1. limx→c

k = k and (dan) limx→c

x = c

2. limx→c

kf(x) = k limx→c

f(x)

3. limx→c

[f(x)± g(x)] = limx→c

f(x)± limx→c

g(x)

4. limx→c

f(x) · g(x) = limx→c

f(x) · limx→c

g(x)

5. limx→c

f(x)

g(x)=

limx→c f(x)

limx→c g(x), provided (asalkan) lim

x→cg(x) 6= 0

6. limx→c

[f(x)]n =[

limx→c

f(x)]n

7. limx→c

n√

f(x) = n

√limx→c

f(x), provided (asalkan) limx→c

f(x) ≥ 0

for (untuk) n even (genap)

Definition 4.2.2 Newton’s difference quotient. Let f be a function. Function f ′

(read as ”f prime”) is the derivative of f whose value on any x is

f ′(x) =df(x)

dx= lim

h→0

f(x + h)− f(x)

h

provided the limit exists.

Definition 4.2.3 Hasil beda Newton. Diketahui suatu fungsi f . Fungsi f ′ (dibaca”f aksen”) adalah turunan dari f yang nilainya pada sebarang x adalah

f ′(x) =df(x)

dx= lim

h→0

f(x + h)− f(x)

h

asalkan limit ini ada.

Page 80: Contents · PDF filegai batu loncatan untuk membuktikan teorema-teorema lainnya. Sebuah akibat dari beberapa teorema atau lemma disebut dengan korolary. Sebuah pernyataan yang tidak

Chapter 4. Trigonometry 74

Example. If f(x) = x3, then determine f ′(x).

Contoh. Jika f(x) = x3, maka tentukan f ′(x).

Solution.

Solusi.

f ′(x) = limh→0

f(x + h)− f(x)

h= lim

h→0

(x + h)3 − x3

h

= limh→0

3x2h + 3xh2 + h3

h= lim

h→0(3x2 + 3xh + h2)

= 3x2.

We expect the readers to remember the formula of derivative function for alge-

braic, exponential or trigonometry. Expressing the derivative definition is only

to show the connection between derivative concept and the solution of limit of

function. This module does not mean to discuss the concept of derivative in

detail. In the followings, we describe the solution of limit by using a derivative

concept.

Pembaca diharapkan dapat mengingat rumus-rumus turunan fungsi aljabar,

eksponensial dan trigonometri. Pencantuman definisi turunan dalam bagian

ini hanya untuk menggambarkan kaitan konsep turunan dengan penyelesaian

limit fungsi. Sehingga modul ini tidak dimaksudkan untuk membahas konsep

turunan secara detail. Berikut ini diberikan metode penyelesaian limit fungsi

dengan memanfaatkan konsep turunan ini.

In general, if f(x) = xn the the derivative function f ′(x) is:

Secara umum, jika f(x) = xn maka fungsi turunan f ′(x) adalah:

f ′(x) = ddx

xn = nxn−1

Page 81: Contents · PDF filegai batu loncatan untuk membuktikan teorema-teorema lainnya. Sebuah akibat dari beberapa teorema atau lemma disebut dengan korolary. Sebuah pernyataan yang tidak

Chapter 4. Trigonometry 75

4.2.1 Solution TechniquesMetode Penyelesaian

Some techniques in solving limit have been developed. One of the most popular

is by recognizing an undefined form of function y = F (x) when an input x → p is

substituted. Those undefined forms are 00,∞×∞, ∞∞ ,∞±∞,∞× 0, 00,∞0,∞∞.

Beberapa cara menyelesaikan limit fungsi telah dikembangkan, salah satu yang

paling populer yaitu dengan cara mengetahui bentuk takterdefinisi dari funggsi

y = F (x) apabila masukan x → p disubstitusikan. Bentuk-bentuk takterdefinisi

itu adalah: 00,∞×∞, ∞∞ ,∞±∞,∞× 0, 00,∞0,∞∞.

Based on the undefined forms, we can solve the limit of function by the follow-

ing principles.

Berdasarkan bentuk tak terdefinisi di atas maka dalam menyelesaikan limit fungsi

dapat dilakukan dengan cara berikut.

1. Forms (Bentuk) 00

and (dan) ∞∞ .

limx→c

f(x)

g(x)= lim

x→c

f ′(x)

g′(x)(4.11)

2. Form (Bentuk) ∞× 0.

limx→c

f(x) · g(x) = limx→c

(f(x) · g(x))×1

f(x)

1f(x)

= limx→c

g(x)1

f(x)

(4.12)

such that we get forms 00

or ∞∞ .

sedemikian hingga diperoleh bentuk 00

atau ∞∞ .

3. Form Bentuk ∞±∞.

limx→c

[f(x)± g(x)] = limx→c

1g(x)

± 1f(x)

1f(x)·g(x)

(4.13)

Page 82: Contents · PDF filegai batu loncatan untuk membuktikan teorema-teorema lainnya. Sebuah akibat dari beberapa teorema atau lemma disebut dengan korolary. Sebuah pernyataan yang tidak

Chapter 4. Trigonometry 76

such that we get forms 00

or ∞∞

sedemikian hingga diperoleh bentuk 00

atau ∞∞ .

4. When we do not end up with the three forms, we can solve it with simpli-

fying the function by using special techniques and mathematics properties,

such as factorization, identity formulas etcetera. For the limit of trigono-

metric function can be the properties of trigonometry in Chapter 3

Apabila tidak diperoleh ketiga bentuk di atas maka dapat diselesaikan

dengan menyederhanakan fungsi itu dengan menerapkan teknik-teknik

khusus dan sifat-sifat matematis, seperti pemfaktoran, rumus identitas

dan lain sebagainya. Untuk fungsi trigonometri dapat menggunakan sifat-

sifat trigonometri pada Bab 3.

4.2.2 Limit of Algebraic FunctionLimit Fungsi Aljabar

Theorem 4.2.2 Given that a function f(x) = a0xm + a1x

m−1 + a2xm−2 + · · · + am

and g(x) = b0xn + b1x

n−1 + b2xn−2 + · · ·+ bn. So limx→∞

f(x)g(x)

is

Teorema 4.2.2 Diketahui suatu fungsi f(x) = a0xm + a1x

m−1 + a2xm−2 + · · · + am

dan g(x) = b0xn + b1x

n−1 + b2xn−2 + · · ·+ bn maka limx→∞

f(x)g(x)

adalah

=

a0

b0, jika m = n,

0, jika m < n,

∞, jika m > n.

(4.14)

Proof. We leave for a group discussion. (Hint. When m = n, divide f(x), g(x)

by xm = xn; when m < n, divide f(x), g(x) by xn; and when m > n, divide

Page 83: Contents · PDF filegai batu loncatan untuk membuktikan teorema-teorema lainnya. Sebuah akibat dari beberapa teorema atau lemma disebut dengan korolary. Sebuah pernyataan yang tidak

Chapter 4. Trigonometry 77

f(x), g(x) by xm).

Bukti. Sebagai bahan diskusi kelompok. (Petunjuk. Bila m = n maka bagi

f(x), g(x) dengan xm = xn; bila m < n bagi f(x), g(x) dengan xn; dan bila m > n

maka bagi f(x), g(x) dengan xm). 2

Theorem 4.2.3 Given that a function f(x) = a0xm + a1x

m−1 + a2xm−2 + · · · + am

and g(x) = b0xn + b1x

n−1 + b2xn−2 + · · ·+ bn. So limx→0

f(x)g(x)

is

Teorema 4.2.3 Diketahui suatu fungsi f(x) = a0xm + a1x

m−1 + a2xm−2 + · · · + am

dan g(x) = b0xn + b1x

n−1 + b2xn−2 + · · ·+ bn, maka limx→0

f(x)g(x)

adalah

=

a0

b0, jika m = n,

∞, jika m < n,

0, jika m > n.

(4.15)

Theorem 4.2.4 Given that a function

f(x) =√

ax2 + bx + c and g(x) =√

px2 + qx + r.

So limx→∞[f(x)− g(x)] is

Teorema 4.2.4 Diketahui suatu fungsi

f(x) =√

ax2 + bx + c dan g(x) =√

px2 + qx + r,

maka limx→∞[f(x)− g(x)] adalah

=

b−q2a

√a, jika m = n,

−∞, jika a < p,

+∞, jika a > p.

(4.16)

Page 84: Contents · PDF filegai batu loncatan untuk membuktikan teorema-teorema lainnya. Sebuah akibat dari beberapa teorema atau lemma disebut dengan korolary. Sebuah pernyataan yang tidak

Chapter 4. Trigonometry 78

Proof.

Bukti.

= limx→∞

[√ax2 + bx + c−

√px2 + qx + r

]·√

ax2 + bx + c +√

px2 + qx + r√ax2 + bx + c +

√px2 + qx + r

= limx→∞

(b− q)x + (c− r)√ax2 + bx + c +

√px2 + qx + r

, karena a = p

= limx→∞

(b− q) + (c−r)x√

a + bx

+ cx2 +

√p + q

x+ r

x2

=(b− q)

2a

√a.

Hence limx→∞[√

ax2 + bx + c−√

px2 + qx + r]

= (b−q)2a

√a. When the case a > p

and a < p, we leave for a group discussion.

Sehingga limx→∞[√

ax2 + bx + c−√

px2 + qx + r]

= (b−q)2a

√a. Untuk kasus a > p

dan a < p dapat digunakan sebagai bahan diskusi kelompok. 2

Apart from the three cases, in general we can solve the limit of algebraic function

by referring the methods in Subsection 4.2.1.

Diluar tiga kasus di atas, secara umum limit fungsi aljabar dapat diselesaikan

dengan mengacu pada metode-metode yang terdapat subbab 4.2.1.

Example. Determine limx→ax2+(3−a)x−3a

x−a

Contoh. Tentukan limx→ax2+(3−a)x−3a

x−a

Solution. Case 4.11, we have

Solution. Kasus 4.11, maka

limx→a

x2 + (3− a)x− 3a

x− a= lim

x→a

2x + (3− a)

1= a + 3

Example. Determine limx→c

√x3+2x2c+3xc2−5−√6c3−5)

x−c

Contoh. Tentukan limx→c

√x3+2x2c+3xc2−5−√6c3−5)

x−c

Page 85: Contents · PDF filegai batu loncatan untuk membuktikan teorema-teorema lainnya. Sebuah akibat dari beberapa teorema atau lemma disebut dengan korolary. Sebuah pernyataan yang tidak

Chapter 4. Trigonometry 79

Solution. Case 4.11, we have

Solusi. Kasus 4.11, maka

limx→c

√x3 + 2x2c + 3xc2 − 5−√6c3 − 5)

x− c= lim

x→c

3x2 + 4xc + 3c2

2√

x3 + 2x2c + 3xc2 − 5

=c√

11

2√

6c3 − 5

4.2.3 Limit of Trigonometric FunctionLimit Fungsi Trigonometri

Theorem 4.2.5 The followings hold for limit of trigonometry

Teorema 4.2.5 Berikut ini berlaku untuk limit trigonometri

limx→0

sin x

x= lim

x→0

x

sin x= 1 (4.17)

limx→0

sin ax

bx=

a

b(4.18)

limx→0

tan ax

bx=

a

b(4.19)

limx→0

sin ax

tan bx=

a

b(4.20)

Theorem 4.2.6 The followings hold for limit of trigonometry

Page 86: Contents · PDF filegai batu loncatan untuk membuktikan teorema-teorema lainnya. Sebuah akibat dari beberapa teorema atau lemma disebut dengan korolary. Sebuah pernyataan yang tidak

Chapter 4. Trigonometry 80

Teorema 4.2.6 Berikut ini berlaku untuk limit trigonometri

limx→0

sinn x

xn= lim

x→0

xn

sinn x= 1 (4.21)

limx→0

sinn ax

bxn=

an

b(4.22)

limx→0

tann ax

bxn=

an

b(4.23)

limx→0

sinn ax

tann bx=

an

bn(4.24)

Proof. Case (4.22) can be proved by the following way:

Bukti. Kasus (4.22) dapat dibuktikan dengan cara berikut:

limx→0

sinn ax

bxn= lim

x→0

(sin ax)n

bxn

=1

b

(limx→0

sin ax

x

)n

=an

b, respecting to (berdasarkan) Theorem (Teorema) 4.2.6

For the next proof, we leave for an exercise.

Untuk pembuktian selanjutnya diberikan sebagai bahan latihan. 2

Likely the limit of algebraic function, the method for solving trigonometric func-

tion is relatively the same. But, Limit of trigonometric function in general has

undefined output when the input is taken.

Sebagaimana limit fungsi aljabar metode penyelesaian limit fungsi trigonometri

dapat dilakukan dengan cara yang sama. Namun demikian pada umumnya

limit fungsi trigonometri mempunyai bentuk takterdefinisi apabila inputnya di-

gantikan.

Example. Determine limx→0sin 8x−sin 4x

7x

Contoh. Tentukan limx→0sin 8x−sin 4x

7x

Page 87: Contents · PDF filegai batu loncatan untuk membuktikan teorema-teorema lainnya. Sebuah akibat dari beberapa teorema atau lemma disebut dengan korolary. Sebuah pernyataan yang tidak

Chapter 4. Trigonometry 81

Solution. Equation (4.8), so the solution can be obtained in two ways. First, we

should understand the properties of trigonometric function.

Solusi. Persamaan (4.8), maka penyelesaian dapat dilakukan dengan dua cara,

pertama dengan memahami sifat-sifat fungsi trigonometri berikut.

limx→0

sin 8x− sin 4x

7x= lim

x→0

2 cos 6x sin 2x

7x

= limx→0

2 cos 6x · sin 2x

7x

= limx→0

2 cos 6x · limx→0

sin 2x

7x

=4

7

Secondly, we should consider that this problem meets Case (4.11), so

Cara kedua adalah dengan mempertimbangkan bahwa soal ini termasuk kasus

(4.11), sehingga

limx→0

sin 8x− sin 4x

7x= lim

x→0

8 cos 8x− 4 cos 4x

7x

=4

7

(It must be more efficient!)

(Tentu ini lebih efisien!)

PROBLEMS AND SOLUTIONS

SOAL-SOAL DAN PEMBAHASAN

1. Let α, β and γ be angles of a triangle. Determine sine and cosine the angles

α + β, α + γ and β + γ

Misal α, β dan γ adalah sudut-sudut dalam segitiga. Tentukan sinus dan

cosinus dari sudut-sudut α + β, α + γ dan β + γ

Page 88: Contents · PDF filegai batu loncatan untuk membuktikan teorema-teorema lainnya. Sebuah akibat dari beberapa teorema atau lemma disebut dengan korolary. Sebuah pernyataan yang tidak

Chapter 4. Trigonometry 82

2. Solution. The sum of all triangle angles is α + β + γ = 180o. Consider

the angle (Xo − α), the acronyms ASTC and BTBT such that sin(α + β) =

sin(180o − γ) = sin γ. For other problems we leave for exercise.

Solusi. Jumlah seluruh sudut segitiga α + β + γ = 180o. Ingat sudut

(Xo−α), ASTC dan BTBT, maka untuk sin(α + β) = sin(180o− γ) = sin γ.

Selanjutnya untuk pertanyaan lainnya diberikan sebagai latihan.

3. Prove that sin 5x+sin 3xcos 5x+cos 3x

= tg 4x

Buktikan bahwa sin 5x+sin 3xcos 5x+cos 3x

= tg 4x

Solution. Ordinary way. From the formula above:

Solusi. Cara biasa. Sesuai dengan rumus di atas:

sin A + sin B = 2 sin1

2(A + B) cos

1

2(A−B)

cos A + cos B = 2 cos1

2(A + B) cos

1

2(A−B)

Hence, we get

Sehingga diperoleh:

sin 5x + sin 3x = 2 sin1

2(5x + 3x) cos

1

2(5x− 3x)

= 2 sin 4x cos x

cos 5x + cos 3x = 2 cos1

2(5x + 3x) cos

1

2(5x− 3x)

= 2 cos 4x cos x

Therefore

Dengan demikian

sin 5x + sin 3x

cos 5x + cos 3x=

2 sin 4x cos x

2 cos 4x cos x= tg 4x

Page 89: Contents · PDF filegai batu loncatan untuk membuktikan teorema-teorema lainnya. Sebuah akibat dari beberapa teorema atau lemma disebut dengan korolary. Sebuah pernyataan yang tidak

Chapter 4. Trigonometry 83

4. Prove that sin 54o × sin 18o = 14

Buktikan bahwa sin 54o × sin 18o = 14

Solution. If we multiple by 2 cos 18o

2 cos 18o then we will obtain

Solusi. Jika kita mengalikan dengan 2 cos 18o

2 cos 18o maka kita akan mendapat

sin 45o sin 18o =(2 sin 18o cos 18o) sin 45o

2 cos 18o=

sin 36o cos(90o − 54o)

2 sin(90o − 18o)

=sin 36o cos 36o

2 sin 72o=

sin 36o cos 36o

2 sin(36o + 36o)

=sin 36o cos 36o

2(2 sin 36o cos 36o)=

1

4

5. Determine limx→1(1

x4−2x+1− 1

x7−1)

Tentukan limx→1(1

x4−2x+1− 1

x7−1)

Solution. Case category 4.13, thus

Solusi. Kategori kasus 4.13, maka

limx→1

(1

x4 − 2x + 1− 1

x7 − 1) = lim

x→1

(x4 − 2x + 1)− (x7 − 1)

(x4 − 2x + 1)(x7 − 1)

refer to Case 4.11, we have

Menjadi kategori kasus 4.11, maka

limx→1

(x4 − 2x + 1)− (x7 − 1)

(x4 − 2x + 1)(x7 − 1)= lim

x→1

(4x3 − 2− 7x6)

(4x3 − 2)(x7 − 1) + 7(x4 − 2x + 1)x6

= ∞

6. Determine limx→∞ 5x5−x3+2x2−x+1413x5−5

Tentukan limx→∞ 5x5−x3+2x2−x+1413x5−5

Solution. Case 4.14, thus limx→∞ 5x5−x3+2x2−x+1413x5−5

= 513

Solusi. Kasus 4.14, maka limx→∞ 5x5−x3+2x2−x+1413x5−5

= 513

7. Determine limx→03

√−18−5x7−x9+243x12

7−x+9x5+9x12

Tentukan limx→03

√−18−5x7−x9+243x12

7−x+9x5+9x12

Page 90: Contents · PDF filegai batu loncatan untuk membuktikan teorema-teorema lainnya. Sebuah akibat dari beberapa teorema atau lemma disebut dengan korolary. Sebuah pernyataan yang tidak

Chapter 4. Trigonometry 84

Solution. Case 4.15, applying Theorem 4.2.1, thus we have

Solusi. Kasus 4.15, dengan menerapkan Teorema 4.2.1 maka diperoleh

limx→0

3

√−18− 5x7 − x9 + 243x12

7− x + 9x5 + 9x12=

3

√limx→0

−18− 5x7 − x9 + 243x12

7− x + 9x5 + 9x12

=3

√243

9= 3

8. Determine limx→∞[√

16x2 + 6x + 7−√16x2 + 10x + 6]2

Tentukan limx→∞[√

16x2 + 6x + 7−√16x2 + 10x + 6]2

Solution. Case 4.15, applying Theorem 4.2.1, thus we have

Solusi. Kasus 4.15, dengan menerapkan Teorema 4.2.1 maka diperoleh

limx→∞

[√16x2 − 5x + 7−

√16x2 + 10x + 6

]2

=[

limx→∞

√16x2 − 5x + 7−

√16x2 + 10x + 6

]2

=[6− 10

2× 16

√16

]2=

1

4

9. Obtain the value of the following limits

Tentukan harga limit berikut ini:

(i) limx→0

2x

sin 3x(ii) lim

x→0

tg 5x

6x

(iii) limx→0

sin 8x

tg 5x(iv) lim

x→0

3 tg 2x

sin 12x

Solution. From Theorem 4.2.6, thus (i) = 23; (ii) = 5

6; (iii) = 8

5; (iv) = 1

2.

Solution. Sesuai Teorema 4.2.6, maka (i) = 23; (ii) = 5

6; (iii) = 8

5; (iv) =

12.

10. Determine limx→0 cossec 4x− 15x+x2

Tentukan limx→0 cossec 4x− 15x+x2

Solution. Case 4.14, thus

Page 91: Contents · PDF filegai batu loncatan untuk membuktikan teorema-teorema lainnya. Sebuah akibat dari beberapa teorema atau lemma disebut dengan korolary. Sebuah pernyataan yang tidak

Chapter 4. Trigonometry 85

Solusi. Kategori kasus 4.13, maka

limx→0

cossec 4x− 1

5x + x2= lim

x→0

(5x + x2)− sin 4x

(5x + x2) sin 4x

= limx→0

(5 + 2x)− cos 4x

(5 + 2x) sin 4x + 4(5x + x2) cos 4x

=4

0= ∞

11. Determine limx→0sin3 3x−sin3 3x cos 2x

4x2 tg 39x

Tentukan limx→0sin3 3x−sin3 3x cos 2x

4x2 tg 39x

Solution. Referring Theorem 4.14, thus

Solusi. Mengacu pada Teorema 4.2.6, maka

limx→0

sin3 3x− sin3 3x cos 2x

4x2 tg 39x= lim

x→0

sin3 3x(1− cos 2x)

4x2 tg 39x

= limx→0

sin3 3x

4 tg 39x· 2 sin2 x

x2

=1

54

Page 92: Contents · PDF filegai batu loncatan untuk membuktikan teorema-teorema lainnya. Sebuah akibat dari beberapa teorema atau lemma disebut dengan korolary. Sebuah pernyataan yang tidak

¥¥¥¥¥¥ CHAPTER 5

Geometri

5.1 Segitiga

Definition 5.1.1 Poligon adalah gabungan himpunan titik-titik P1, P2, . . . , Pn den-gan ruas garis P1P2, P2P3, . . . , Pn−1Pn, PnP1 sedemikian hingga jika dua sebarang ruasgaris berpotongan maka titik potongnya adalah salah satu dari titik-titik P1, P2, . . . , Pn.Selanjutnya P1, P2, . . . , Pn disebut titik-titik sudut poligon dan P1P2, P2P3, . . . , Pn−1Pn, PnP1

disebut sisi-sisi poligon.

Definition 5.1.2 Transversal adalah suatu garis yang memotong dua garis lain di duatitik yang berlainan.

Theorem 5.1.1 Jika dua garis sejajar dipotong oleh transversal maka sudut-sudut se-hadapnya, bersebrangan luarnya dan bersebrangan dalamnya kongruen.

γ

6

4 1

87

f

g

h

3 2

5

(i) (ii)CB

A

c b

β

Figure 5.1:

86

Page 93: Contents · PDF filegai batu loncatan untuk membuktikan teorema-teorema lainnya. Sebuah akibat dari beberapa teorema atau lemma disebut dengan korolary. Sebuah pernyataan yang tidak

Chapter 4. Geometri 87

Definition 5.1.3 Segitiga adalah poligon yang bersisi tiga.

Lema 5.1.1 Beberapa hal yang dipenuhi segitiga (lihat Gambar 5.1) adalah:

1. Jumlah semua sudut segitiga adalah 180o

2. Jika AB = AC maka ∠B = ∠C, dan jika AB = AC = BC maka ∠A = ∠B =

∠C.

Bukti. Bukti nomor 2. Perhatikan Gambar 5.1 (ii), sin α = tc

dan sin γ = tb

karena b = c maka α = γ. 2

GEOMETRI RIRIS

Pada bab ini akan dibahas mengenai teknik untuk menyelesaikan masalah-masalah

geometri. Pada umumnya, banyak masalah segi banyak yang dapat disele-

saikan dengan mengunakan konsep segitiga dan lingkaran. Oleh sebab itu, kali

ini akan dibahas hal-hal penting yang perlu diketahui menegenai segitiga dan

lingkaran.

Sebelumnya perhatikan definisi berikut:

Definisi

Diberikan dua titik A dan B. Garis yang menghubungkan kedua titik ini ke-

mudian disebut garis AB dan dari dua titik ini akan terbentuk sebuah segmen

garis dengan panjang tertentu. Selanjutnya segmen berarah AB dan BA mem-

punyai panjang yang sama tetapi arahnya berlawanan. Jarak dari titik A dan B

kemudian dinotasikan AB.

5.2 Segitiga

Beberapa hal penting dalam sebuah segitiga ABC yaitu

Page 94: Contents · PDF filegai batu loncatan untuk membuktikan teorema-teorema lainnya. Sebuah akibat dari beberapa teorema atau lemma disebut dengan korolary. Sebuah pernyataan yang tidak

Chapter 4. Geometri 88

A

βγ

C

α

B

1. Jumlah ketiga sudutnya adalah 1800

2. Aturan sinusnyasin α

BC=

sin β

AC=

sin γ

AB

3. Aturan kosinus BC2 = AB2 + AC2 − 2.AB.AC cos α

Definisi kesebangunan:

α

C

Q

α

γ

β B

A

γR

P

β

Dua buah segitiga ABC dan PQR dikatakan sebangun jika dan hanya jika mem-

punyai sudut-sudut bersesuaian yang sama besar atau sisi-sisi yang bersesuaian

mempunyai perbandingan yang sama pula. ABPQ

= BCQR

= CARP

Teorema:

Luas segitiga-segitiga yang tingginya sama sebanding dengan alas-alasnya.

Page 95: Contents · PDF filegai batu loncatan untuk membuktikan teorema-teorema lainnya. Sebuah akibat dari beberapa teorema atau lemma disebut dengan korolary. Sebuah pernyataan yang tidak

Chapter 4. Geometri 89

1. Pada segitiga ABC,M terletak pada sisi AB sehingga AM : MB = 1 : 3

dan N pada sisi AC sehingga AN : AC = 3 : 5. berapakah L4MNC

L4ABC

(Soal kompetensi matematika XVIII Jakarta)

Jawab:

1A

B

C3

3

5

L4ABC =1

2bc sin A

L4MNC = L4AMC − L4AMN

=1

2.1

4c.b sin A− 1

2.1

4c.

3

8b sin A

=5

64bc sin A

Jadi, didapatkan L4MNC

L4ABC

12bc sin A

564

bc sin A

2. Tentukan perbandingan luas antara daerah yang diarsir dan yang tidak

diarsir pada gambar berikut ini!

Jawab:

Perhatikan gambar!

Dari gambar tersebut jelas terlihat bahwa perbandingan antara daerah yang

diarsir dengan daerah yang tidak diarsir 1 : 1.

Garis-garis yang kongruen dan titik kolinear

Page 96: Contents · PDF filegai batu loncatan untuk membuktikan teorema-teorema lainnya. Sebuah akibat dari beberapa teorema atau lemma disebut dengan korolary. Sebuah pernyataan yang tidak

Chapter 4. Geometri 90

Teorema: CEV A

Jika titik D, E dan F terletak pada sisi-sisi BC, CA dan AB dari4ABC sedemikian

hingga garis AD, BE dan CF kongruen melalui titik 0, maka

BD

DC.CE

EA.AF

FB= 1

Bukti:

E D

F

C

BA

O

Page 97: Contents · PDF filegai batu loncatan untuk membuktikan teorema-teorema lainnya. Sebuah akibat dari beberapa teorema atau lemma disebut dengan korolary. Sebuah pernyataan yang tidak

Chapter 4. Geometri 91

Pada 4ABD dan 4ADC mempunyai tinggi yang sama, demikian halnya den-

gan 4OAB dan 4ODC. Menurut teorema sebelumnya diperoleh

BD

DC=4ABD

4ADC=4OBD

4ODC=4ABD −4OBD

4ADC −4ODC=4ABO

4CAO

Kemudian pada 4CBE dan 4ABE mempunyai tinggi yang sama, demikian

halnya dengan 4COE dan 4AOE. Menurut teorema sebelumnya diperoleh

CE

EA=4CBE

4ABE=4COE

4AOE=4CBE −4COE

4ABE −4AOE=4BCO

4ABO

Dengan cara yang sama, diperoleh pula AFFB

= 4CAO4BCO

Dari sisi didapatkan

BD

DC.CE

EA.AF

FB=4ABO

4CAO.4BCO

4ABO.4CAO

4BCO= 1

Terbukti

Contoh:

D, E dan F adalah titik-titik pada sisi BC, CA dan AB dari4ABC dan AD,BE

dan CF kongruen terhadap titik O.

Tunjukkan bahwa ODAD

+ OEBE

+ OFCF

= 1!

Jawab:

Perhatikan gambar :

E D

F

C

BA

O

Menurut teorema CEV A didapatkan

DO

AD=

L4ODB

L4ADB

=L4ODC

L4ADC

=L4ODB + L4ADC

L4ADB + L4ADC

=L4L4

Page 98: Contents · PDF filegai batu loncatan untuk membuktikan teorema-teorema lainnya. Sebuah akibat dari beberapa teorema atau lemma disebut dengan korolary. Sebuah pernyataan yang tidak

Chapter 4. Geometri 92

Kemudian dengan cara yang sama didapatkan pula

OE

BE=

L4OCA

L4ABC

danOF

CF=

L4OAB

L4ABC

Dari ketiga persamaan diatas, maka

OD

AD+

OE

BE+

OF

CF=

L4OBC

L4ABC

+L4OCA

L4ABC

+L4OAB

L4ABC

= 1

Teorema berikut merupakan kebalikan dari teorema CEV A

Teorema:

Jika tiga titik D, E dan F terletak pada sisi-sisi 4ABC yaitu di BC, CA dan AB

sedemikian hingga berlaku BDDC

.CEEA

.AFFB

= 1, maka AD, BE dan CF adalah kon-

gruen.

Bukti:

Misalkan AD dan BE berpotongan di O, perpanjangan CO akan memotong AB

di F ′. Karena AD, BE dan CF ′ kongruen maka BDDC

.CEEA

.AF ′F ′B = 1

karena diketahui pula bahwa BDDC

.CEEA

.AFFB

= 1, maka dari dua persamaan ini di-

dapatkan AFFB

= AF ′F ′B ⇔ AB−FB

FB= AB−F ′B

F ′BAFFB

= AF ′F ′B ⇔ AB

FB− 1 = AB

F ′B − 1AFFB

= AF ′F ′B ⇔ AB

FB= AB

F ′B

Dari sini dapat dilihat bahwa FB = F ′B, sehingga F dan F ′ berhimpit. Jadi

titik O juga dilalui CF , sehingga terbukti AD, BE dan CF kongruen.

Contoh

Buktikan bahwa semua garis tinggi dalam 4ABC adalah kongruen.

Jawab:

Misalkan AD, BE dan CF adalah garis4ABC. Sehingga BE⊥CA dan CF⊥AB.

Diperoleh pula CF⊥AB. Diperoleh pula 4AEB dengan 4AFC dengan ∠A

merupakan sudut persekutuan. Sebagai akibatnya AEAF

= ABAC

. Dengan cara yang

sama jika AD⊥BC, BEBD

= BCBA

dan CDCE

= CACB

sehingga BDDC

.CEEA

.AFFB

= ABBC

.BCCA

.CAAB

=

1 Terbukti

Page 99: Contents · PDF filegai batu loncatan untuk membuktikan teorema-teorema lainnya. Sebuah akibat dari beberapa teorema atau lemma disebut dengan korolary. Sebuah pernyataan yang tidak

Chapter 4. Geometri 93

Berikut adalah definisi dari garis transfersal sisi dan transfersal sudut dari4ABC.

Definisi:

kA B

Cl

Diberikan 4ABC. Garis lurus k disebut sebagai transversal sisi jika k mem-

otong sisi-sisi atau perpanjangan sisi 4ABC. Garis lurus ` disebut sebagai

transversal sudut jika ` melalui titik sudut 4ABC.

Teorema:Manelaos

Jika sebuah garis tranversal sisi memotong BC, CA dan AB dari 4ABC pada

titik-titik D, E dan F , maka BDDC

.CEEA

.AFFB

= −1

Perhatikan gambar di bawah ini.

B

F

AP

E

C D

Karena4APF dan4BDF sebangun maka AFFB

= PABD

. Berikutnya karena4CDE

sebangun dengan 4APF maka DCAP

= CEEA

.

Karena PA = −AP maka berlaku BDDC

.CEEA

.AFFB

= BDDC

.DCAP

. PABD

= −1 Terbukti

Page 100: Contents · PDF filegai batu loncatan untuk membuktikan teorema-teorema lainnya. Sebuah akibat dari beberapa teorema atau lemma disebut dengan korolary. Sebuah pernyataan yang tidak

Chapter 4. Geometri 94

Teorema berikut merupakan kebalikan dari teorema Menelaus.

Teorema:

Jika titik-titik D,E dan F terletak pada sisi-sisi4ABC sedemikian hingga berlakuBDDC

CEEA

.AFFB

= −1, maka D, E dan F kolinear(segaris).

Misal perpotongan DE terhadap AB adalah F ′. Menurut Teorema Menelaus

diperoleh BDDC

CEEA

.AFFB

= −1 dari sini didapatkan AF ′F ′B = AF

FB, akibatnya F dan F ′

berimpit dan F terletak di DE. Jadi, terbukti D,E dan kolinear.

D

A

F

E

B C

Contoh:

Sebuah garis transversal memotong sisi-sisi AB, BC, CD dan DA dari segiem-

pat ABCD di P,Q, R, dan S berturut-turut.

Buktikan bahwa APPB

.BQQC

CRRD

.DSSA

= 1

Jawab:

P

A

BC

D S

Q

Rx

Menurut teorema Manelaus dari gambar di samping, pada 4ABC didapatkanAPPB

.BQQC

.CXXA

= 1....1

Page 101: Contents · PDF filegai batu loncatan untuk membuktikan teorema-teorema lainnya. Sebuah akibat dari beberapa teorema atau lemma disebut dengan korolary. Sebuah pernyataan yang tidak

Chapter 4. Geometri 95

Kemudian pada4ACD didapatkan CXXA

.ASSD

.DRRC

= −1, sehingga CXXA

= −DSSA

.CRRD

....2

Jika (2) disubstitusikan ke (1), akan didapatkan APPB

.BQQC

.

(− DS

SA.CRRD

)= −1

Ekivalen dengan APPB

.BQQC

.DSSA

.CRRD

= 1. Terbukti.

Teorema Phytagoras merupakan bentuk khusus dari aturan cosinus dengan sudut

α = 900.

Teorema:Phytagoras

C

a

B

c

b A

Pada sebuah segitiga siku-siku, maka kuadrat hypotenuse sama dengan jumlah

kuadrat dua sisi yang lain.

Bukti:

Teorema di atas sama dengan mengatakan bahwa kuadarat panjang sisi terpan-

jang sama dengan jumlah kuadrat sisi yang lainnya.

Untuk segitiga ABC siku-siku dengan siku-siku di C. Misalnya panjang AB

adalah c, panjang BC adalah a dan panjang AC adalah b. Akan ditunjukkan

bahwa c2n = b2 + a2.

Dibentuk persegi dengan panjang sisi a + b seperti pada gambar berikut

Dari gambar di atas, jelas bahwa luas ABCD = 4x luas 4APS+ luas PQRS.

Sehingga didapatkan

(a + b)2 =

(4x1

2(axb)

)+ c2

a2 + 2ab + b2 = 2ab + c2

a2 + b2 = c2. Terbukti.

Contoh:

Page 102: Contents · PDF filegai batu loncatan untuk membuktikan teorema-teorema lainnya. Sebuah akibat dari beberapa teorema atau lemma disebut dengan korolary. Sebuah pernyataan yang tidak

Chapter 4. Geometri 96

a

a

b

S

ba PA

R abD

c

c

c

c

B

C

b

Q

Buktikan teorema Apollonius berikut.Jika D adalah titik tengah BC pada4ABC,

maka berlaku AB2 + AC2 = 2AD2 + 2DC2!

Jawab:

Misalkan ∠BDA = α0 maka ∠CDA = (180 − α)0. dari segitiga ABD, didap-

atkan aturan cosinus sebagai berikut:

AB2 = BD2 + AD2 − 2BD.AD. cos α .....1

Kemudian dari segitiga ACD, didapatkan aturan cosinus sebagai berikut:

AC2 = DC2 + AD2 − 2DC.AD. cos(180− α)0

AC2 = DC2 + AD2 − 2DC.AD. cos α0 .....2

Jika persamaan 1 dan 2 dijumlahkan maka didapatkan

AB2 + AC2 = DC2 + BD2 + 2AD2 + DC2

Karena D merupakan titik tengah BD maka panjang BD sama dengan panjang

DC sehingga diperoleh

AB2 + AC2 = 2AD2 + 2DC2 Terbukti.

5.3 Lingkaran

Definisi:

Lingkaran adalah himpunan semua titik yang mempunyai jarak r terhadap titik O. r

kemudian disebut degan jari-jari dan O disebut dengan titik pusat.

Page 103: Contents · PDF filegai batu loncatan untuk membuktikan teorema-teorema lainnya. Sebuah akibat dari beberapa teorema atau lemma disebut dengan korolary. Sebuah pernyataan yang tidak

Chapter 4. Geometri 97

Beberapa sifat penting dalam lingkaran adalah:

1. Sudut keliling yang menghadap busur yang sama maka besarnya sama

pula.

α

α 2α

α

2. Sudut pusat yang menghadap busur yang sama dengan sudut keliling,

besarnya dua kali sudut keliling.

3. Sudut-sudut yang berhadapan dari segiempat tali busur saling berpelurus.

a

O

A C B

E

D

F

180− a

4. Jika garis AB menyinggung lingkaran di titik C, maka untuk setiap tali

busur CD melalui titik C diperoleh ∠ACD = ∠CED dan ∠BCD = ∠CFD

Page 104: Contents · PDF filegai batu loncatan untuk membuktikan teorema-teorema lainnya. Sebuah akibat dari beberapa teorema atau lemma disebut dengan korolary. Sebuah pernyataan yang tidak

Chapter 4. Geometri 98

O

AC

B

400S

Contoh:

Diketahui segitiga ABC dengan sisi-sisi AB,CAdanCB masing-masing meny-

inggung lingkaran yang pusatnya O. Jika ∠ACB = 400, tentukan ∠AOB!

Jawab:

Misalkan R, S dan T adalah titik singgung lingkaran. Karena ∠ACB = 400,

maka ∠ACB + ∠CAB = 1400. Akibatnya ∠TAS = 1800 − ∠CAB dan ∠RBS =

1800 − ∠CBA. Dari sini didapatkan ∠TAS − ∠RBS = 3600 − 1400 = 2200

Teorema: Talibusur

P

CA

DB

Jika AC dan BD adalah taibusur-talibusur sebuah lingkaran yang berpotongan

di dalam lingkaran dititik P , maka PA.PC = PB.PD.

Bukti:

Perhatikan dua buah segitiga4APBdan4CPD. Karena ketiga sudut yang bers-

esuaian sama besarnya yaitu maka dua segitiga tersebut adalah sebangun. Aki-

batnya diperoleh PAPD

= PBPC

. Terbukti bahwa PA.PC = PB.PD

Page 105: Contents · PDF filegai batu loncatan untuk membuktikan teorema-teorema lainnya. Sebuah akibat dari beberapa teorema atau lemma disebut dengan korolary. Sebuah pernyataan yang tidak

Chapter 4. Geometri 99

B

A

A′

B′

P

Contoh:

Diketahui AA′ dan BB′ adalah talibusur-talibusur sebuah lingkaran yang berpo-

tongan di titik P diluar lingkaran. Tunjukkan bahwa PA.PA′ = PB.PB′!

Jawab:

Perhatikan bahwa ∠A′PB dan ∠B′PA adalah sudut yang menghadap busur

yang sama, akibatnya ∠A′PB = ∠B′PA. Demikian pula halnya dengan ∠A′PB =

∠B′PA. Demikian pula halnya dengan ∠A′PB = ∠B′PA. Dari sini diperoleh

dua buah segitiga yang kongruen yaitu 4PA′B dan 4PB′A, sehingga diper-

oleh PA′PB′ = PB

PA′ , ekivalen dengan PA.PA′ = PB.PB′ Terbukti

Persamaan tersebut sering pula disebut Teorema Secant.

Teorema: Secant-Tangent

Jika P adalah sebuah titik diluar lingkaran dan garis singgung dari P menying-

gung lingkaran di titik T dan 2 garis melalui memotong A di A′ dan memotong

B di B′ maka

PA.PA′ = PB.PB′ = PT 2

Bukti:

Misal ∠PA′T = α. Karena AOT menghadap busur yang sama maka ∠AOT =

2α. Perhatikan 4AOT adalah segitiga sama sisi, sehingga sudut kakinya mem-

punyai besar yang sama. Dari sini didapatkan ∠OTA = 900 − α. Karena

Page 106: Contents · PDF filegai batu loncatan untuk membuktikan teorema-teorema lainnya. Sebuah akibat dari beberapa teorema atau lemma disebut dengan korolary. Sebuah pernyataan yang tidak

Chapter 4. Geometri 100

α

A O

P

A′

T

PT adalah garis singgung, maka didaptkan ∠PTA = α. Bandingkan kedua

segitiga 4PA′T dan 4PTA. Karena mempunyai sudut-sudut yan besarnya

sama, 4PTA. Karena mempunyai sudut-sudut yang besarnya sama, maka ke-

dua segitiga ini adalah sebangun. Jadi diperoleh PTPA′ = PA

PT ′ . Ekivalen dengan

PA.PA′ = PT 2. Terbukti.

Contoh:

Jika panjang PT = 6cm, SQ = 2, 5cm. Berapkah panjang TQ

O

P

T

R Q

Jawab:

Karena ∠OSR = ∠OSQ maka didapatkan dua buah segitiga yang kongruen se-

hingga QR = 2SQ = 2x2, 5cm = 5cm. Menurut teorema Secant-Tangent diper-

Page 107: Contents · PDF filegai batu loncatan untuk membuktikan teorema-teorema lainnya. Sebuah akibat dari beberapa teorema atau lemma disebut dengan korolary. Sebuah pernyataan yang tidak

Chapter 4. Geometri 101

oleh

TQ.TR = TP 2

TQ.(TQ + 5) = 62

TQ2 + 5TQ− 36 = 0

(TQ + 9)(TQ− 4) = 0

Karena panjang tidak mungkin bernilai negatif maka nilai TQ yang tidak memenuhi

adalah 4.

SOAL-SOAL DAN PEMBAHASAN

Bagian 1

1. Tentukan perbandingan luas antara daerah yang diarsir dengan daerah

yang tidak diarsir pada gambar berikut!

(Soal kompetensi matematika XVII Jakarta)

Pembahasan:

Perhatikan gambar segitiga kecil. Sisi-sisi segitiga tersebut adalah 1, 1,√

2.

Dari sini diperoleh luas setiap segitiga kecil adalah 12.1.1 = 1

2satuan luas

dan luas sehingga daerah yang diarsir luasnya 8.12

= 4 satuan luas. Kemu-

dian luas daerah yang tidak diarsir adalah:

Page 108: Contents · PDF filegai batu loncatan untuk membuktikan teorema-teorema lainnya. Sebuah akibat dari beberapa teorema atau lemma disebut dengan korolary. Sebuah pernyataan yang tidak

Chapter 4. Geometri 102

(2 +√

2)2 - 4.12

= 4 + 4√

2 + 2− 2

= 4(1 +√

2)

2. Perhatikan gambar,segitiga ABC siku-siku di A dan D adalah pertengahan

BC. Titik F membagi dua sama panjang sisi AB, sedangkan titik E dan

G berturut-turut membagi AF dan FB menjadi dua bagian sama panjang.

Garis AD memotong garis hubung CE, CF dan CG berturut-turut dititik

P,Q dan R. Tentukan nilai perbandingan PQ : PR?

(soal kompetensi matematika XVII Jakarta)

R

C

A FE G B

PQ

D

Pembahasan:

Untuk mempermudah, AD diperpanjang sampai titik H sedemikian hingga

ABHC merupakan persegi. Misalkan panjang AH = x. Perhatikan bahwa

4AEP dan 4HCP adalah dua segitiga yang sebangun. Akibatnya diper-

oleh APHP

= AEHC

= 14. Dari sini didapatkan AP = 1

5AH . Jadi AP = 1

5x. ke-

mudian selanjutny perhatikan pula bahwa 4APQ dan 4HCQ sebangun,

demikian pula 4AGR dan 4HCR.Dengan cara yang sama, akan didap-

Page 109: Contents · PDF filegai batu loncatan untuk membuktikan teorema-teorema lainnya. Sebuah akibat dari beberapa teorema atau lemma disebut dengan korolary. Sebuah pernyataan yang tidak

Chapter 4. Geometri 103

atkan AQ = 13x dan AR = 3

7x. Karena

PQ = AQ− PQ

=1

3x− 1

5x

=2

15x

dan

PR = AR− AP

=3

7x− 1

5x

=8

35x

Didapatkan PQ : PR = 215

x : 835

x = 7 : 12

3. Keliling sebuah segitiga sama sisi adalah p. Misalkan Q sebuah titik di

dalam segitiga tersebut. Jika jumlah jarak dari Q ke ketiga sisi segitiga

adalah s. Nyatakan panjang p dalam s !

(soal olimpiade matematika SMA tk.Propinsi)

Pembahasan:

Perhatikan gambar berikut:

Y

CB

A

X

Q

Z

Page 110: Contents · PDF filegai batu loncatan untuk membuktikan teorema-teorema lainnya. Sebuah akibat dari beberapa teorema atau lemma disebut dengan korolary. Sebuah pernyataan yang tidak

Chapter 4. Geometri 104

Misalkan QX, QY dan QZ berturut-turut adalah jarak Q ke sisi BC,AC

dan AB. Perhatikan bahwa Luas 4ABC = Luas 4BQC + Luas 4AQC +

Luas 4AQB sehingga

1

2.AB.AC. sin 600 =

1

2.BC.QX +

1

2.AC.QY +

1

2.AB.QZ

1

2.1

3p.

1

2

√3 =

1

2.1

3p.QX +

1

2.1

3p.QY +

1

2.1

3p.QZ

Diperoleh, 13p.1

2

√3 = QX + QY + QZ

Sehingga p = 6√3s = 2s

√3

4. Segitiga AOB adalah segitiga samakaki yang siku-siku diO dengan pan-

jang AB sedemikian hingga pajang AP dan panjang AO sama. Tentukan

jari-jari lingkaran yang memenuhi semua syarat berikut:

(a) berpusat di ruas garis AB

(b) melalui titik P dan

(c) menyinggung ruas garis AO

(olimpiade matematika jawa timur)

Pembahasan:

Perhatikan gambar

K

A

O B

P

L

K

Page 111: Contents · PDF filegai batu loncatan untuk membuktikan teorema-teorema lainnya. Sebuah akibat dari beberapa teorema atau lemma disebut dengan korolary. Sebuah pernyataan yang tidak

Chapter 4. Geometri 105

Misalkan jari-jari r. Maka AL = LP = r√

2. Karena AB = m, maka

AO = AP = m√

22

. Padahal AP = AL+LP . Oleh karena itu m√

22

= r√

2+r.

Dari sini diperoleh r = m(2−√2)2

5. Tunjukkkan bahwa sudut antara garis tinggi D dan diameter lingkaran

luar 4ABC yang dibuat dari A yaitu AL, besarnya sama dengan seliih

sudut-sudut pada titik-titik sudut B dan C dan terbagi dua sama besar

oleh garis bagi ∠A !

Pembahasan:

Perhatikan gambar

D

MO

A

L

CB X

Karena ∠AOC = 2B, kita dapatkan ∠CAO = 900 − B dan juga ∠DAB =

900 − B,sehingga ∠CO = ∠DAB. Karena garis bagi AX maka ∠XAB =

∠XAC. Sehingga kita dapatkan ∠XAD = ∠XAO atau dengan kata lain

AX merupakan garis bagi ∠ADL.∠DAL = 2(∠XAB − ∠DAX) = B − C

6. Dinotasikan O, I, R dan r berturut-turut sebagai pusat lingkaran luar dan

dalam, jari-jari lingkaran luar dan dalam. Tunjukkan bahwa berlaku OI2 =

R2 − 2Rr!

Pembahasan:

Perhatikan gambar

Karena ∠AMC = ∠MAC = 12A, diperoleh ∠MBI = 1

2(A + B) = ∠IBA +

∠BAI yang berakibat BM = BI . Misalkan garis bagi dalam dan luar

Page 112: Contents · PDF filegai batu loncatan untuk membuktikan teorema-teorema lainnya. Sebuah akibat dari beberapa teorema atau lemma disebut dengan korolary. Sebuah pernyataan yang tidak

Chapter 4. Geometri 106

L

MB C

AN

Y

X

OI

∠A memotong lingkaran luar ABC di M da N , maka ∠MAN = 900, se-

bab MN adalah diameter. Dibentuk garis XY yang merupakan diameter

yang memuat O dan I , sehingga XI.IY = AI.IM sedangkan AI.IM =

(R+OI)(R−OI) = R2− IO2. Perhatikan bahwa IL⊥AB, karena ∠LAI =

∠BNM dan ∠ALI = ∠MBN , maka 4ALI sebangun dengan 4NBM .

Akibatnya berlaku ALIL

= MNMB

dan selanjutnya karena BM = IM , didap-

atkan:

AI.IM = MN.IL = Rr

AI.IM = R2 −OI2

2Rr = R2 −OI2

OI2 = R2 −Rr

Persamaan ini disebut persamaan Euler

7. Diberikan sebarang segitiga sama kaki ABC dengan AB = AC. Jika r

adalah jari-jari lingkaran luarnya dan p adalah jari-jari lingkaran dalam-

nya, tunjukkan bahwa jika d merupakan jarak kedua titik pusat kedua

lingkaran itu maka d =√

r(r − 2p)!

Pembahasan:

Perhatikan gambar

Page 113: Contents · PDF filegai batu loncatan untuk membuktikan teorema-teorema lainnya. Sebuah akibat dari beberapa teorema atau lemma disebut dengan korolary. Sebuah pernyataan yang tidak

Chapter 4. Geometri 107

M

B C

I

OL

A

Menurut persamaan euler maka BM = IM = r − d, sedangkan BD = 12a.

Luas 4ABI adalah:

1

2AB.IL =

1

2AI.BD

c.p = (r + d)1

2a

2cp = a(r + d)

Sedangkan luas 4ABM adalah

1

2AB.BM =

1

2AM.BD

c(r − d) = 2r

(1

2a

)

c(r − d) = 2r

c(r − d) =2cpr

r + d

2pr = (r − d)(r + d) = r2 − d2

d2 = r62− 2prr(r − 2p)

d2 =√

r(r − 2p)

Terbukti

Page 114: Contents · PDF filegai batu loncatan untuk membuktikan teorema-teorema lainnya. Sebuah akibat dari beberapa teorema atau lemma disebut dengan korolary. Sebuah pernyataan yang tidak

Chapter 4. Geometri 108

8. Buktikan torema Ptolemeus berikut: ’pada setiapsegiempat talibusur se-

lalu brlaku perkalian diagonal-diagonalnya sama denga jumlah perkalian

sisi-sisi yang berhadapan.’

Pembahasan:

Perhatikan gambar

E

B

A

D

C

Pada gambar dibentuk garis AE sedemikian hingga berlaku ∠BAE =

∠CAD.4BAE sebangun dengan 4CAD karena ∠BAE = ∠CAD dan

∠ABE = ∠ACD. Sehingga ABAC

= BECD

atau AC.BE = AB.CD. Analog

dengan cara diatas 4ADE sebangun dengan 4ACB sehingga ADAC

= DEBC

atau AC.DE = AD.BC. Jika kedua persamaan ini dijumlahkan akan di-

dapat

AC.BE + AC.DE = AB.CD + AD.BC

AC(BE + DE) = AB.CD + AD.BC

AC.BD = AB.CD + AD.BC

Terbukti

9. Jika ABCD adalah segiempat yang bukan segiempat tali busur, tunjukkan

bahwa AB.CD + AD.BC > AC.BD!

Page 115: Contents · PDF filegai batu loncatan untuk membuktikan teorema-teorema lainnya. Sebuah akibat dari beberapa teorema atau lemma disebut dengan korolary. Sebuah pernyataan yang tidak

Chapter 4. Geometri 109

Pembahasan:

Perhatikan gambar

E

B

A

D

C

Karena ABCD bukan segiempat tali busur maka ∠ABD 6= ∠ACD se-

hingga dapat kita tentukan titik E di luar BD sedemikian hingga ∠ABE =

∠ACD dan ∠BAE = ∠CAD. Akibatnya 4BAE sebangun denga 4CAD.ABAC

= BECD

atau AB.CD = AC.BE. Analog dengan cara diatas didapatkan

4ABC sebangun dengan 4AED. BCAC

= EDAD

atau BC.AD = AC.ED. Jika

kedua persamaan tersebut dijumlahkan maka didapat:

AB.CD + BC.AD = AC.BE + AC.ED

AB.CD + BC.AD = AC(BE + DE)

AB.CD + BC.AD > AC.BD

Terbukti

10. Jika panjang sisi-sisi BC, CA dan AB pada segitiga ABC adalah a, b dan

c berturut-turut dan 2s = a + b + c, buktikan bahwa luas daerah segitiga

ABC adalah L =√

s(s− a)(s− b)(s− c) !

(soal kompetensi matematika SMU Bandung)

Page 116: Contents · PDF filegai batu loncatan untuk membuktikan teorema-teorema lainnya. Sebuah akibat dari beberapa teorema atau lemma disebut dengan korolary. Sebuah pernyataan yang tidak

Chapter 4. Geometri 110

Pembahasan:

Perhatikan pada 4ADC berlaku

t2 = b2 − x2......1

t2 = a2 − (c− x)2

t2 = a2 − c2 + 2cx− x2......2

Dari persamaan 1 dan 3 didapatkan

b2 − x2 = a2 − c2 + 2cx− x2

b2 + c2 − a2 = 2cx

x =b2 + c2 − a2

2c......3

Jika 3 disubsitusikan ke 1, maka t2 = b2 −(

b2+c2−a2

2c

)2

t2 =

(b−

(b2 + c2 − a2

2c

)2)(b +

(b2 + c2 − a2

2c

)2)

=2bc− b2 − c2 + a2

2c.2bc + b2 + c2 − a2

2c

=a2 − (b− c)2

2c.(b + c)2 − a2

2c

=(a− b + c)(a + b− c)(b + c− a)(b + c− a)

4c2

=(2s− 2b)(2s− 2c)(2s− 2a)(2s)

4c2

Sehingga t = 2c

√s(s− a)(s− b)(s− c)

Luas segitiga ABC adalah

L =1

2.c.t =

1

2.c.

2

c

√s(s− a)(s− b)(s− c)

Page 117: Contents · PDF filegai batu loncatan untuk membuktikan teorema-teorema lainnya. Sebuah akibat dari beberapa teorema atau lemma disebut dengan korolary. Sebuah pernyataan yang tidak

Chapter 4. Geometri 111

Jadi, L =√

s(s− a)(s− b)(s− c) Terbukti

Bagian 2

1. Jika dalam4ABC, AD adalah garis tinggi dan AE adalah diameter lingkaran

luar yang melalui A maka AB.AC = AD.AE

Pembahasan:

B CDO

A

E

Karena ∠ABC = ∠ABD dan ∠ABD = ∠ACE = 900, sehingga 4ABD

sebangun dengan4AEC maka ABAE

= ADAC

atau AB.AC = AD.AE. Terbukti

Pernyataan ini disebut Brahmagupta.

2. Perhatikan gambar

B

C

A

Tiga lingkaran berpusat di A,B dan C. Lingkaran berpusat di A mempun-

yai jari-jari 3. Lingkaran berpusat di B mempunyai jari-jari 5. Besar sudut

BAC adalah π3

radian. Besar sudut ABC adalah

Pembahasan:

Page 118: Contents · PDF filegai batu loncatan untuk membuktikan teorema-teorema lainnya. Sebuah akibat dari beberapa teorema atau lemma disebut dengan korolary. Sebuah pernyataan yang tidak

Chapter 4. Geometri 112

Misalkan jari-jari lingkaran berpusat di C = r. Dengan mengunakan atu-

ran cosinus didapatkan

(r + 5)2 = 82 + (r + 3)2 − 16(r + 3) cosπ

3

Maka diperoleh r = 2. Misalkan β adalah sudut ABC maka dengan meng-

gunkan aturan cosinus didapatkan pula 52 = 82 + 72− 2.8.7 cos β Dari sini

diperoleh cos β = 1114

sehingga β = arccos 1114

3. Perhatikan gambar. Jika sudut q = 14p. Tunjukkkan bahwa kedua daerah

yang diarsir mempunyai luas yang sama! (soal kompetensi Jakarta IV Jakarta)

Pembahasan:

Misalkan jari-jari lingkaran besar adalah R

L1 = LjuringBAE − L4BAE

=900

3600xπ

(1

2R

)2

− 1

2

(1

2R

)2(1

2R

)2

=1

16πR2 − 1

8R2......(1)

L1 = LjuringACD − LjuringBCE − L4BAE

=14π

2πxπR2 − 900

3600xπ

(1

2R

)2

− 1

2

(1

2R

)2(1

2R

)2

=1

8R2 − 1

16πR2 − 1

8R2

=1

16πR2 − 1

8R2......(2)

Dari (1) dan (2) didapatkan Terbukti.

4. Suatu persegi panjang berukuran 8x2√

2 mempunyai titik pusat yang sama

dengan suatu lingkaran berjari-jari 2. Berapakah luas daerah irisan antara

persegi panjang da lingkaran tersebut?

Page 119: Contents · PDF filegai batu loncatan untuk membuktikan teorema-teorema lainnya. Sebuah akibat dari beberapa teorema atau lemma disebut dengan korolary. Sebuah pernyataan yang tidak

Chapter 4. Geometri 113

Soal olimpiade matematika kabupaten

Pembahasan:

Dari gambar diperoleh bahwa cos =√

22

. Jadi θ = 450.

Luas tembereng = luasjuring − luas4PAB

=90

360xπx2x2− 1

2x2x2

= π − 2

Luas daerah irisan = luas lingkaran− luas tembereng

= πx2x2− 2(π − 2)

= 4π − 2π + 4

= 2π + 4

5. Buktikan bahwa jumlah luas bujur sangkar yang diluar sama dengan tiga

kali jumlah luas bujur sangkar yang di dalam!

(Soal matematika RIA Bogor)

Pembahasan:

Misalkan panjang sisi busur sangkar yang ada didalam adalah a, b, dan c,

maka jumlah luas bujur sangkar tersebut adalah a2 + b2 + c2. Keemudian

dimisalkan panjang sisi bujur sangkar yang ada di luar dalah x, y dan z,

maka jumlah luas bujur sangkar tersebut adalah x2 + y2 + z2. Akan ditun-

jukkan bahwa x2 + y2 + z2 = 3(a2 + b2 + c2). Perhatikan 4ABC. Menurut

Page 120: Contents · PDF filegai batu loncatan untuk membuktikan teorema-teorema lainnya. Sebuah akibat dari beberapa teorema atau lemma disebut dengan korolary. Sebuah pernyataan yang tidak

Chapter 4. Geometri 114

aturan cosinus didapatkan

cos α =b2 + c2 − a2

2bc

cos β =a2 + c2 − b2

2ac

cos γ =a2 + b2 − c2

2ab

Kemudian untuk setiap segitiga sama kaki diatas didapatkan

x2 = a2 + c2 − 2ac cos(1800 − β)

= a2 + c2 − 2ac cos β

= a2 + c2 − 2ac

(a2 + c2 − b2

2ac

)

= 2a2 + 2c2 − b

Dengan cara yang sama didapatkan pula

y2 = 2b2 + 2c2 − a2

z2 = 2a2 + 2b2 − c2

Dari ketiga persamaan ini diperoleh

x2 + y2 + z2 = 2a2 + 2c2 − b2 + 2b2 + 2c2 − a2 + 2a2 + 2b2 − c

x2 + y2 + z2 = 3a2 + 3b2 + 3c2

Terbukti

Page 121: Contents · PDF filegai batu loncatan untuk membuktikan teorema-teorema lainnya. Sebuah akibat dari beberapa teorema atau lemma disebut dengan korolary. Sebuah pernyataan yang tidak

Chapter 4. Geometri 115

6. Diberikan segitiga sama sisi ABC dan sebuah titik P sehingga jarak P ke

A dan ke C tidak lebih jauh dari jarak P ke B. Buktikan bahwa jika P ter-

letak pada lingkaran luar segitiga ABC maka PB = PA.PC!Soal seleksi

olimpiade matematika

Pembahasan:

O

B

1200

A

C

P

∠BPC =1

2x∠BOC =

1

2x1200 = 600

∠APB =1

2x∠AOB =

1

2x1200 = 600

AB2 = PA2 + PB2 − 2PA.PB cos 600

AB2 = PA2 + PB2 − PA.PB......(1)

BC2 = PC2 + PB2 − 2PC.PB cos 600

BC2 = PC2 + PB2 − 2PC.PB......(2)

Karena ABC segitiga samasisi maka AB = BC, sehingga didapatkan

PA2 + PB2 − PA.PB = PC2 + PB2 − PC.PB

PA2 − PC2 = PA.PB − PC.PB

(PA− PC)(PA + PC) = (PA− PC)PB

Terbukti bahwa PB = PA.PC

Page 122: Contents · PDF filegai batu loncatan untuk membuktikan teorema-teorema lainnya. Sebuah akibat dari beberapa teorema atau lemma disebut dengan korolary. Sebuah pernyataan yang tidak

Chapter 4. Geometri 116

7. Segitiga ABC siku-siku di C. Garis berat CM tegak lurus garis berat BN .

Panjang sisi BC = s. Tentukan panjang BN !

Pembahasan:

Misalkan AC = b dan AB = c. Karena BN dan CP garis berat, maka

BN : PN = 1 : 2 dan CM : PM = 2 : 1.

Perhatikan 4BNC

B A

N

C

M

BN2 = s2 + CN2

BN2 = s2 +1

4b2.......(1)

Perhatikan 4BPM

BP 2 = BM2 + PM2

(2

3BN

)2

=

(1

2c

)2

+

(1

2CP

)2

4

9BN2 =

1

4c2 +

1

4CP 2

4

9BN2 =

1

4c2 +

1

4(s2 − CBP 2)

4

9BN2 =

1

4c2 +

1

4s2 − 1

9BN2

5

9BN2 =

1

4c2 +

1

4s2.......(2)

Page 123: Contents · PDF filegai batu loncatan untuk membuktikan teorema-teorema lainnya. Sebuah akibat dari beberapa teorema atau lemma disebut dengan korolary. Sebuah pernyataan yang tidak

Chapter 4. Geometri 117

Jika (2) - (1) didapatkan

−4

9BN2 =

1

4(c2 − b2)− 3

4s2

−4

9BN2 =

1

4s2 − 3

4s2

−4

9BN2 =

2

4s2

BN2 =18

16s2

Jadi panjang BN adalah BN = 34s√

2

8. Misalkan sisi BC dari segitiga siku-siku ABC adalah garis tengah sebuah

lingkaran yang memoton sisi miring AB di D. Garis singgung lingkaran

di D memotong sisi CA di F . Buktikan bahwa besar sudut CDF adalah

dua kali besar sudut A. Soal seleksi olimpiade Matematika

Pembahasan:

Perhatikan 4OBD pada gambar dibawah.

C

O

AF

D

B

Page 124: Contents · PDF filegai batu loncatan untuk membuktikan teorema-teorema lainnya. Sebuah akibat dari beberapa teorema atau lemma disebut dengan korolary. Sebuah pernyataan yang tidak

Chapter 4. Geometri 118

∠OBD = ∠OBD

∠OBD = ∠B

∠DOB = 1800 − 2∠B

∠COD = 2∠B

∠CFD = 1800 − 2∠B

= 2(900 − ∠B)

= 2∠A

Terbukti

9. Segitiga ABC siku-siku di C. Titik P dan Q terletak pada AB sedemikian

hingga AB terbagi menjadi tiga bagian yang sama. Buktikan bahwa CP 2+

PQ2 + QC2 = 23AB2

Soal kompetensi matematika V Jakarta

Pembahasan: Perhatikan 4CAP .

C

AB

D

P

Page 125: Contents · PDF filegai batu loncatan untuk membuktikan teorema-teorema lainnya. Sebuah akibat dari beberapa teorema atau lemma disebut dengan korolary. Sebuah pernyataan yang tidak

Chapter 4. Geometri 119

Menurut aturan cosinus didapatkan

CP 2 = CA2 + PA2 − 2CA.AP cos A

CP 2 = CA2 +

(1

2AB

)2

− 2CA.1

3AB

CA

AB

CP 2 =1

3CA2 +

1

9AB2.......(1)

Perhatikan 4CAQ

QC2 = CA2 + AQ2 − 2CA.AQ cos A

QC2 = CA2 +

(2

3AB

)2

− 2CA.2

3AB

CA

AB

QC2 = −1

3CA2 +

4

9AB2.......(1)

Jadi,

CP 2 + PQ2 + QC2 =1

3CA2 +

1

9AB2 +

1

9AB2 − 1

3CA2 +

4

9AB2

=2

3AB2

Terbukti

10. Sebuah titik A terletak disebelah luar lingkaran yang berpusat di titik M .

Dari titik A ditarik garis yang memotong lingkaran dititik B da C ( titik

B terletak diantara C). Dibuat garis CM sehingga memotong lingkaran

di titik D, ternyata AD menyinggung lingkaran dan titik E terletak pada

garis KD. Jika panjang AE = 1cm, XB = 2 cm dan BC = 6 cm, Buk-

tikan bahwa AM,DB dan CE berpotongan di sebuah titik!Soal kompe-

Page 126: Contents · PDF filegai batu loncatan untuk membuktikan teorema-teorema lainnya. Sebuah akibat dari beberapa teorema atau lemma disebut dengan korolary. Sebuah pernyataan yang tidak

Chapter 4. Geometri 120

tensi matematika XII jakarta

Pembahasan:

C

M

D

1A

2 B6

R

R

E

Berdasarkan teorema Secant-Tangent didapatkan

AD2 = AB.AC = 2.(2 + 6)

AD2 = 16makaAD = 4

ED = 4− 1 = 3

ABBC

.CMMD

.DEEA

= 26.RR.31

= 1, Berarti AM,DB, dan CE berpotongan pada satu

titik

11. Panjang sisi AB, BC,CD, DA dari persegi panjang ABCD yang panjang

sisinya a dan b. Dipilih titik E, F, G dan H sedemikian hingga. Tentukan

luas daerah segiempat yang dibatasi oleh garis AG,BH, CE dan DF .

Pembahasan:

Karena tan α. tan β = a23a

=23a

a= 1, maka BH ⊥ EC. Berarti segiempat

yang terarsir berupa persegi. Misalkan panjang sisinya x maka berlaku

x

AE=

BC

CE⇔ x

13a

=a

13a√

13

sehingga diperoleh x = a√13

. Jadi luas daerah terarsir x2 = a2√13

Page 127: Contents · PDF filegai batu loncatan untuk membuktikan teorema-teorema lainnya. Sebuah akibat dari beberapa teorema atau lemma disebut dengan korolary. Sebuah pernyataan yang tidak

Chapter 4. Geometri 121

BA P

12. Dari gambar diketahui luas yang diarsir adalah A. Tunjukkan bahwa luas

persegi panjang juga A.

Pembahasan:

2p + 2q = A (5.1)

x + p =1

2luas lingkaran (5.2)

x + p =1

(1

2b

)2

(5.3)

2x + 2p =1

4πb2 (5.4)

(5.5)

y + q =1

2luas lingkaran (5.6)

y + q =1

(1

2a

)2

(5.7)

2y + 2q =1

4πa2 (5.8)

(5.9)

Page 128: Contents · PDF filegai batu loncatan untuk membuktikan teorema-teorema lainnya. Sebuah akibat dari beberapa teorema atau lemma disebut dengan korolary. Sebuah pernyataan yang tidak

Chapter 4. Geometri 122

Jika (1) dan (2)dijumlahkan didaptkan

2x + 2p + 2y + 2q =1

4πb2 +

1

4πa2

2x + 2y + A =1

4π(b2 + a2)

2x + 2y =1

4π(b2 + a2)− A

Luas lingkaran = luas persegi + 2x + 2y14π(b2 + a2) = luas persegi + 1

4π(b2 + a2) − A, terbukti bahwa luas persegi

adalah A.

13. Sebuah lingkaran memotong sisi-sisi pada bagian dalam yaitu DC di D

dan D′, CA di E da E ′, AB di F dan F ′. Jika AD, BE, dan CF kongruen,

tunjukkan bahwa AD′, BE ′ dan CF ′ juga kongruen.

Pembahasan: Menurut teorema Secant diperoleh:

AF.AF ′ = AE.AE ′ ⇒ AF

AE=

AE ′

AF ′

BD.BD′ = BF.BF ′ ⇒ BD

BF=

BF ′

BD′

CE.CE ′ = CD.CD′ ⇒ CE

CD=

CD′

BD′

Page 129: Contents · PDF filegai batu loncatan untuk membuktikan teorema-teorema lainnya. Sebuah akibat dari beberapa teorema atau lemma disebut dengan korolary. Sebuah pernyataan yang tidak

Chapter 4. Geometri 123

karena AD,BE dan DF kongruen maka

AF

FB.BD

DC.CE

EA= 1

AF

EA.BD

FB.CE

DC= 1

AE ′

AF ′ .BF ′

BD′ .CD′

CE ′ = 1

AF ′

AE ′ .BD′

BF ′ .CE ′

CD′ = 1

AF ′

F ′B.AF ′

F ′B.CE ′

E ′A= 1

Terbukti

14. Misalkan ABC segitiga dan P suatu titik di dalam segitiga tersebut. Garis

AP,BP dan CP memotong garis BC,CA dan AB berturut-turut dititik

D,E, dan F . Garis DE, EF , dan FD berturut-turut memotong garis AB, BC

dan CA di titik titik K, M dan N . Tunjukkan bahwa K,M, dan N terletak

dalamsatu garis.

Pembahasan: Karena garis-garis AD, BE, dan CF berpotongan disatu

titik P , maka menurut teorema Ceva diperoleh

AF

FB.BD

DC.CE

EA= 1

Karena titik-titik D, E, dan K terletak pada satu garis, dengan Teorema

Manelaos diperoleh AKKB

.BDDC

.CEEA

= 1. Ekuivalen dengan AKKB

= DCBD

.EACE

. Den-

gan cara yang sama akan diperoleh juga BMMC

= EACE

.FBAF

dan CNNA

= FBAF

.DCBD

.

Dari persamaan (1),(2) dan (3) didapatkan

AK

KB.BM

MC.CN

NA=

(FB

AF.DC

BD

EA

CE

)2

= 1

Sehingga menurut Teorema Menelaos terbukti bahwa K,M , dan N ter-

letak pada satu garis.

Page 130: Contents · PDF filegai batu loncatan untuk membuktikan teorema-teorema lainnya. Sebuah akibat dari beberapa teorema atau lemma disebut dengan korolary. Sebuah pernyataan yang tidak

Chapter 4. Geometri 124

15. Lingkaran dalam segitiga ABC menyingguan sisi sisi BC,CA, dan AB

berturut-turut di titik titik D, E, dan F . Misalkan M titik tengah EF dan

garis AD memotong lingkaran dalam segitiga ABC di titik K, buktikan

bahwa ∠KME = ∠KMD = ∠DME

Pembahasan:Misalkan I adalah titik pusat lingkaran dalam segitiga ABC.

Tampa mengurangi sifat keumuman, misalkan K dan F terletak pada sisi

yang sama terhadap garis AM . Maka berlaku ∠KME = ∠KMA + 900

dan ∠DME = ∠DMI + 900. Akan dibuktikan bahwa ∠KMA = ∠DMI .

karena segitiga AMF dan AFI sebangun maka berlaku AM.AI = AF 2.

Dengan teorema titik kuasa pada A didapatkan AK.AD = AF 2. Dari sini

didapatkanAK.AD = AM.AI yang artinya KMID adalah suatu segiem-

pat tali busur. Karena ID = IK, maka sudut keliling yang meghadap

busur-busur tersebut adalah sama besar yaitu ∠DMI = ∠IDK. Dengan

sifat segiempat talibusur didapatkan ∠KMA = ∠IDK. Dengan demikian

didapatkan ∠KME = ∠KMA + 900 = ∠DMI + 900 = ∠DME.

Page 131: Contents · PDF filegai batu loncatan untuk membuktikan teorema-teorema lainnya. Sebuah akibat dari beberapa teorema atau lemma disebut dengan korolary. Sebuah pernyataan yang tidak

¥¥¥¥¥¥ CHAPTER 6

Kombinatorika

Konsep-konsep penting yang perlu dipahami dalam menyelesaikan masalah

dalam kombinatorika diantaranya adalah konsep permutasi dan kombinasi, prin-

sip inklusi-ekslusi, koefisien binomial, prinsip sarang merpati (pigeon hole prin-

ciple), paritas dan relasi rekurensi.

6.1 Permutasi dan Kombinasi

Definition 6.1.1 Permutasi. Banyaknya cara menyusun r anggota dari sebuah him-punan yang mempunyai n anggota dengan memperhatikan urutan adalah P (n, r) =

P nr = n!

(n−r)!, dimana n! = n · (n − 1) . . . 3 · 2 · 1. Apabila sebuah objek dapat dip-

ilih lebih dari satu kali maka disebut permutasi dengan pengulangan dengan rumusP (n, r) = P n

r = nr.

Definition 6.1.2 Kombinasi. Banyaknya cara menyusun r anggota dari sebuah him-punan yang mempunyai n anggota tanpa memperhatikan urutan adalah C(n, r) =

Cnr = n!

r!(n−r)!. Apabila sebuah objek dapat dipilih lebih dari satu kali maka disebut

kombinasi dengan pengulangan dimana C(n + r − 1, r) = Cn+r−1r = (n+r−1)!

r!(n−1)!.

Example. Tentukan banyaknya susunan dua angka dari lima angka 1,2,3,4,5.

Solution. Susunan selain diagonal utama adalah suatu permutasi, dan bila ter-

masuk diagonal utama disebut permutasi dengan pengulangan. Sedangkan

susunan kombinasi adalah hanya di atas diagonal utama atau hanya di bawah

diagonal utama.

125

Page 132: Contents · PDF filegai batu loncatan untuk membuktikan teorema-teorema lainnya. Sebuah akibat dari beberapa teorema atau lemma disebut dengan korolary. Sebuah pernyataan yang tidak

Chapter 9. Kombinatorika 126

1 2 3 4 5

1︷︸︸︷11︸︷︷︸ 12 13 14 15

2 21︷︸︸︷22︸︷︷︸ 23 24 25

3 31 32︷︸︸︷33︸︷︷︸ 34 35

4 41 42 43︷︸︸︷44︸︷︷︸ 45

5 51 52 53 54︷︸︸︷55︸︷︷︸

6.2 Prinsip Inklusi-Ekslusi dan Peluang

Prinsip ini digunakan untuk menentukan kardinalitas dari gabungan himpunan-

himpunan yang tidak harus saling lepas.

Theorem 6.2.1 Misal Ai adalah sebarang himpunan, 1 ≤ i ≤ n. Kardinalitas darigabungan himpunan-himpunan yang tidak harus saling lepas adalah:

∣∣n⋃

i=1

Ai

∣∣ =∑i=1

|Ai| −∑i<j

|Ai ∩ Aj|+∑

i<j<k

|Ai ∩ Aj ∩ Ak| − . . . (−1)n+1∣∣

n⋂i=1

Ai

∣∣

Definition 6.2.1 Jika S = {k1, k2, . . . , kn} adalah himpunan dari semua kejadian dalamsuatu ruang sampel, sedangkan K adalah kejadian yang terjadi pada himpunan K ⊆ S

maka peluang terjadinya K adalah p(K) = n(K)n(S)

. Dalam hal ini pi ≥ 0 dengani = 1, 2, . . . , n dan p1 + p2 + · · ·+ pn = 1.

Definition 6.2.2 Misalkan S adalah ruang sampel berhingga dengan kejadian A danB, maka probabilitas bersyarat dari kejadian A dengan syarat B, ditulis p(A/B) adalah:p(A/B) = p(A/B)

p(B).

Example. Pada sebuah klub olahraga diketahui bahwa 10 orang menyukai ten-

nis, 15 orang menyukai squash, 12 orang menyukai badminton, 5 orang menyukai

Page 133: Contents · PDF filegai batu loncatan untuk membuktikan teorema-teorema lainnya. Sebuah akibat dari beberapa teorema atau lemma disebut dengan korolary. Sebuah pernyataan yang tidak

Chapter 9. Kombinatorika 127

tennis dan squash, 4 orang menyukai tennis dan badminton, 3 orang menyukai

squash dan badminton dan 2 orang menyukai ketiga olahraga. Berapa banyak

anggota klub yang menyukai sedikitnya satu dari ketiga cabang olahraga ini?

Solution. Misalkan T, S dan B masing-masing adalah himpunan anggota klub

yang menyukai tennis, squash dan badminton. Maka menurut teorema Inklusi-

Ekslusi

|T ∪ S ∪B| = |T |+ |S|+ |B||T ∩ S| − |T ∩B| − |S ∩B|+ |T ∩ S ∩B|= 10 + 15 + 12 + 5− 43 + 2

= 12

Jadi, banyaknya anggota klub yang menyukai sedikitnya satu dari ketiga ca-

bang olahraga ini adalah 27 orang.

Example. Sepasang dadu dilempar satu kali. Misal A adalah kejadian muncul

mata 2 pada paling sedikit satu dadu, dan B kejadian jumlah mata yang muncul

pada kedua dadu sama dengan 6. Tentukan peluang kejadian A dengan syarat

B.

Solution. S = {11, 12, . . . , 66} sehingga n(S) = 36. A = {21, 22, 23, . . . , 62}, B =

{15, 24, 33, . . . , 51} dan n(A) = 11, n(B) = 5. Dengan demikian A ∩ B = {24, 42}dan n(A ∩B) = 2 sehingga p(A/B) = p(A/B)

p(B)= 2/36

5/36= 2

5.

6.3 Koefisien Binomial

Prinsip ini digunakan untuk menentukan kardinalitas dari gabungan himpunan-

himpunan yang tidak harus saling lepas.

Page 134: Contents · PDF filegai batu loncatan untuk membuktikan teorema-teorema lainnya. Sebuah akibat dari beberapa teorema atau lemma disebut dengan korolary. Sebuah pernyataan yang tidak

Chapter 9. Kombinatorika 128

Theorem 6.3.1 Jika x dan y adalah variabel dan n adalah bilangan asli maka berlaku:

(x + y)n =n∑

r=0

(n

r

)xn−ryr

Bila (x + y)n diekspansikan maka berlaku beberapa sifat berikut:

• Terdapat (n + 1) suku, jumlah pangkat dari x dan y pada tiap suku adalah

n.

• Pangkat dari x turun satu demi satu dari n ke 0, sedangkan pangkat dari y

naik satu demi satu dari 0 ke n.

• Koefisien dari suku-suku yang berjarak sama dari xn dan yn adalah sama.

Beberapa contoh dapat dilihat dalam berikut ini.

(x + y)0 = 1

(x + y)1 = x + y

(x + y)2 = x2 + 2xy + y2

(x + y)3 = x3 + 3x2y + 3xy2 + y3

(x + y)4 = x4 + 4x3y + 6x2y2 + 4xy3 + y4

(x + y)5 = x5 + 5x4y + 10x3y2 + 10x2y3 + 5xy4 + y5

(x + y)6 = x6 + 6x5y + 15x4y2 + 20x3y3 + 15x2y4 + 6xy5 + y6

Kalau koefisien-koefisiennya disusun dalam bentuk segitiga, maka susunan se-

gitiga tersebut dinamakan ’Segitiga Pascal’. Segitiga ini mempunyai sifat: (i) Bi-

langan pertama dan bilangan terakhir dalam tiap baris adalah 1, (ii) Tiap bilan-

gan selain 1, dapat dicari dengan menjumlahkan dua bilangan tepat di atasnya.

1

Page 135: Contents · PDF filegai batu loncatan untuk membuktikan teorema-teorema lainnya. Sebuah akibat dari beberapa teorema atau lemma disebut dengan korolary. Sebuah pernyataan yang tidak

Chapter 9. Kombinatorika 129

1 + 1

1 + 2 + 1

1 + 3 + 3 + 1

1 + 4 + 6 + 4 + 1

1 + 5 + 10 + 10 + 5 + 1

1 + 6 + 15 + 20 + 15 + 6 + 1

Example. Hitunglah berapa nilai dari∑1003

k=0

(20072k+1

).

Solution. Perhatikan bahwa binomial untuk x = 1 dan y = 1 adalah

(1 + 1)2007 =2007∑

k=0

(2007

k

)(1)2007−k(1)k = 22007

2007∑

k=0

(2007

k

)= 22007 (6.1)

(1 + (−1))2007 =2007∑

k=0

(2007

k

)(1)2007−k(−1)k = 02007 = 0

2007∑

k=0

(2007

k

)(−1)k = 0

k genap

(2007

k

)(−1)k +

k ganjil

(2007

k

)(−1)k = 0

k genap

(2007

k

)−

k ganjil

(2007

k

)= 0

k genap

(2007

k

)=

k ganjil

(2007

k

)

2007∑

k=0

(2007

2k

)=

2007∑

k=0

(2007

2k + 1

)(6.2)

Page 136: Contents · PDF filegai batu loncatan untuk membuktikan teorema-teorema lainnya. Sebuah akibat dari beberapa teorema atau lemma disebut dengan korolary. Sebuah pernyataan yang tidak

Chapter 9. Kombinatorika 130

Karena menurut (6.2)

2007∑

k=0

(2007

k

)= 2

2007∑

k=0

(2007

2k

)= 2

2007∑

k=0

(2007

2k + 1

)

Maka sesuai (6.1) diperoleh

2007∑

k=0

(2007

2k + 1

)=

1

2

2007∑

k=0

(2007

k

)=

1

2· 22007 = 22006

Example. Jika n bilangan asli, buktikan bahwa∑n

r=1

(nr

)r = n2n−1.

Solution. Menurut koefisien binomial didapat (x+1)n =∑n

r=0

(nr

)xr. Turunkan

sekali terhadap x didapat n(x + 1)n−1 =∑n

r=0

(nr

)rxr−1. Substitusikan untuk

x = 1 diperoleh n(1 + 1)n−1 =∑n

r=0

(nr

)r(1)r−1 atau

∑nr=0

(nr

)r = n2n−1.

Example. Buktikan bahwa∑n

r=1 r2(

nr

)= n(n + 1)2n−2.

Solution. Menurut koefisien binomial didapat (x+1)n =∑n

r=0

(nr

)xr. Turunkan

dua kali terhadap x didapat n(n−1)(x+1)n−2 =∑n

r=2

(nr

)r(r−1)xr−2. Kemudian

substitusikan x = 1 didapat:

n(n− 1)(1 + 1)n−2 =n∑

r=2

(n

r

)r(r − 1)

n(n− 1)2n−2 =n∑

r=2

(n

r

)r2 −

n∑r=2

(n

r

)r

n(n− 1)2n−2 +n∑

r=2

(n

r

)r =

n∑r=2

(n

r

)r2

Page 137: Contents · PDF filegai batu loncatan untuk membuktikan teorema-teorema lainnya. Sebuah akibat dari beberapa teorema atau lemma disebut dengan korolary. Sebuah pernyataan yang tidak

Chapter 9. Kombinatorika 131

Tambahkan n di kedua ruas

n + n(n− 1)2n−2 +n∑

r=2

(n

r

)r = n +

n∑r=2

(n

r

)r2

n(n− 1)2n−2 +n∑

r=1

(n

r

)r =

n∑r=1

(n

r

)r2

n(n− 1)2n−2 + n2n−1 =n∑

r=1

(n

r

)r2

Sehingga∑n

r=1

(nr

)r2 = n(n + 1)2n−2.

6.4 Prinsip Sarang Merpati

Prinsip sarang merpati (pigeon hole principle) juga dikenal dengan nama shoe

box argument atau dirichlet drawer principle.

Theorem 6.4.1 Jika terdapat lebih dari n barang yang didistribusikan ke dalam n buahkotak, maka sedikitnya satu kotak akan menerima lebih dari satu barang.

Contoh:

Selama bulan juni (30 april) Antony melakukan pertandingan catur sedikitnya

satu kali sehari. Banyak pertandingan selama bulan tersebut tidak lebih dari 45

kali. Tunjukkan bahwa terdapat periode dimana Antony melakukan tepat 14

pertandingan!

Jawab:

Misalkan ai menyatakan banyaknya pertandingan yang dilakukan Antony se-

lama i hari. Karena dalam sebulan banyaknya pertandingan tidak lebih dari 45

kali maka

0 < a1 < a2 < ... < a30 ≤ 45

Page 138: Contents · PDF filegai batu loncatan untuk membuktikan teorema-teorema lainnya. Sebuah akibat dari beberapa teorema atau lemma disebut dengan korolary. Sebuah pernyataan yang tidak

Chapter 9. Kombinatorika 132

Jumlahkan dengan 14 didapatkan

14 < a1 + 14 < a2 + 14 < ... < a30 + 14 ≤ 59

Dari ketaksamaan (1) dan (2) didapatkan 60 bilangan. Karena maksimal hanya

ada 59 bilangan maka setidaknya terdapat i dan j sedemikian hingga ai = aj +

14. Ekivalen dengan ai − aj = 14. Terbukti

6.5 Paritas

Prinsip ini digunakan untuk mengeliminasi kemungkinan-kemungkinan ter-

tentu dengan cara memperhatikan dua masalah saja, misalnya ganjil genap atau

hitam putih. Misal kedua bilangan adalah genap atau ganjil maka bilangan-

bilangan itu mempunyai paritas sama. Jika satu ganjil dan satunya genap maka

bilangan-bilangan itu mempunyai paritas yang berbeda.

Contoh:

Misalkan n > 1 adalah bilangan ganjil dan misalkan pula A adalah matriks

berukuran nxn yang simetris. Jika setiap baris dan kolom S merupakan suatu

permutasi dari 1, 2, ...n Buktikan bahwa setiap i, i = 1, 2, ...n akan muncul pada

diagonal utama!

Jawab:

Karena setiap baris dan kolom adalah permutasi dari 1, 2, 3, ...n akan muncul

sebanyak n kali dalam matriks A. Misalkan submatriks dari A yang terletak

dibawah dan diatas diagonal utama sebagai A1 dan A2. Andaikan terdapat bi-

langan i0 diantara 1, 2, 3, ...n yang tidak muncul di A1dan A2 haruslah sama.

Akibatnya banyaknya i0 yang muncul di A adalah genap, suatu kontradiksi.

Page 139: Contents · PDF filegai batu loncatan untuk membuktikan teorema-teorema lainnya. Sebuah akibat dari beberapa teorema atau lemma disebut dengan korolary. Sebuah pernyataan yang tidak

Chapter 9. Kombinatorika 133

6.6 Relasi Rekurensi

Definition 6.6.1 Diberikan suatu fungsi numerik a0, a1, . . . , ar, . . . untuk sebarang r.Suatu persamaan yang mengkaitkan ar dengan satu atau lebih ai, untuk i < r, disebutrelasi rekurensi atau persamaan beda (difference equations).

Contoh: Misalkan kita melempar sebuah koin n kali. Berapa banyaknya hasil

percobaan yang didalamnya sisi gambar tidak pernah muncul dua kali berturut-

turut? Jawab: Misalkan an menyatakan banyaknya hasil percobaan ke-n. Per-

hatikan bahwa untuk mendapatkan hasil percobaan sehingga sisi gambar tidak

boleh muncul dua kali berturut-turut maka

1. n− 1 barisan yang didalamnya sisi gambar tidak pernah muncuk dua kali

bertrut-turut pada barisan ke-n dipasangkan dengan sisi angka.

2. n− 2 barisan yang didalamnya sisi gambar tidak pernah muncul dua kali

berturut-turut pada barisab ke-(n − 1) dipasankan dengan sisi angka ke-

mudian barisan ke-n dipasangkan dengan sisi gambar.

6.7 Soal-soal dan Pembahasan

Bagian I

1. Berapa cara untuk bergerak diruang xyz dari titk (0, 0, 0) ke (4, 3, 5) se-

hingga langkah yang diambil adalah arah x positif, y positif dan z positif?

Pembahasan: Karena langkah yang diambil adalah arah x positif,y posi-

tif dan z positif, maka langkah dari (0, 0, 0) ke (4, 3, 5) adalah langkah

yang terpendek. Sehingga banyaknya cara untuk bergerak di ruang terse-

but yaitu harus terdiri dari 12 langkah dimana 4 langkah ke kanan(x), 3

Page 140: Contents · PDF filegai batu loncatan untuk membuktikan teorema-teorema lainnya. Sebuah akibat dari beberapa teorema atau lemma disebut dengan korolary. Sebuah pernyataan yang tidak

Chapter 9. Kombinatorika 134

langkah ke belakang y dan 5 langkah ke atas (z). Jadi banyaknya cara

adalah(124

)(83

)(55

)= 27720

2. Hitunglah berapa koefesien dari x102y98 dari ekspansi (2x− 5y)200!

Pembahasan:

Menurut teorema binomial untuk menghitung koefesien x102y98 dari ekspansi

(2x − 5y)200 yaitu dengan mengambil r = 102, sehingga didapatkan suku

ke-102, sehingga didapatkan suku ke-102 sebagai berikut(200102

)(2x)102(−5y)98

Jadi koefesien x102y98 adalah(200102

)(2x)102(−5y)98

3. Berapa banyak solusi bilangan asli untuk persamaan x1+x2+x3+...+xk =

n?

Pembahasan:

Dibentuk bilangan yi = xi − 1. Karena xi merupakan bilangna asli, maka

jelas bahwa yi merupakan bilangan bulat. Sehingga banyak solusi untuk

x1+x2+x3+...+xk = n ekivalen dengan banyak solusi persamaan berikut:

x1 + x2 + x3 + ... + xk = n

(x1 − 1) + (x2 − 1) + (x3 − 1) + ... + (xk − 1) = n− k

y1 + y2 + y3 + ... + yk = n− k

Sehingga banyaknya solusi bilangan bulat adalah(

n−k+k−1k−1

)=

(n−1k−1

)

4. Buktikan bahwa diantara n + 1 bilangan bulat yang dipilih terdapat dua

bilangan yan selisihnya hais dibagi n!

Pembahasan:

Misalkan bilangan yang dipilih tersebut adalah a1, a2, ..., an+1. Diambil se-

barang bilangan dari a1, a2, ..., an+1 misalkan a1. Dibentuk

a1 − a2,

a1 − a3,

...,

a1 − an+1

Page 141: Contents · PDF filegai batu loncatan untuk membuktikan teorema-teorema lainnya. Sebuah akibat dari beberapa teorema atau lemma disebut dengan korolary. Sebuah pernyataan yang tidak

Chapter 9. Kombinatorika 135

Jika salah satu dariselisih diatas habis dibagi n, maka pernyataan terbukti.

jika n selisih diatas tidak habis dibagi n, maka kemudian dibentuk

a1 − a2 = t1n + r1,

a1 − a3 = t2n + r2,

...,

a1 − an+1 = tnn + rn,

dengan 0 < ri < n. Karena terdapat n hasil bagi sedangkan banyaknya

bilangan 0 < ri < n hanya ada n− 1 bilangan, maka pasti terdapat dua bi-

langan yang mempunyai sisa hasil bagi yang sama. Akibatnya didapatkan

dua bilangan yang selisihnya habis dibagi n. Terbukti.

5. Diberikan himpunan n + 1 bilangan bulat yang kurang dari 2n. Buktikan

bahwa setidaknya terdapat satu anggota himpunan yang membagi habis

anggota himpunan yang lain!

Pembahasan:

Misalkan anggota himpunan tersbut adalah x1, x2, x3, ..., xn+1. Untuk se-

tiap i, xi = 2niiyi, dengan ni adalah bilangan bulat non-negatif dan yi adalah

bilangan ganjil. T = {yi : i = 1, 2, ..., n + 1}, maka T adalah koleksi n + 1

bilangan ganjil yang kurang dari 2n. Karena hanya terdapat n bilangan

ganjilyang kurang dari 2n, maka setidaknya terdapat dua bilangan yang

sama misalkan yi = yj, i < j. Akibatnya xi = 2niyi dan xj = 2njyj . Jika

ni ≤ nj maka xj membagi xi dan jika ni ≥ nj maka xi membagi xj . Ter-

bukti.

6. Misalkan x1, x2, x3, ..., xn adalah sebarang permutasi dari 1, 2, 3, ...n. Jika

n adalah bilangan ganjil buktikan bahwa perkalian (x1 − 1)(x2 − 2)(x3 −3)...(xn − n) adalah genap!

Pembahasan:

Andaikan hasil perkalian tersebut ganjil, maka tiap-tiap faktor tidak berni-

lai genap. Jadi (xk− k) bernilai ganjil untuk setiap k. Diketahui n bilangan

Page 142: Contents · PDF filegai batu loncatan untuk membuktikan teorema-teorema lainnya. Sebuah akibat dari beberapa teorema atau lemma disebut dengan korolary. Sebuah pernyataan yang tidak

Chapter 9. Kombinatorika 136

ganjil maka jumlah semua bilangan ganjil tersebut juga ganjil. Sedangkan

(x1 − 1)(x2 − 2)(x3 − 3)...(xn − n) = 0 bilangan genap. Kontradiksi, Jadi,

terbukti bahwa hasil perkalian tersebut bernilai genap.

7. Buktikan bahwa ada 2n− 2 bilangan yang terdiri dari atas n angka 1 dan 2

dengan masing-masing angka muncul sedikitnya sekali!

Pembahasan:

Karena banyaknya bilangan dengan angka 1 tidak pernah muncul adalah

1. Jadi banyaknya bilangan yang terdiri dari angka 1 dan 2 dengan masing-

masin angka muncul sedikitnya sekali adalah 2n− 1− 1 = 2n− 2 Terbukti.

8. Misakan T adalah sebuah segitiga sama sisi dengan panjang 1. Tunjukkan

bahwa jika terdapat 5 titik pada T maka dua diantaranya akan berjarak

tidak lebih dari 12!

Pembahasan:

Bentuk tiga garis pada segitiga sama sisi tersebut sehingga tiga garis terse-

but akan menghasilkan empat buah segitiga kecil sama sisi. Jelas bahwa

maksimal panjang segitiga kecil sama sisi tersebut adalah 12. karena terda-

pat lima titik dan empat segitiga kecil, maka pasti terdapat dua titik yang

terletak pada segitiga kecil yang sama. Terbukti.

9. Tentukan banyaknya daerah yang dibentuk oleh n garis pada bidang datar.Pembahasan:

Diasumsikan tidak ada dua garis yang sejajar dan tidak ada tiga garis yang

berpotongan pada titik yang sama. Misalkan an menyatakan banyaknya

daerah yang dicari. Karena garis ke-n dipotong oleh n − 1 garis sebelum-

nya menjadi n ruas garis dan setiap ruas garis memotong suatu daerah

menjadi dua daerah baru maka didapatkan an = an−1 + n dengan a0 =

1, a2 = 2, a3 = 4, a4 = 7. Persamaan yang memenuhi syarat ini adalah

an = 1 + 12n(n + 1)

10. Misalkan dimiliki perangko dengan nilai 5 sen dan 3 sen. Tunjukkan bahwa

Page 143: Contents · PDF filegai batu loncatan untuk membuktikan teorema-teorema lainnya. Sebuah akibat dari beberapa teorema atau lemma disebut dengan korolary. Sebuah pernyataan yang tidak

Chapter 9. Kombinatorika 137

dapat dibentuk perangko senilai tepat 8 sen atau lebih!Pembahasan:

(a) Perangko senilai 8 sen jelas dapat dibentuk

(b) Perangko senilai 9 sen jelas dibentuk dengan 3 perangko nil ai 3 sen

dan perangko senilai 10 senjelas dibentuk dengan 2 perangko senilai

n > 10 dilakukan dengan cara berikut:

i. jika perangk n sen terdiri dari minimal 3 perangko bernilai 3 sen,

maka jika perangko ini diganti 2 perangko masing-masing berni-

lai 5 sen maka akan diperoleh perangko bernilai n + 1 sen.

ii. jika perangko k sen terdiri dari minimal 1 perangko bernilai 5

sen, maka jika perangko 5 sen diganti dengan 2 perangko masing-

masing bernilai 3 sen akan diperoleh perangko bernilai n + 1 sen.

Bagian II

1. Berapa banyaknya bilangan bulat positif yang merupakan faktor dari 30030?

Pembahasan:

Karena 30030 = 2x3x5x7x11x13, dan hasil kali r bilangan 2, 3, 5, 7, 11, 13, 1 ≤r ≤ 6 juga merupakan faktor dari 30030, maka diperoleh banyaknya faktor

adalah∑6

n=0

(6n

)= 26

2. Misalkan S sebuah himpunan dengan n unsur. Buktikan bahwa banyaknya

himpunan bagian bagi S ada 2n!

Pembahasan:

Banyaknya unsur di dalam suatu himpunan bagian adalah 0 atau 1 atau

2,... atau n. Selain itu banyaknya himpunan bagian yang memiliki r unsur

adalah(

nr

)untuk r = 0, 1, 2, ...n. Dengan demikian, banyaknya himpunan

bagian ialah∑n

n=0

(nr

)= 2n.Terbukti.

3. Sebuah kata biner yang panjangnya n adalah suatu barisan angka yang

terdiri atas angka 0 atau 1. Berapa banyak kata biner dengan panjang 10

Page 144: Contents · PDF filegai batu loncatan untuk membuktikan teorema-teorema lainnya. Sebuah akibat dari beberapa teorema atau lemma disebut dengan korolary. Sebuah pernyataan yang tidak

Chapter 9. Kombinatorika 138

yang diawali dengan tiga angka 0 atau diakhiri dengan dua angka 1?

Pembahasan:

Banyak kata biner dengan panjang 10 yang diawali dengan tiga angka 0

atau diakhiri dengan dua angka 1 = (banyak kata biner yang diawali den-

gan tiga angka 0) + (banyak kata biner yang diakhiri dengan dua angka

1) - (banyak kata biner yang diawalai dengan tiga angka 0 dan diakhiri

dengan dua angka 1). Kemudian, untuk mendapatkan kata biner yang

diawali dengan tiga angka 0 adalah 1.1.1.2.2.2.2.2.2.2 = 27, diakhiri den-

gan dua angka 1 adalah 2.2.2.2.2.2.2.2.1.1 = 28 dan diawali tiga angka 0

dan diakhiri dua angka 1 adalah 1.1.1.2.2.2.2.2.1.1 = 25. Dengan demikian

didapatkan 27 + 28 − 25.

4. Hitunglah berapa nilai dari∑k=0

1003

(20072k−1

)!

Pembahasan:

Perhatikan bahwa binomial untuk x = 1 dan y = 1 adalah

k=0∑2007

(2007

k

)= (1 + 1)2007 = 22007

.....(1)

Sedangkan binomial untuk x = 1 dan y = −1 adalah

k=0∑2007

(2007

k

)(1)2007−k(−1)k = (1− 1)2007 = 02007 = 0

Sehinggak=0∑2007

(2007

k

)(−1)k = 0

Akibatnya∑genap

(2007

k

)(−1)k +

ganjil

(2007

k

)(−1)k = 0

Page 145: Contents · PDF filegai batu loncatan untuk membuktikan teorema-teorema lainnya. Sebuah akibat dari beberapa teorema atau lemma disebut dengan korolary. Sebuah pernyataan yang tidak

Chapter 9. Kombinatorika 139

Ekivale dengan∑genap

(2007

k

)+

ganjil

(2007

k

)= 0

∑genap

(2007

k

)=

ganjil

(2007

k

)

.....(2)

Sehingga dari (1) dan (2) didapatkan

1003∑

k=0

(2007

2k + 1

)=

1003∑

k=0

(2007

2k + 1

)= 22007

21003∑

k=0

(2007

2k + 1

)= 22007

Diperoleh1003∑

k=0

(2007

2k + 1

)= 22006

5. Buktikan bahwa∑n

r=1 r2(

nr

)= n(n + 1)2n−2!

Pembahasan:

Menurut koefisien binomial didapatkan (x + 1)n =∑n

r=0

(nr

)xr Jika ruas

kanan dan kiri diturunkan dua kali terhadap x maka didapatkan

n(x + 1)n−1 =n∑

r=1

(n

r

)rxr−1

n(n− 1)(x + 1)n−2 =n∑

r=2

(n

r

)r(r − 1)xr−2

Page 146: Contents · PDF filegai batu loncatan untuk membuktikan teorema-teorema lainnya. Sebuah akibat dari beberapa teorema atau lemma disebut dengan korolary. Sebuah pernyataan yang tidak

Chapter 9. Kombinatorika 140

Kemudian subsitusikan x = 1 sehingga didapakan

n(n− 1)2n−2 =n∑

r=2

(n

r

)r(r − 1)

n(n− 1)2n−2 =n∑

r=2

(n

r

)r2 −

n∑r=2

(n

r

)r

n(n− 1)2n−2 −n∑

r=2

(n

r

)r =

n∑r=2

(n

r

)r2

Tambahkan n diruas kanan dan kiri sehingga diperoleh

n + n(n− 1)2n−2 +n∑

r=2

(n

r

)r = n +

n∑r=2

(n

r

)r2

Dari sini diperoleh

n(n− 1)2n−2 +n∑

r=1

(n

r

)r = n +

n∑r=2

(n

r

)r2

n(n− 1)2n−2 + n2n−1 =n∑

r=1

(n

r

)r2

n(n− 1)2n−2 =n∑

r=1

(n

r

)r2

6. Misalkan terdapat lima titik P1, P2, P3, P4, danP5 pada bidang dimana masing-

masing mempunyai koordinat bilangan bulat. Buktikan bahwa terdapat

sedikitnya sepasang titik (Pi, Pj) dengan i 6= j sedemikian hingga segmen

garis (Pi, Pj) memuat sebuah titik Q yang mempunyai koordinat bilangan

bulat dimana Q 6= P atau Q 6= Pj!

Pembahasan:

Berdasarkan ganjil genapnya, terdapat 4 jenis koordinat untuk kelima titik

Page 147: Contents · PDF filegai batu loncatan untuk membuktikan teorema-teorema lainnya. Sebuah akibat dari beberapa teorema atau lemma disebut dengan korolary. Sebuah pernyataan yang tidak

Chapter 9. Kombinatorika 141

ini yaitu (genap, genap), (ganjil,ganjil), (genap, ganjil) dan (ganjil, genap).

Karena terdapat lima buah titik maka menurut prinsip sangkar merpati,

setidaknya terdapat dua buah titik yang mempunyai jenis yang sama. mis-

alkan kedua titik ini adalah Pi = (xi, yj)danPj = (xj, yj) dengan i 6= j.

Karena xi da xj paritas yang sama maka xi + xj adalah genap, demikian

juga dengan yi + yj . Akibatnya Q =

(xi+xj

2,

yi+yj

2

)yang merupakan koor-

dinat titik tengah segmen garis mempunyai koordinat bilangan bulat.

7. Misalkan A adalah sebuah himpunan yang beranggotaan 10 bilangan bu-

lat positif yang kurang dari 107. Buktikan bahwa terdapat dua suphim-

punan saling lepas dari A yang hasil tamabah dari anggota-anggotanya

adalah sama!

Pembahasan:

Himpunan A yang hasil tambah dari anggotanya terbesar adalah A = den-

gan hasil tambah sama dengan 1050. Buatlah 1060 kotak dengan label

0 sampai dengan 1050. Sekarang perhatikan sebarang himpunan A dan

semua subhimpunan, tulis hasil tambah anggota-anggota pada selembar

kertas dan masukkan kertas tersebut pada kotak yang sesuai. Terdapat

1024 kertas dan 1060 kotak. Menurut prinsip sangkar merpati maka terda-

pat sebuah kotak yang berisi sedikitnya dua kertas. Hal ini berarti jumlah

anggota dari dua subhimpunan ini adalah sama. Perhatikan bahwa ke-

dua sub himpunan ini bisa tidak saling lepas. Tetapi jika demikian kita

dapat menghapus anggota sekutu dari dua sub himpunan ini untuk men-

dapatkan dua sub himpunan lain yang saling lepas dan hasil tambahan

anggota anggotanya sama.

8. Misalkan n > 1 adalah bilangan bulat ganjil, dan misalkan A adalah ma-

triks berukuran nxn yang simetris. Jika setiap baris dan kolom A meru-

pakan suatu permutasi dari 1, 2, ...n, buktikan bahwa setiap i, i = 1, 2, ...n

Page 148: Contents · PDF filegai batu loncatan untuk membuktikan teorema-teorema lainnya. Sebuah akibat dari beberapa teorema atau lemma disebut dengan korolary. Sebuah pernyataan yang tidak

Chapter 9. Kombinatorika 142

akan muncul pada diagonal utama A!

Pembahasan:

karena setiap baris dan kolom adalah permutasi dari 1, 2, ...n maka setiap

bilangan i, i = 1, 2, ...n akan muncul sebanyak n kali dalam matriks A. Se-

but submatriks dari A yang terletak dibawah dan diatas diagonal utama

sebagai A1 dan A2. Andaikan terdapat bilangan i0 diantara 1, 2, ...n yang

tidak muncul pada diagonal utama A. Karena A simetris maka banyaknya

i0 yang munculnya di A adalah genap. Suatu kontradiksi.

9. Misalkan P1, P2, P3, P4 dan P5 terletak didalam sebuah persegi yang pan-

jang sisinya adalah 1. Notasikan dij adalah jarak antara titik Pi dan Pj .

Tunjukkan bahwa terdapat dua titik yang jaraknya kurang dari√

22

!

Pembahasan:

Bagi persegi tersebut menjadi empat buah persegi kecil yang sama sisi.

Akibatnya menurut pigeon hole setidaknya terdapat dua titik yang ter-

letak pada persegi kecil yang sama. Karena panjang sisi persegi adalah 1,

maka panjang sisi persegi kecil adalah 12. Menurut teorema phytagoras,

jarak maksimal dari titik sudut ke sudut yang lain yaitu√

22

. Terbukti.

10. Berapakah banyaknya bilangan bulat positif diantara 1 dan 1000 yang tidak

habis dibagi 2 dan tidak habis dibagi 5?

Pembahasan:

Misalkan S = {1, 2, 3, ..., 1000}, A = {xεS, 2|x} dan {B = {xεS, 5|x}. Akan

Page 149: Contents · PDF filegai batu loncatan untuk membuktikan teorema-teorema lainnya. Sebuah akibat dari beberapa teorema atau lemma disebut dengan korolary. Sebuah pernyataan yang tidak

Chapter 9. Kombinatorika 143

dicari |A′ ∩B′|. Menurut hukum De-Morgan, maka didapatkan

|A′ ∩B′| = |(A ∪B)′|= |S| − |(A ∪B)|= |S| − (|A|+ |B| − |A ∩B|)= 1000−

([1000

2

])+

([1000

5

])−

([1000

10

])

= 400

Jadi banyaknya bilangan bulat positif ada 400 bilangan.

11. Berapakah banyaknya penyelesaian bilangan bulat untuk persamaan x1 +

x2 + x3 = 24 dengan 1 ≤ x1 ≤ 5, 12 ≤ x2 ≤ 18,−1 ≤ x3 ≤ 5?

Pembahasan:

Notasikan y1 = x1−1, y2 = x2−12, y3 = x3 +1, sehingga persamaan di atas

menjadi y1 + y2 + y3 = 12 dengan penyelesaikan bilangan bulat nonnegatif

dan y1 ≤ 4, y2 ≤ 6, y3 ≤ 13. Misalkan A,B dan C berturut-turut adalah

himpunan semua penyeesaian sedemikian hingga y1 > 4, y2 > 6, y3 >

13. Dengan demikian, banyaknya solusi nonnegatif untuk y1 + y2 + y3 =

12 adalah(12−1+3

12

)= 19, |A| =

(3−1+7

7

), |B| =

(3−1+5

5

), |C| = 0 dan juga

diperoleh |A ⋂B| = 1, |A ⋂

C| = |B ⋂C| = |A⋂

B⋂

C| = 0. Jadi, dari sini

menurut PIE didapatkan 91− (36 + 21 + 0)− 0 = 35.

12. Buktikan bahwa banyaknya pemetaan dari M = {1, 2, ...m} ke N = {1, 2, ..., n}adalah

nm −(

n

1

)(n− 1)m +

(n

2

)(n− 2)m − ... + (−1)n−11m

Pembahasan:

Banyaknya pemetaan dari M ke N adalah nm. Jika Ai adalah himpunan

Page 150: Contents · PDF filegai batu loncatan untuk membuktikan teorema-teorema lainnya. Sebuah akibat dari beberapa teorema atau lemma disebut dengan korolary. Sebuah pernyataan yang tidak

Chapter 9. Kombinatorika 144

pemetaan dari M ke N sehingga iεN tidak mempunyai prapeta, maka

banyaknya pemetaan dari M ke N yang tidak pada adalah |A1 ∪ A2 ∪ ... ∪An|.Perhatikan bahwa :

(a) Untuk setiap iεN, |Ai| = (n− 1)m. Akibatnya∑i=1

n |Ai| =(

n1

)(n− 1)m

(b) Untuk setiap i1, i2εN, |Ai1 ∪ Ai2| = (n − 2)m. Akibatnya∑i1<i2

n |Ai1 ∪Ai2| = (n− 2)m

Demikian seterusnya sehingga untuk setiap untuk i1, i2...in−1εN , akibat-

nya

Ai1 ∩ Ai2 ∩ .... ∩ Ain| = (n(n− 1))m = 1m

Jadi

|A1∪A2∪ ...∪An| =(

n

1

)(n−1)m−

(n

2

)(n−1)m− ...− (−1)n−1

(n

n− 1

)1m

Akibanya banyaknya pemetaan dari M ke N adalah

nm−|A1∪A2∪...∪An| =(

n

1

)(n−1)m−

(n

2

)(n−1)m−...−(−1)n−1

(n

n− 1

)1m

13. Sebuah pesta dihadiri oleh n orang dan setiap hadirin meninggalkan top-

inya dibagian penyimpanan. Dalam berapa banyak cara topi-topi itu dikem-

balikan sehingga tidak seorang pun menerima topinya kembali?

Pembahasan:

Misalkan Ai melambangkan hmpunan kemungkinan hadirin ke-i menda-

patkan kembali topinya, untuk setiap i. Banyaknya cara pengembalian

sehingga hadirin ke-i mendapatkan kembali topinya ada (n − 1)!. Jadi ,

|Ai = (n − 1)!. Selanjutnya untuk setiap pilihan k bilangan bulat berbeda,

misalkan Ai1i2i3...ik = Ai1 ∪Ai2 ∪Ai3 ...∪Aik adalah semua himpunan kemu-

ngkinan pengembalian sehingga hadirin ke-ik mendapatka kembali top-

inya. Dari sini dieroleh |Ai1i2i3...ik | = (n− k)!. pencacahan ini tidak bergan-

Page 151: Contents · PDF filegai batu loncatan untuk membuktikan teorema-teorema lainnya. Sebuah akibat dari beberapa teorema atau lemma disebut dengan korolary. Sebuah pernyataan yang tidak

Chapter 9. Kombinatorika 145

tung pada pemilihan i1, i2, i3...ik. Karena banyaknya cara mengabilk bi-

langan adalah(

nk

). sehingga Sk =

∑ |Ai1i2i3...ik | =(

nk

)! = n!

k!. Selanjutnya

menurut prinsip inklusi eksklusi diperoleh

|n⋂

i=1

Ai| = n!−(

n

1

)(n− 1) + ... + (−1)k

(n

k

).(n− k)! + ... + (−1)n

= n!

[1− 1

1!+

1

2!+ ... +

1

n!

]

14. Letakkan sebuah kuda catur didalam papan catur berukuran 7x7. apakah

mungkin setiap kuda catur secara serempak dapat bergerak menurut atu-

ran yang berlaku?

Pembahasan:

Asumsikan papan catur tersebut diwarnai dengan pola seperti umumnya.

Papan tersebut mempunyai 49 kotak, misalkan 24 diantaranya putih dan

25 warna hitam. Dengan mempertimbangkan kedua catur yang terletak

dalam kotak hitam, jika setiap kuda catur membuat gerakan sesuai aturan,

maka kuda catur tersebut harus bergerak menuju 25 kotak putih. Karena

hanya ada 24 kotak putih yang tersedia, maka ada sebuah gerakan yang

tidak dapat dilakukan. Jadi kuda catur tidak dapat bergerak secara serem-

pak.

15. Letakkan sebuah kuda catur ke dala papan catur berukuran 4xn. Apakah

mungkin kuda catur dapat menempati setiap kotak dan kembali ke kotak

awal dengan gerakan berurutan dalam 4n kali gerakan?

Pembahasan:

Sebelumnya perhatikan untuk kasus papan catur berukuran 7x7. Misalkan

pada awalnya kuda catur terletak dikotak warna hitam. Pada gerakan

pertama, kuda catur akan bergerak menempati kotak berwarna hitam.

Page 152: Contents · PDF filegai batu loncatan untuk membuktikan teorema-teorema lainnya. Sebuah akibat dari beberapa teorema atau lemma disebut dengan korolary. Sebuah pernyataan yang tidak

Chapter 9. Kombinatorika 146

Demikian seerusnya. Sehingga agar kuda catur dapat kembali menem-

pati posisinya semula dibutuhkan sejumlah gerakan genap. Karena di-

iginkan kda catur dapat menemati setiap kotak maka dibutuhkan 49 ger-

akan harus menempati bilangan genap. Jadi dapat disimpulkan bahwa

kuda catur tidak dapat menempati setipa kotak dan kembali ke kotak awal

sesuai dengan aturan.

Selanjutnya untuk kasus diatas, perhatikan bahwa 4xn adalah bilangan

genap. Untuk menanganinya, warnai papan catur dengan pola yang berbeda,

contohnya seperti gambar dibawah. Perhatikan bahwa gerakan kuda catur

dari kotak putih diatas dan dibawah akan mengantarkan kekotak putih

baris kedua dan ketiga. Demikian sebaliknya gerakan dari kotak putih

baris kedua dan ketiga akan mengantarkan ke kotak putih baris pertama

dan keempat. Terdapat n kotak putih di baris pertama dan keempat yang

hanya dapat dicapai dari n kotak putih baris kedua dan ketiga. Hal ini

berakibat jalur kuda catur tidak akan bergerak dari kotak putih ke kotak

hitam. Jadi, kuda catur tidak mungkin dapat menempati setiap kotak dan

kembali ke posisi semula.

16. Misalkan a1, a2, ..., a2n+1 adalah himpunan bilangan bulat yang mempun-

yai sifat jika salah satunya ada yang dipindahkan maka bilangan yang ter-

sisa dapat dibagi dalam dua himpunan bilangan bulat yang jumlahnya

sama. Buktikan bahwa a1 = a2 = ... = a2n+1

Pembahasan:

Page 153: Contents · PDF filegai batu loncatan untuk membuktikan teorema-teorema lainnya. Sebuah akibat dari beberapa teorema atau lemma disebut dengan korolary. Sebuah pernyataan yang tidak

Chapter 9. Kombinatorika 147

Misalkan A = a1 + a2 + ... + a2n+1. Dari claim diatas berakibat bahwa

A−ai merupakan bilangan genap untuk setiap i. Jadi semua bilangan bulat

a1, a2, ..., a2n+1 mempunyai paritas yang sama. Notasikan a adalah bilan-

gan bulat terkecil dari a1, a2, ..., a2n+1. Kemudian dibentuk bi = ai − a un-

tuk setiap i. Jelas bahwa b1, b2, ..., b2n+1 juga memenuhi sifat diatas. Karena

b1, b2, ..., b2n+1 mempunyai paritas yang sama dan salah satu dan salah sat-

unya merupakan bilangan nol, maka paritas dari b1, b2, ..., b2n+1 adalah bi-

langan genap. Andaikan terdapat bilangan diantara b1, b2, ..., b2n+1 yang

bukan bilangan nol. Notasikan k adalah bilangan bulat terbesar sedemikian

hingga 2k membagi habis setiap bi. Dibentuk ci = bi

2k untuk setiap i. Jelas

bahwa ci, c2, ..., c2n+1 juga memenuhi sifat diatas. Karena k adalah bilangan

bulat terbesar yang membagi habis setiap bi, maka terdapat ci yang meru-

pakan bilangan ganjil sedangkan salah satu dari ci, c2, ..., c2n+1 merupakan

bilangan genap. Jadi ci, c2, ..., c2n+1 tidak mempunyai paritas yang sama.

Kontradiksi dengan yang diketahui bahwa ci, c2, ..., c2n+1 memenuhi sifat

diatas. Jadi tidak ada bilangan di antara b1, b2, ..., b2n+1 yang bukan bilan-

gan nol, sehingga terbukti bahwa a1 = a2 = ... = a2n+1

17. Misalkan an adalah barisan yang terdiri dari n suku dimana suku-suku

1, 2atau3. Jika barisan ini tidak memiliki dua angka 1 yang berurutan, hi-

tunglah berapa banyak barisan a10!

Pembahasan:

Misalkan Sn menyatakan banyaknya barisan an yang tidak memiliki dua

angka satu yang berurutan. Perhatikan bahwa Sn dapat dibagi menjadi

dua bagian yaitu Sn,1 yaitu banyaknya barisan an yang diawali dengan

angka 1 dan tidak memiliki dua angka 1 dan Sn,2,3 adalah banyaknya barisan

an yang diawali dengan angka 2 dan 3 dan tidak memiliki dua angka 1.

Jelas bahwa Sn,1 diperoleh dengan meletakkan angka 2 atau 3 di awal

unsur-unsur Sn−1.Disamping itu pada setiap unsur di dalam Sn,2,3 angka

Page 154: Contents · PDF filegai batu loncatan untuk membuktikan teorema-teorema lainnya. Sebuah akibat dari beberapa teorema atau lemma disebut dengan korolary. Sebuah pernyataan yang tidak

Chapter 9. Kombinatorika 148

kedua pasti 1 dan barisan n − 2 bersal dari Sn−2. Dari sini diperoleh Sn =

Sn−1 + 2Sn−2

18. Notasi Qn menyatakan banyaknya cara meletakkan benteng catur pada

papan catur berukuran nxn sedemikian hingga pengaturannya simetri ter-

hadap diagonaldari sudut kiri bawah dengan sudut kanan atas. Buktikan

bahwa Qn = Qn−1 + (n− 1)Qn−2!

Pembahasan:

Sebuah benteng pada kolom pertama mungkin atau tidak mungkin men-

duduki kotak pada sudut kiri bawah. Jika iya, maka terdapat Qn−1 cara

meletakkan sisa n− 1 benteng. Jika tidak, maka benteng dapat diletakkan

di n − 1 kotak pada kolom pertama. Peletakan benteng yang pertama

menentukan lokasi kesimettrisannya pada baris pertama. n − 2 benteg

sisanya kemudian diletakkan dengan Qn−2 benteng sisanya kemudian da-

pat diletakkan dengan Qn−2 cara. Dari sini maka didapatkan Qn = Qn−1 +

(n− 1)Qn−2.

19. Sebuah koin diundi n kali. Berapakah peluang didapatkan dua kepala

secara berurutan?

Pembahasan:

Notasikan Pn sebagai peluang bahwa dua kepla secara berurutan tidak

muncul dalam n lemparan. Jelas bahwa P1 = 1, P2 = 34. Untuk n > 2,

maka ada kasus:

(a) Jika lemparan pertama adalah ekor, maka dua ke[ala secara berurutan

tidak muncul di sisa n− 1 undian berikutnya dengan peluang Pn−1.

(b) Jika lemparan pertama muncul kepala, maka pada undian kedua harus

kepala untuk menghindari dua kepala muncul berurutan. Kemudian

dua kepala tidak akan muncul pada n−1 sisa undian berikutnya den-

gan peluang Pn−2.

Page 155: Contents · PDF filegai batu loncatan untuk membuktikan teorema-teorema lainnya. Sebuah akibat dari beberapa teorema atau lemma disebut dengan korolary. Sebuah pernyataan yang tidak

Chapter 9. Kombinatorika 149

Dari sini didapatkan Pn = 12Pn−1 + 1

4Pn−2, n > 2 transformasi ini dapat

dirubah kedalam bentuk rekurensi yang lebih umum dengan mengalikan

2n sehingga didapatkan 2nPn = 2n−1Pn−1 + 2n−2Pn−2 Misalkan Sn = 2nPn,

akan didapatkan Sn = Sn−1 + Sn−2

20. Misalkan n sebuah bilangan bulat yang lebih besar atau sama dengan 3.

Berikan suatu tafsiran kombinatorial langsung bagi identitas berikut

= 3

(n + 1

4

)

Pembahasan:

Ambil n titik pada sebuah lingkaran yang berpusat di O. Banyaknya tali

busur yang terbentuk dari titik ini adalah(

n2

)dan banyaknya pasangan

tali busur adalah ... Selanjutnya perhatikan n + 1 titik termasuk n titik

yang diberika dan titik pusat O. Dapat diambil 4 dari n + 1 titik ini untuk

meperoleh(

n+14

)kombinasi. Jika kombinasi demikian ini mengandung O,

maka akan diperoleh 6 ruas garis yang 3 diantaranya adalah jari-jari dan 3

lainnya tali busur. Ketiga tali busur ini kemudian menhasilkan(32

)pasan-

gan tali busur dengan tepat satu titik sekutu terhadap suatu pasanga yang

dihasilkan. Misalkan {O,A, B,C} menghasilkan tali busur AB, AC, BC

dan tiga pasangan {AB,AC}, {AB, BC} dan {AC,BC}. Selanjutnya su-

atu himpunan 4 titik tidak termasuk O menghasilkan(42

)tali busur dan(

62

)pasangan tali busur. Dari 15 pasangan tali busur ini, terdapat 3 pasan-

gan tali busur yang tidak memiliki sekutu. Misalkan {O, A,B,C} meng-

hasilkan tiga pasangan tali busur {AB,AC}, {AB, BC} dan {AC, BC}.